You are on page 1of 196

CHAPTER

1
NUMBER SYSTEMS

Points to Remember : 1. Number used for counting 1, 2, 3, 4, ... are known as Natural numbers. 2. All natural numbers together with zero i.e. 0, 1, 2, 3, 4, ..... are known as whole numbers. 3. All natural numbers, zero and negative numbers together i.e. ...., 4, 3, 2, 1, 0, 1, 2, 3, 4, ... are known as Integers. p 2 5 4 , 4. Rational Numbers : Numbers of the form where p, q both are integers and q 0. For e.g. , q 3 7 1 etc. 5. Every rational number have either terminating or repeating (recurring) decimal representation. Terminating Repeating (Recurring) For eg. 2 0.4 , 13 3.25 etc. 5 4 here, prime factors of denominator are 2 and 5 only. For e.g. 1 0.333..... 0.3 3 15 = 2.142857142857... 7

2.142857 etc.

6. There are infinitely many rational numbers between any two given rational numbers.
p 7. Irrational Numbers : Numbers which cannot be written in the form of q , where p, q are integers and

q 0. For e.g.
2 , 3 , 17 , , 0.202202220 3 9 etc. ......,

7 8. Real numbers : Collection of both rational and irrational numbers. For e.g. 3, , 0, 2 , 5 , etc. 5

9. Every real number is represented by a unique point on the number line. Also, every point on the number line represents a unique real number. 10. For every given positive real number x, we can find 11. Identities related to square root : Let p, q be positive real numbers. Then, (i)
pq p. q

. x geometrically.

(ii)

p q

p q

;q 0

(iii) ( p q ) ( p q ) p q (i) xp xq = xp+q (iii) ( x p ) q x pq MATHEMATICSIX

(iv) ( p q ) 2 p 2 pq q (ii) x p x q x p q (iv) x p . y p ( xy ) p NUMBER SYSTEMS 1

12. Laws of Radicals : Let x, y > 0 be real numbers and p, q be rationals. then

ILLUSTRATIVE EXAMPLES
Example 1. Find six rational numbers between 3 and 4. NCERT. Solution. We know that between two rational numbers a and b, such that a < b, there is a rational number a b . 2 1 7 A rational number between 3 and 4 is (3 4) . 2 2 Now, a rational number between 3 and
1 7 1 6 7 13 7 . is 3 2 2 2 2 4 2

A rational number between

17 1 7 8 15 7 . and 4 is 4 22 2 2 4 2 1 13 1 12 13 25 13 is 3 2 4 2 4 8 4

Also, a rational number between 3 and

A rational number between

1 15 1 15 16 31 15 and 4 is 4 2 4 4 2 4 8 1 31 1 31 32 63 31 and 4 is 4 2 8 8 2 8 16

A rational number between 3

25 13 7 15 31 63 4 8 4 2 4 8 16 This can be represented on number line as follows :

OR (without using formula)


10 40 We have, 3 3 10 30 and 4 4 10 10 10 10 We need to find six rational numbers between 3 and 4 i.e.
40 31 32 33 34 35 36 30 , , . Ans. and , which are , , , 10 10 10 10 10 10 10 10

Example 2. Find four rational numbers between 1 and 4 . 4 3 Solution.


1 1 3 3 4 4 4 16 and 4 4 3 12 3 3 4 12

4 rational numbers between 2

3 16 1 4 4 5 6 7 and i.e. and are , , , . 12 12 4 3 12 12 12 12

NUMBER SYSTEMS

MATHEMATICSIX

Example 3. Express 0.12 in the form of rational number, Solution. Let x 0.12 0.121212 ..... multiplying both sides by 100, we get 100 x = 12.1212..... Subtracting (1) from (2), 100 x x = 12.1212..... 0.1212.... 99 x = 12 x Example 4. Represent Solution.
12 4 Ans. 99 33

p. q

...(1) ...(2)

3 on the number line.

Let XOX be number line with O as origin. Let OA = 1 unit. Draw AB OA such that AB = 1 unit. Join OB. Then using Pythagoras theorem, In OAB OB OA 2 AB2 12 12 2 units Again, draw DC OB such that BC = 1 unit. Join OC. then, OC OB 2 BC 2 ( 2 ) 2 (1) 2 3 units . With O as centre and OC as radius, draw on arc, meeting OX at

P. Then OC = OP = 3 units. Example 5. State whether the following statements are true or false. Justify your answers. (i) Every irrational number is a real number. (ii) Every point on the number line is of the form Solution. . m , where m is a natural number. (iii) Every real number is an irrational number NCERT (i) True, since collection of real numbers is made up of rational and irrational numbers. (ii) False, no negative number can be the square root of any natural number. (iii) False, for example 5 is a real but not irrational. Example 6. Write the following in decimal form and say what kind of decimal expansion each as: 36 1 3 329 1 2 (iii) 4 (vi) (ii) (iv) (v) NCERT (i) 100 8 13 400 11 11 Solution.
36 0.36 Terminating decimal expansion. 100 1 , by long division, we have : (ii) Consider 11 0.090909

(i)

11 1.000000

99 100 99 100 99 1 MATHEMATICSIX NUMBER SYSTEMS 3

1 0.090909...... 0.09, which is non-terminating and repeating decimal expansion. 11

1 4 8 1 33 (iii) 4 . By long division, we have 8 8 8

4.125 8 33.000

32 10 8 20 16 40 40 0 33 4.125, terminating decimal expansion. 8 (iv) Consider,


3 by long division, we have 13

0.23076923.... 13 3.0000000

26 40 39 100 91 90 78 120 117 30 26 40 39 1


3 0.23076923....... 0.230769, which is non-terminating and repeating decimal expan13 sion.

NUMBER SYSTEMS

MATHEMATICSIX

(v) Consider,

2 , by long division, we have 11

0.181818 11 2.00000

11 90 88 20 11 90 88 20 11 90 88 2
2 0.181818..... 0.18 , which is non-terminating and repeating decimal expansion. 11
329 , by long division, we have 400

(vi) Consider,
0.8225 400 329.0000

3200 900 800 1000 800 2000 2000 0


329 0.8225, which is terminating decimal expansion. 400

Example 7. What can be the maximum number of digits be in the repeating block of digits in the decimal expansion of
1 ? Perform the division to check your answer. . 17

NCERT

MATHEMATICSIX

NUMBER SYSTEMS

Solution.

0.588235294117647.... 17 1.0000000000000000

85 150 136 140 136 40 34 60 51 90 85 50 34 160 153 70 68 20 17 30 17 130 119 110 102 80 68 120 119 1 Thus,
1 0.588235294117647 7
1 are 15. 17

The maximum number of digits in the quotient while computing

p Example 8. Look at several examples of rational numbers in the form q (q 0) , where p and q are integers

Solution. 6

with no common factors other than 1 and having terminating decimal representations (expansions). Can you guess what property q must satisfy? NCERT Let us consider various such rotational numbers having terminating decimal representation. NUMBER SYSTEMS MATHEMATICSIX

1 3 5 0.5 ; 0.75; 0.625 2 4 8 39 11 43 1.56; 0.088; 0.215 etc. 25 125 200 from the examples shown above, it can be easily observe that, If the denominator of a rational number in standard form has no prime factors other than 2 or 5 or both, then and the only then it can be represented as a terminating decimal. Example 9. Visualise 3.765 on the number line, using successive magnification. NCERT Solution. We know that 3.765 lies between 3 and 4. We divide portion of number line between 3 and 4 in 10 equal parts i.e. 3.1, 3.2, ....., 3.9 and then look at the interval [3.7, 3.8] through a magnifying glass and observe that 3.765 lies between 3.7 and 3.8 (see figure).

Now, we imagine that each new intervals [3.1, 3.2], [3.2, 3.3], ...... , [3.9, 4] have been sub-divided into 10 equal parts. As before, we can now visualize through the magnifying glass that 3.765 lies in the interval [3.76, 3.77]. (see figure).

Again, 3.765 lies between 3.76 and 3.77. So, let us focus on this portion of the number line, and imagine to divide it again into 10 equal parts. The first mark represents 3.761, second mark represents 3.762, and so on. So, 3.765 is the 5th mark in these subdivisions. Example 10. Recall, is defined as the ratio of the circumference (say c) of a circle to its diameter (say d).
c . This seem to contradict the fact that is irrational. How will you resolve this d contradiction? NCERT Solution. There is no contradiction. Remember that when you measure a length with a scale or any other device, you only get on approximate rational value. So, you may not realise that either c or d is irrational. Example 11. Simplify the following :

That is,

(i) 3 7 4 7 (iii) ( 5 3 ) 2 MATHEMATICSIX

(ii) ( 7 3 ) ( 7 3 ) (iv) 8 30 2 5 NUMBER SYSTEMS 7

Solution.

(i) 3 4 ( 7 ) 2 12 7 84 (ii) ( 7 ) 2 ( 3 ) 2 7 3 4 (iii) ( 5 ) 2 ( 3 ) 2 2( 5 )( 3 ) 5 3 2 5 3 8 2 15 (iv)


8 30 4 6 2 5

. Example 12. Find 5.3 geometrically. Solution. Draw AB = 5.3 units and extend it to C such that BC = 1 unit. Find mid-point O of AC. With O as centre, and OA as radius, draw a semicircle. Draw

BD AC, interesting semicircle at D. Then BD


5.3 units. With B as centre and BD as radius, draw an arc, intersecting AC produced at E. Then,
BE BD 5.3 units.

Example 13. Find value of a and b, where a b 2


3 2 3 2 3 2 3 2 3 2 3 2

3 2 3 2

Solution.

We have,

(3 2 ) 2 (3) 2 ( 2 ) 2

9 2 6 2 11 6 2 92 7

11 6 2 ab 2 7 7

11 6 and b 7 7 Example 14. Simplify the following :


2 4 1 1

(i) 3 5 3 5 Solution. (i) 3 5


2 4 5 1 1 4

(ii) 7 3 7 4
2 4 5 4 3 12 6

4 1 (iii) (3 )

(iv) (32) 2/5

35
1

(ii) 7 3

7 12
1 3
4

(iii) 34 x(1) 34
2 5 5 2 5

1 81

(iv) (25 ) 8

22

1 2
2

1 4

NUMBER SYSTEMS

MATHEMATICSIX

PRACTICE EXERCISE
1. Represent each of the following rational numbers on the number line : (i)
3 5

(ii)

7 4
1 2 and . 5 3

(iii) 3.6

(iv) 4.53

2.

(i) Find 4 rational numbers between (ii) Find 5 rational numbers between

3 4 and . 4 3

3.

(i) Represent (ii) Represent

2 and

3 on the some number line

5 on the number line. 4. Without actual division, find which of the following rationals are terminating decimals.
19 6 13 (iii) (ii) 125 35 24 5. Find the decimal expansions of the following :

(i)

(iv)

17 80
37 175

(v)

31 200

(i)

16 9

(ii)

22 7

(iii)

11 30

(iv)

6. Find the decimal representation of

1 1 . Deduce from the decimal representation of , without actual 7 7


2 3 4 5 , , and . 7 7 7 7

calculations, the decimal representation of

7. Express each of the following recurring decimals in the form of a rational number, (i) 0.7 (ii) 0.123 (iii) 0.45 6

p : q

(iv) 3.456

8. (i) Find three irrational numbers between

2 4 and . 3 5

(ii) Insert three irrational numbers between

2 and

3.

9. Give an example of two irrational numbers whose : (i) difference is a rational number. (ii) difference is an irrational number. (iii) sum is a rational number. (iv) sum is an irrational number. (v) quotient is a rational number. (vi) quotient is an irrational number. (vii) Product is a rational number. (viii) Product is an irrational number. 10. Simplify each of the following : (i) (7 3 ) (7 3 ) (iii) 12 20 3 5 (v) (4 2 3) (4 2 3) 11. Represent the following on the number line : (i) (ii) ( 5 3 ) 2 (iv) (2 3 3 2 ) 2 (vii) 3 20 3 5 2 2 4 18

2.4

(ii)

5.7

(iii)

6.8

(iv) 9.2

12. Rationalize the denominator :


1 2

(i)

2 3

(ii)

3 5

(iii)

2 3 2 3

(iv)

5 3

13. Rationalize the denominator:


1 3 1 1

(i)

52 3

(ii)

6 3 2

(iii)

5 33 5

(iv)

2 3 3 2

14. Simplify the following : (i)


4 3 4 3 4 3 4 3 3 2 5 3 2 5

(ii)

3 5 3 5

3 5 3 5

15. If a and b are rational numbers and if

a b 5 , find a and b. 5 1 5 1

16. If x and y are the rational numbers and 17. Evaluate the following : (i) 53 52 (v) (3 8 )1/2 (ii) 58 55 (vi) 3 2 4 2

5 1 5 1

x y 5 , find x and y.

(iii) (32 ) 2

(iv) (64) 2/3

18. Find the valueof x if : (i) 23 x 1 1


7 3 x

1 (ii) 32 x 1 9

3x 2

1 (iii) 6

2 (iv) 3

x 3

2 3

2 3 x

3 2

x 1

19. Find whether the product of irrational numbers (5 2 ), (3 7 ), (3 7 ) and (5 2 ) is a rational or irrational number. 20. (i) Given
2 1.414 , find value of 1 2 1

(ii)

3 1.732, find value of

2 3 2 3

21. (i) Prove that (ii) Prove that

. 2 is not a rational number. . 2 3 is an irrational number.

22. Simplify the following : (i) 0.2 0.3 0.4 (ii) 0.42 0.34 23. Simplify the following : (i)
2 n 1 2 n 2 n 2 n 1
a b c

(iii) 2.13 1.16

(iv) 4.16 1.25

(ii)

x 1 y y 1 z z 1 x

24. Assuming that x is positive real number and a, b, c are rational numbers, show that :
xb (i) c x xc xa xa xb 1 xa (ii) b x
a b

xb xc

b c

xc xa

ca

25. If 2 x 3 y 6 z , show that

1 1 1 0 x y z
1 x2 .

2 26. (i) If x 4 15 , find the value of x

(ii) If x 5 2 6 , find the value of 27. Simplify the following : (i)


1 2 1

1 . x

(ii)

3 2 6 3
2

4 3 6 2

2 3 6 2

2 3 5 3 2 5 28. Rationalize the denominator of following :

(i)

1 1 2 3

(ii)

1 3 5

29. Prove that : (i)


1 1 2 1 2 3 1 3 4 .... 1 8 9 2

(ii)

1 2 3 5

1 2 3 5

2 2

30. Represent the following on the number line : (i) 13 (ii) 17 (iii) 2 3 (iv) 1 2

PRACTICE TEST
M.M : 30 General Instructions : Q. 1-4 carry 2 marks, Q. 5-8 carry 3 marks and Q. 9-10 carry 5 marks each. 1. Find three rational numbers between 2. Represent
3 4 and . 4 3

Time : 1 hour

5 on the number line.

3. Rationalise the denominator :


1 52
16 . 45

4. Find decimal representation of 5. Simplify the following : (i) ( 5 2 ) 2

(ii) (3 2 2 3 )(3 2 2 3 )
4 3 4 3 a 3b , find a and b.

6. If a and b are rational numbers and 7. Evaluate the following :


7 3

11 4 11 4 (i) 2 2

1 (ii) 32

2 / 5

8. Express 0.246 as a rational number in the simplest form. 9. Represent 10. Simplify :

4.2 on the number line. Also, give step of constructions.


6 2 3 3 2 6 3 4 3 6 2

ANSWERS OF PRACTICE EXERCISE


2. (i)
4 5 6 7 , , , 15 15 15 15

(ii)

10 11 12 13 14 , , , , 12 12 12 12 12

4. (ii), (iv) and (v) 6.

5. (i) 1.7

(ii) 3.142857

(iii) 0.36

(iv) 0.21142856

4 5 1 2 3 0.142857 , 0.284714, 0.428571, 0.571428, 0.714285 7 7 7 7 7

7 61 137 1711 (ii) (iii) (iv) 9 495 300 495 8. (i) 0.68010010001 ..., 0.69010110111 .... and 0.7101001000...

7. (i)

(ii) 1.501001000 ... and 1.601001000...

9. (i) 3 5 and 5 3 (iv) 4 5 and 2 5 (vii) 2 3 and 3 10. (i) 46 (ii) 8 2 15


1 (3 5 ) 2

(ii) 3 5 and 5 (v)


20 and 5

(iii) 3 5 and 3 5 (vi)


20 and 6

(viii) 3 2 and 4 3 (iii) 8 (iii) 7 4 3 (iv) 30 12 6 (iv)


1 ( 5 3) 2

(v) 23

(vi) 3 5 10 2

12. (i) 2 3 (ii) 13. (i) 14. (i)

1 1 1 (5 3 3 5 ) (iv) 1 (2 3 3 2 ) ( 5 2 3 ) (ii) ( 6 3 2 ) (iii) 4 30 7 6


38 (ii) 3 5 13

15. a

29 12 ,b 11 11

16. x = 3, y = 0

17. (i) 3125 (ii) 125 (iii) 18. (i)


1 3

1 1 (iv) (v) 81 16

1 2 (vi) 144

(ii)

5 8

(iii)

5 2

(iv)

4 5

19. Rational 23. (i) 2 (ii) 1 28. (i)

20. (i) 0.414

(ii) 13.928

22. (i) 1 (ii)

26. (i) 62 (ii) 12

7 422 1540 (iii) (iv) 90 231 297 27. (i) 0 (ii) 0

1 2 ( 2 2 6 ) (ii) (7 3 3 5 2 15 ) 4 11

ANSWERS OF PRACTICE TEST


1. 6.
10 11 12 , , 12 12 12
a 19 8 ,b 13 13

3.

52

4. 0.35

5. (i) 7 2 10 (ii) 6 10. 0

7. (i)

111 11 (ii) 4 8. 450 2

MATHEMATICSIX

NUMBER SYSTEMS

13

CHAPTER

2
POLYNOMIALS

Points to Remember : 1. A symbol having a fixed numerical value is called a constant. For e.g. 9 ,
7 , , 2 etc. 3

2. A symbol which may take different numerical values is known as a variable. We usually denotes variable by x, y, z etc. 3. A combination of constants and variables which are connected by basic mathematical operations, is known as Algebraic Expression. For e.g. x2 7x + 2, xy2 3 etc. 4. An algebraic expression in which variable have only whole numbers as a power is called a Polynomial. 5. Highest power of the variable is called the degree of the polynomial. For e.g. 7x3 9x2 + 7x 3 is a polynomial in x of degree 3. 6. A polynomial of degree 1 is called a linear polynomial. For e.g. 7x + 3 is a linear polynomial in x. 7. A polynomial of degree 2 is called a Quadratic Polynomial. For e.g. 3y2 7y + 11 is a Quadratic polynomial in y. 8. A polynomial of degree 3 is called a Cubic Polynomial. For e.g. 3t3 7t2 + t 3 is a cubic polynomial in t. 9. According to number of terms, a polynomial having one non-zero term is a monomial, a polynomial having two non-zero terms is a bionomial and a polynomial have three non-zero terms is a trinomial. 10. Remainder Theorem : Let f(x) be a polynomial of degree n 1 and let a be any real number. If f(x) is divided by linear polynomial (x a), then the remainder is f(a). 11. Factor Theorem : Let f(x) be a polynomial of degree n > 1 and a be any real number. (i) If f(a) = 0, then x a is a factor of f(x). (ii) If (x a) is a factor of f(x) then f(a) = 0. 12. Algebraic Identities : (i) (x + y)2 = x2 + 2xy + y2 (ii) (x y)2 = x2 2xy + y2 2 2 (iii) x y = (x y) (x + y) (iv) (x + y + z)2 = x2 + y2 + z2 + 2xy + 2yz + 2xz 3 3 3 (v) (x + y) = x + y + 3xy (x + y) (vi) (x y)3 = x3 y3 3xy (x y) 3 3 2 2 (vii) x y = (x y) (x + xy + y ) (viii) x3 + y3 = (x + y) (x2 xy + y2) 3 3 3 2 (ix) x + y + z 3xyz = (x + y + z) (x + y2 + z2 xy yz xz) 13. If x + y + z = 0 then, x3 + y3 + z3 = 3xyz.

ILLUSTRATIVE EXAMPLES
Example 1. Which of the following expressions are Polynomials? In case of a polynomial, give degree of polynomial. (i) x4 3x3 + 7x2 + 3 (iv) x 14
1 x

(ii) (v) 7

3 y3 7 y 6

(iii)

x 3
2 x

(vi) x 2 / 3 POLYNOMIALS

MATHEMATICSIX

Solution.

(i) is a polynomial of degree 4. (ii) is a polynomial of degree 3. (v) is a polynomial of degree 0. Example 2. Verify whether the following are zeros of the polynomial, indicated against them. (i) p ( x ) 3 x 1, x
1 3 2 (iii) p(x) = x 1, x = 1, 1

(ii) p ( x ) 5 x , x

4 5 (iv) p(x) = (x + 1) (x 2), x = 1, 2

(v) p(x) = x2, x = 0


1 3 2 3

(vi) p(x) = lx + m, x

m l

(vii) p(x) = 3x2 1, x Solution.

(viii) p(x) = 2x + 1, x

1 2

NCERT

(i) We have, p(x) = 3x + 1 At x


1 1 1 , p 3 1 1 1 0 3 3 3

1 is a zero of p(x). 3 (ii) We have, p(x) = 5x

At x

4 4 4 , p 5 4 0 5 5 5

4 is not a zero of p(x). 5

(iii) We have, p( x) x 2 1 At x = 1, p(1) = (1)2 1 = 1 1 = 0 also, at x = 1, p(1) = (1)2 1 = 1 1 = 0 1, 1 both are zeros of p(x). (iv) We have, p(x) = (x + 1) (x 2) At x = 1, p(1) = (1 + 1) (1 2) = 0 (3) = 0 also, at x = 2, p(2) = (2 + 1) (2 2) = 3 0 = 0 1, 2 both are zeros of p(x). (v) We have, p(x) = x2 At x = 0, p(0) = 02 = 0 0 is a zero of p(x). (vi) We have, p(x) = lx + m At, x
m m m , p l m m m 0 l l l

m is a zero of p(x). l (vii) We have, p(x) = 3x2 1

MATHEMATICSIX

POLYNOMIALS

15

At, x

1 1 1 , p 3 3 3 1 3 3 1 1 1 0 3 2 2 2 4 , p 3 3 3 1 3 3 1 4 1 3 0 3
2 3
2

and at, x
1

is a zero of p(x), but

is not a zero of p(x).

3 (viii) We have p(x) = 2x + 1

At x

1 1 1 , p 2 1 1 1 2 0 2 2 2

1 is not a zero of p(x). 2 Example 3. Find the zero of the polynomial in each of the following cases : (i) p(x) = x + 5 (ii) p(x) = x 5 (iii) p(x) = 2x + 5 (iv) p(x) = 3x 2 (v) p(x) = 3x (vi) p(x) = ax ; a 0 (vii) p(x) = cx + d, c 0, c, d are real numbers. Solution. We know, finding a zero of p(x), is the same as solving the equation p(x) = 0. (i) p(x) = 0 x + 5 = 0 x = 5 5 is a zero of p(x). (ii) p(x) = 0 x 5 = 0 x = 5 5 is a zero of p(x).

NCERT

(iii) p(x) = 0 2x + 5 = 0 2x = 5 x
5 is a zero of p(x). 2
2 3

5 2

(iv) p(x) = 0 3x 2 = 0 3x = 2 x
2 is a zero of p(x). 3

(v) p(x) = 0 3x = 0 x = 0 0 is a zero of p(x). (vi) p(x) = 0 ax = 0 x = 0 0 is a zero of p(x).

( Given a 0)
d c

(vii) p(x) = 0 cx + d = 0 cx = d x
d is a zero of p(x). c

16

POLYNOMIALS

MATHEMATICSIX

Example 4. Using remainder theorem, find the remainder when p(x) = 2x3 5x2 + 9x 8 is divided by (x 3). Solution. Remainder = p(3) = 2(3)3 5(3)2 + 9 (3) 8 = 54 45 + 27 8 = 81 53 = 28 Ans. Example 5. Find the remainder when x 3 3x 2 3x 1 is divided by : (i) x + 1 Solution. (ii) x
1 2

(iii) x

(iv) x +

(v) 5 + 2x

NCERT

(i) x + 1 = 0 x = 1 by remainder theorem, the required remainder is p (1). Now, p(x) = x3 + 3x2 + 3x + 1 p(1) = (1)3 + 3(1)2 + 3(1) + 1 = 1 + 3 3 + 1 = 0 Remainder = 0 (ii) x
1 1 0x . 2 2

1 by remainder theorem, the required remainder is p . 2

Now,

1 1 1 1 p 3 3 1 2 2 2 2
1 3 3 1 6 12 8 27 1 8 4 2 8 8

Remainder 27 8 (iii) x = 0 by remainder theorem, the required remainder is p(0). Now, p(0) = (0)3 + 3(0)2 + 3(0) + 1 = 1 Remainder = 1 (iv) x + = 0 x = by remainder theorem, the required remainder is p(). Now, p() = ()3 + 3()2 + 3 () + 1 = 3 + 32 3 + 1 Remainder = 3 + 32 3 + 1. (v) 5 + 2x = 0 2x = 5 x =
5 2

5 . By remainder theorem, the required remainder is p 2

5 5 5 5 Now, p 3 3 1 2 2 2 2
125 75 15 1 8 4 2

MATHEMATICSIX

POLYNOMIALS

17

125 150 60 8 27 8 8

Remainder

27 . 8

Example 6. Using Factor theorem, show that (x + 2) is a factor of x4 x2 12. Solution. Let p(x) = x4 x2 12. Now, x + 2 = 0 x = 2. (x + 2) is a factor of p(x) p(2) = 0 Now, p(2) = (2)4 (2)2 12 = 16 4 12 = 0, which shows that (x + 2) is a factor of p(x). Example 7. Use the factor theorem to determine whether g(x) is a factor of p(x) is each of the following cases: (i) p(x) = 2x3 + x2 2x 1, g(x) = x + 1 (ii) p(x) = x3 + 3x2 + 3x + 1, g(x) = x + 2 (iii) p(x) = x3 4x2 + x + 6, g(x) = x 3 NCERT Solution. (i) In order to prove that g(x) = x + 1 is a factor of p(x) = 2x3 + x2 2x 1, it is sufficient to show that p(1) = 0. Now, p(1) = 2 (1)3 + (1)2 2(1) 1 =2+1+21=0 g(x) is a factor of p(x). (ii) In order to prove that g(x) = x + 2 is a factor of p(x) = x3 + 3x2 + 3x + 1, it is sufficient to show that p(2) = 0. Now, p (2) = (2)3 + 3(2)2 + 3(2) + 1 = 8 + 12 6 + 1 = 1 0. g(x) is not a factor of p(x). (iii) In order to prove that g(x) = x 3 is a factor of p(x) = x3 4x2 + x 6, it is sufficient to show that p(3) = 0 Now, p(3) = (3)3 4(3)2 + 3 6 = 27 36 + 3 6 = 12 0. g(x) is not a factor of p(x). Example 8. If x3 + ax2 + bx + 6 has x 2 as a factor and leaves a remainder 3, when divided by x 3, find the value of a and b. Solution. Let p(x) = x3 + ax2 + bx + 6 Since, x2 is a factor of p(x) p(2) = 0 ( factor theorem) (2)3 + a(2)2 + b(2) + 6 = 0 2a + b = 7 ...(1) Also, p(x) leaves remainder 3, when divided by x 3. p(3) = 3 ( Remainder theorem) 3 2 (3) + a(3) + b(3) + 6 = 3 3a + b = 10 ...(2) Solving (1) and (2), we get a = 3, b = 1 Ans. 18 POLYNOMIALS

MATHEMATICSIX

Example 9. Factorize the following: (i) 6x2 18xy (ii) x3 + 7x2 x 7 (iii) 16a2 81b2 2 2 (iv) x + 5x 24 (v) 9x 22x + 8 Solution. (i) 6x (x 3y) (ii) x2 (x + 7) 1 (x + 7) = (x2 1) (x + 7) = (x 1) (x + 1) (x + 7) (iii) (4a)2 (9b)2 = (4a + 9b) (4a 9b) (iv) x 2 8 x 3x 24 x( x 8) 3( x 8) = (x 3) (x + 8) (v) 9x2 18x 4x + 8 = 9x (x 2) 4(x 2) = (x 2) ( 9x 4) Example 10. Factorise : (i) 12x2 7x + 1 (ii) 2x2 + 7x + 3 (iii) 6x2 + 5x 6 (iv) 3x2 x 4 NCERT 2 Solution. (i) 12x 7x + 1 here, p + q = coefficient of x = 7 pq = coefficient of x2 constant term = 12 1 =12 p + q = 7 = 4 3, pq = 12 = (4) (3) 12x2 7x + 1 = 12x2 4x 3x + 1 = 4x (3x 1) 1(3x 1) = (4x 1) (3x 1) Ans. (ii) 2x2 + 7x + 3 = 2x2 + x + 6x + 3 = x (2x + 1) + 3 (2x + 1) = (x + 3) (2x + 1) (iii) 6x2 + 5x 6 = 6x2 + 9x 4x 6 = 3x (2x + 3) 2(2x + 3) = (3x 2) (2x + 3) (iv) 3x2 x 4 = 3x2 + 3x 4x 4 = 3x(x + 1) 4(x + 1) = (3x 4) (x + 1) Example 11. Factorize x3 3x2 9x 5 using factor theorem. Solution. Let p(x) = x3 3x2 9x 5. factors of constant term are 1 and 5. Now, p(1) = 13 3(1)2 9(1) 5 0 p(1) = (1)3 3(1)2 9(1) 5 0. (x + 1) is a factor of p(x). p(x) = (x + 1) (x2 4x 5) = (x + 1) [x2 5x + x 5] = (x + 1) [x(x 5) + 1 (x 5)] = (x + 1) (x + 1) (x 5) Ans. Example 12. Factorise : x3 + 13x2 + 32x + 20 NCERT 3 2 Solution. Let p(x) = x + 13x + 32x + 20 Now, factors of constant term 20 are 1, 2, 5, 10 and 20. Now, p(1) = (1)3 + 13(1)2 + 32(1) + 20 = 1 + 13 + 32 + 20 = 66 0 p(1) = (1)3 + 13(1)2 + 32(1) + 20 = 1 + 13 32 + 20 = 33 + 33 = 0 (x + 1) is a factor of x3 + 13x2 + 3x + 20. Now, divide p(x) by x + 1.

MATHEMATICSIX

POLYNOMIALS

19

p(x) = (x +1) (x2 + 12x + 20) = (x + 1) [x2 + 2x + 10x + 20] = (x + 1) [x(x + 2) + 10 (x + 2)] = (x + 1) (x + 2) (x + 10) Ans. Example 13. Expand the following : (i) (a 2b + 3c)2 (ii) (2x + y)3 (iii) (3x 2y)3 Solution. (i) (a)2 + (2b)2 + (3c)2 + 2(a) (2b) + 2(2b) (3c) + 2 (a) (3c) = a2 + 4b2 + 9c2 4ab 12bc + 6ac (ii) (2x)3 + (y)3 + 3(2x)(y) [2x + y] = 8x3 + y3 + 6xy (2x + y) = 8x3 + y3 + 12x2y + 6xy2 (iii) (3x)3 (2y)3 3(3x) (2y) [3x 2y] = 27x3 8y3 18xy (3x 2y) = 27x3 8y3 54x2y + 36xy2 Example 14. Factorize the following : (i) x3 + 27y3 (ii) 27a3 64b3 (iii) a3 8b3 + 64c3 + 24abc [ a3 + b3 = (a + b) (a2 ab + b2)] Solution. (i) (x)3 + (3y)3 = (x + 3y) (x2 3xy + 9y2) 3 3 2 2 (ii) (3a) (4b) = (3a 4b) (9a + 12ab + 16b ) [ a3 b3 = (a b) (a2 + ab + b2)] (iii) (a)3 + (2b)3 + (4c)3 3 a (2b) (4c) = (a 2b + 4c) (a2 + 4b2 + 16c2 + 2ab + 8bc 4ac) [ a3 + b3 + c3 3abc = (a + b + c) (a2 + b2 + c2 ab bc ca)] Example 15. Verify that : x3 + y3 + z3 3xyz Solution. Consider,
1 ( x y z ).[( x y ) 2 ( y z ) 2 ( z x) 2 ] 2 LHS = x3 + y3 + z3 3xyz = (x + y + z) (x2 + y2 + z2 xy yz zx) 1 (x + y + z) . 2 (x2 + y2 + z2 xy yz zx) 2 1 (x + y + z) (2x2 + 2y2 + 2z2 2xy 2yz 2zx) 2 1 (x + y + z) (x2 + x2 + y2 + y2 + z2 + z2 2xy 2yz 2zx) 2 1 (x + y + z) [(x2 + y2 2xy) + (y2 + z2 2yz) + (z2 +x2 2zx)] 2

NCERT

1 (x + y + z) [(x y)2 + (y z)2 + (z x)2] 2 = RHS. Hence verify. Example 16. If x + y + z = 0, show that x3 + y3 + z3 = 3xyz. Solution. We have x + y + z = 0 x+y=z Cubing both sides, we get (x + y)3 = (z)3 x3 + y3 + 3xy (x + y) = z3 x3 + y3 + 3xy (z) = z3 ( x + y = z)

NCERT

20

POLYNOMIALS

MATHEMATICSIX

x3 + y3 + z3 3xyz = 0 x3 + y3 + z3 = 3xyz. Hence shown. Example 17. Give possible expressions for the length and breadth of each of the following rectangles, in which their areas are given : (i) Area : 25a2 35a + 12 (ii) 35y2 + 13y 12 NCERT Solution. (i) Given, Area of rectangle = 25a2 35a + 12 = 25a2 15a 20a + 12 = 5a (5a 3) 4 (5a 3) = (5a 3) (5a 4) Possible length and breadth are (5a 3) and (5a 4) units. (ii) Area of given rectangle = 35y2 + 13y 12 = 35y2 + 28y 15y 12 = 7y (5y + 4) 3 (5y + 4) = (5y + 4) (7y 3). Possible length and breadth are (5y + 4) and (7y 3) units. Example 18. What are the possible expressions for the dimensions of the cuboid whose volumes are given below: (i) Volume : 3x2 12x (ii) Volume : 12ky2 + 8 ky 20 k NCERT 2 Solution. (i) Volume : 3x 12x = 3x (x 4) = 3 x (x 4) Possible dimensions of cuboid are 3, x and (x 4) units. (ii) Volume = 12ky2 + 8ky 20k = 4 k (3y2 + 2y 5) = 4k (3y2 3y + 5y 5) = 4k [3y (y 1) + 5 (y 1)] = 4k (y 1) (3y + 5) Possible dimensions of a cuboid are 4k, (y 1) and (3y + 5) units.

PRACTICE EXERCISE
1. Which of the following expressions are polynomials in one variable? (i) 3x2 x3 + 7x + 1
4 x (v) 2x4 7x3 + 3

(ii) x

(iii)

2 y2 7 y 2

(iv) y 3 / 2 y 2 1 2. Write degree of each the following polynomial: (i) 7x4 9x3 + 2x + 4

(ii) 7 y2 + y3

(iii)

5t 3 7t 2 1

(iv) 10 (v) 3x2 7x + 4 3. Classify each of the following as linear, quadratic and cubic polynomial: (i) 3x3 7x (ii) 4 y2 + 3y 1
2 3

(iii) 7r

(v) y y (iv) x 3x 3 2 4. Find the value of p(x) = 5x x + 3x + 4 at (i) x = 0 (ii) x = 2 (iii) x = 1 5. Find the value of p(0), p(2) and p(3) where p(x) = x3 x2 + x 1. 6. Verify whether the following are zeros of the polynomial, indicated against them:
3 2 (iii) p(x) = x2 + x 6; x = 3, 2

(i) p( x ) 2 x 3; x

(ii) p(x) = (x + 3) (x 4) ; x = 3, 4 (iv) p(x) = x x3 ; x = 0, 1, 1 POLYNOMIALS 21

MATHEMATICSIX

7. Find the zero of the polynomial in each of the following : (i) p(x) = x 4 (ii) p(x) = 3x + 4 (iii) p(x) = 7x (iv) p(x) = rx + s; r 0, r, s are real numbers. 8. Using Remainder Theorem, find the remainder when : (i) 4x3 7x2 + 3x 2 is divided by x 1 (ii) x3 7x2 + 6x + 4 is divided by x 3 (iii) x3 + 2x2 x + 3 is divided by x + 3 (iv) 4x3 4x2 + x 2 is divided by 2x + 1 (v) x3 ax2 + 5x + a is divided by x a (vi) x3 + ax2 6x + 2a is divided by x + a 9. If 35 is the remainder when 2x2 + ax + 7 is divided by x 4, find the value of a. 10. Without actual division, prove that 2x3 + 13x2 + x 70 is exactly divisible by x 2. 11. Show that x 1 is a factor of x3 2x2 5x + 6. 12. Find the value of p for which the polynomial x4 2x3 + px2 + 2x + 8 is exactly divisible by x + 2. 13. Find values of a and b so that the polynomial x4 + 7x3 + 4x2 + ax + b is exactly divisible by x 1 and x + 3. 14. The polynomials ax3 3x2 + 7 and 2x3 + 7x 2a are divided by x + 3. If the remainder in each case is same, find the value of a. 15. The polynomials ax3 + 4x2 3 and 4x3 + 4x a when divided by x 3, leaves the remainder R1 and R2 respectively. Find the value of a if R1 = 3R2. 16. Find the integral zeros of x3 3x2 x + 3. 17. Find the integral zeros of x3 + 4x2 x 4. 18. Show that x 2 is a factor of p(x) = x3 6x2 + 15x 14. 19. Show that x + 4 is a factor of x4 + 3x3 4x2 + x + 4. 20. Find value of a for which x + a is a factor of f(x) = x3 + ax2 + 3x + a + 4. 21. Without actual division, prove that 2x4 + x3 2x2 + 5x 6 is exactly divisible by x2 + x 2. 22. For what value of a is the polynomial 2x3 ax2 + 8x + a + 4 is exactly divisible by 2x + 1. 23. If x3 + ax2 + bx 12 has x 3 as a factor and leaves a remainder 10 when divided by x + 2, find a and b. 24. Factorise the following, using factor theorem : (i) x3 6x2 + 11x 6 (ii) x3 6x2 + 3x + 10 (iii) 2x3 5x2 x + 6 (iv) x3 3x2 + 3x 1 (v) x3 + 4x2 11x 30 (vi) 4x3 + 9x2 19x 30 3 2 25. Factorize 6x + 35x 7x 6, given x + 6 is one of its factor. 26. Factorize 2x3 + 5x2 124x 63, given x + 9 is one of its factor. 27. Use suitable identity to find the following products : (i) (x + 4) (x + 6) (ii) (x 3) (x + 8) (iii) (2x + 3) (3x 2)
2 5 2 5 (iv) t t 2 2

28. Evaluate the following products without multiplying directly: (i) 87 93 (ii) 106 94 22 POLYNOMIALS MATHEMATICSIX

29. Factorize the following using appropriate identities :

y2 (ii) 9 x 2 6 x 1 144 (iv) 9 x 2 30 xy 25 y 2 (iii) x 4 y 4 30. Factorize by splitting the middle term : (i) x2 21x + 108 (ii) 12y2 y 6 (iii) t2 11t 42 (v) 8x2 2x 15 (vi) 20x2 + 13x 84 31. Expand each of the following using suitable identity :
(i) 9 x 2 (i) ( x 2 y 3z ) 2 (iii) (3x 2y)
3

(iv) 6z2 + 5z 6

a (ii) b 3c 3 y (iv) 2 x 2
3

32. Evaluate the following, using suitable identity: (i) (101)3 (ii) (99)3 33. Factorize the following: (i) 4x2 + y2 + 9z2 + 4xy 6yz 12xz (ii) 64a3 144a2b + 108ab2 27b3 3 2 2 3 (iii) x + 6x y + 12xy + 8y (iv) 8x3 + 125 (v) 27a3 64b3 (vi) 27a3 + 8b3 + c3 18abc 34. Without actually calculating the cubes, find the value of each of the following : (i) (14)3 + (8)3 + (6)3 (ii) (19)3 + (11)3 + (8)3 35. If x + y + z = 0, prove that x3 + y3 + z3 = 3xyz. 36. Prove that: x3 + y3 + z3 3xyz
1 ( x y z ) [( x y ) 2 ( y z ) 2 ( z x) 2 ] 2 37. Using factor theorem, show that a b, b c and c a are the factors of a(b2 c2) + b (c2 a2) + c (a2 b2). 38. Factorize the following : (i) 1 2ab a2 b2 (ii) x4 + 5x2 + 9

(iv) 9( x y ) 2 24( x 2 y 2 ) 16( x y ) 2 (iii) 5 5 x 2 30 x 8 5 (v) x6 y6 (vi) x6 + y6 39. Factorize the following : (i) x3 (y z)3 + y3 (z x)3 + z2(x y)3 (ii) (a 4b)3 + (4b 3c)3 + (3c a)3 3 3 3 40. Prove that : (a + b) + (b + c) + (c + a) 3 (a + b) (b + c) (c + a) = 2 (a3 + b3 + c3 3abc)

PRACTICE TEST
MM : 30 General Instructions : Q. 1-4 carry 2 marks, Q. 5-8 carry 3 marks and Q. 9-10 carry 5 marks each. 1. 2. 3. 4. Find the remainder when 3x3 8x2 + 9x 10 is divided by x 3. Find the value of a for which x3 + ax2 3x + 14 is exactly divisible by x + 2. Factorize : 64a3 27b3 144a2b + 108ab2 Evaluate : (103)3 , using suitable identity. POLYNOMIALS 23 Time : 1 hour

MATHEMATICSIX

5. Factorize : 3x2 + 13x 10 6. Expand the following : (i) (x 2y 3z)


2

x (ii) y 3

7. Without actually calculating the cubes, evaluate the following : (20)3 + (15)3 + (5)3 8. Factorize : x3 64y3 8z3 24xyz. 9. Factorize 6x3 + 25x2 + 21x 10 using factor theorem. 10. Prove that : a3 + b3 + c3 3abc =
1 (a b c) (a b) 2 (b c ) 2 (c a ) 2 2

ANSWERS OF PRACTICE EXERCISE


1. 3. 4. 6. (i), (iii) and (v) 2. (i) 4 (ii) 3 (iii) 3 (iv) 0 (v) 2 (i) Binomial (ii) Trinomial (iii) Monomial (iv) Bionomial (v) Bionomial 4, 46, 5 5. 1, 5, 40 (i) yes (ii) yes (iii) no (iv) yes (ii) (iii) 0
5 r (iii) 3 (iv) 4 (v) 6a

7. (i) 4

4 3 8. (i) 2 (ii) 175

(iv)

(vi) 8a 14. a 22. a


11 5

9. a = 1 16. 1, 1, 3

12. p = 9 17. 1, 1, 4

13. a = 21, b = 9 20. a = 2 (ii) (x + 1) (x 2) (x 5) (v) (x + 2) (x 3) (x + 5) 26. (x + 9) (x 7) (2x 1)

15. a

109 10

24. (i) (x 1) (x 2) (x 3) (iv) (x 1) (x 1) (x 1) 25. (x + 6) (3x + 1) (2x 1) 27. (i) x2 + 10 x + 24 28. (i) 8091

1 23. a = 1, b = 8 3 (iii) (x + 1) (x 2) (2x 3) (vi) (x 2) (x + 3) (4x + 5)

(ii) x2 + 5x 24 (ii) 9964

(iii) 6x2 + 5x 6
y y 29. (i) 3 x 3 x 12 12

4 (iv) t

25 4

(ii) (3x 1) (3x 1)

(iii) ( x y)( x y)( x 2 y 2 ) (iv) (3x + 5y) (3x + 5y) 30. (i) (x 9) (x 12) (ii) (3y 2) (4y + 3) (iii) (t + 3) (t 14) (iv) (2z + 3) (3z 2) (iv) (2x 3) (4x + 5) (vi) (4x 7) (5x + 12) 31. (i) x2 + 4y2 + 9z2 4xy 12yz + 6xz (iii) 27x3 8y3 54x2y + 36xy2 32. (i) 1030301 (ii) 970299 24 POLYNOMIALS MATHEMATICSIX (ii)
a2 2 b 2 9c 2 ab 6bc 2ac 9 3 y3 3 6 x 2 y xy 2 8 2

(iv) 8 x 3

33.

(i) (2x + y 3z) (2x + y 3z) (ii) (4a 3b) (4a 3b) (4a 3b) (iv) (2x + 5) (4x2 10x + 25) (v) (3a 4b) (9a2 + 12ab + 16b2) (vi) (3a + 2b + c) (9a2 + 4b2 + c2 6ab 2bc 3ac) 34. (i) 2016 (ii) 5016 38. (i) (1 + a + b) (1 a b) (ii) (x2 + x + 3) (x2 x + 3)

(iii) (x + 2y) (x + 2y) (x + 2y)

(iii) ( 5 x 2) (5 x 4 5 )

(iv) (x + 7y) (x + 7y) (v) (x y) (x + y) (x2 xy + y2) (x2 + xy + y2) 2 2 4 2 2 4 (vi) (x + y ) (x x y + y ) 39. (i) 3xyz (y z) (z x) (x y) (ii) 3 (a 4b) (4b 3c) (3c a)

ANSWERS OF PRACTICE TEST


1. 26 4. 1092727 2. a = 3 5. (3x 2) (x + 5) 3. (4a 3b) (4a 3b) (4a 3b)
x3 x2 y y3 xy 2 27 3

6. (i) x2 + 4y2 + 9z2 4xy + 12yz 6xz 7. 4500 9. (x + 2) (2x + 5) (3x 1)

(ii)

8. (x 4y 2z) (x2 + 16y2 + 4z2 + 4xy 8yz + 2xz)

MATHEMATICSIX

POLYNOMIALS

25

CHAPTER

3
CO-ORDINATE GEOMETRY

Points to Remember : 1. Coordinate axes : Two mutually perpendicular lines XOX and YOY known as x-axis and y-axis respectively, constitutes to form a co-ordinate axes system. These axes interests at point O, known as origin.

2. Co-ordinate axes divides the plane into four regions, known as Quadrants. 3. The position of any point in a plane is determined with reference to x-axis and y-axis. 4. The x-coordinate of a point is its perpendicular distance from the y-axis measured along the x-axis. The x-coordinate is known as abscissa. 5. The y-coordinate of a point is its perpendicular distnace from the x-axis measured along the y-axis. The y-coordinate is known as ordinate. 6. Abscissa and ordinate of a point written in the form of ordered pair, (ascissa, ordinate) is known as the co-ordinate of a point. 7. If the point in the plane is given, we can find the ordered pair of its co-ordinate and if the ordered pair of real numbers is given, we can find the point in the plane corresponding to this ordered pair.
Quadrant I Sign of 8. Sing Convention : x coordinate y coordinate

II

III

IV

26

CO-ORDINATE GEOMETRY

MATHEMATICSIX

ILLUSTRATIVE EXAMPLES
Example 1. Write the answer of each of the following questions: (i) What is the name of the horizontal and the vertical lines drawn to determine the position of any point in the cartesian plane? (ii) What is the name of each part of the plane formed by these two lines? (iii) Write the name of the point where these two lines intersect? NCERT Solution. (i) Rectangular axes/co-ordinate axes (ii) Quadrant (iii) Origin Example 2. Write the co-ordinate of points A, B, C, D, E and F.

MATHEMATICSIX

CO-ORDINATE GEOMETRY

27

A (4, 3)

Example 5. Look at the figure given, and write the following : (i) The co-ordinate of A. (ii) The abscissa of point B. (iii) The ordinate of point C. (iv) The co-ordinate of D. (v) The point whose co-ordinate are (2, 5).

Solution.

(i) Co-ordinate of A are (6, 4) (iii) Ordinate of C is 3 (v) Point E have co-ordinate as (2, 5)

(ii) Abscissa of B is 2. (iv) Co-ordinates of D are (4, 3) MATHEMATICSIX

28

CO-ORDINATE GEOMETRY

Example 6. Plot the points (x, y) given in the following table on the plane choosing suitable units of distance on the axes. x 2 1 0 1 3 y 8 7 1.25 3 1 NCERT Solution.

Example 7. Plot the points A (3, 3), B (5,3), C (5, 2) and D (3, 2) on the graph paper. Join them in order and name the figure so formed. Also, find its area. Solution.

MATHEMATICSIX

CO-ORDINATE GEOMETRY

29

ABCD is a rectangle Area of ABCD = AB BC = 8 5 sq. units = 40 sq. units Example 8. Graph the following equations (i) x = 2 (ii) y = 3 (iii) y = x + 2 Solution. (i) x = 2. The given equation can be written as 1.x + 0.y = 2 x is fixed as 2 and y may choose any value. Let us represent following information in a tabular form.
x 2 2 2 2 2 y 2 1 0 1 2

(ii) y = 3. Given equation may be written as 0.x + 1.y = 3 y is fixed as 3 and x may choose any value. Let us represent this information in a tabular form.
x y 2 3 1 3 0 3 1 3 2 3

(iii) y = x + 2 here, when x = 0, y = 2 ; x = 1, y = 3 ; x = 1, y = 1 etc. Represent this in the tabular form.


x 1 0 1 2 y 1 2 3 4

30

CO-ORDINATE GEOMETRY

MATHEMATICSIX

PRACTICE EXERCISE
1. State the quadrant in which the following points lie. (i) A (3, 4) (ii) B (5, 11) (iii) C (10, 15) (iv) D (8, 12) (v) E (11, 5) (vi) F (100, 200) (vii) G (10, 50) (viii) H (20, 5) 2. Look at the figure given, and write the following : (i) The co-ordinate of P (ii) The ordinate of Q (iii) The abscissa of R (iv) The point given by (4, 3) (v) The point which is at a distance of 3 units from y-axis (vi) Co-ordinate of point T

MATHEMATICSIX

CO-ORDINATE GEOMETRY

31

3. Plot the points P (1, 3), Q (3, 7) and R (5, 11). Are these points collinear? 4. Plot the points A (2, 3), B (8, 3) and C (6, 7). Join them in order. Name the figure obtained. Also find its area. 5. Plot the points A (3, 2), B (11, 8), C (8, 12) and D (0, 6). Join them in order. Name the figure thus obtained. 6. Plot the points P (0, 1), Q (2, 1), R (0, 3), S (2, 1). Join them in order. Name the figure obtained. 7. Plot the points A (2, 1), B (1, 1), C (4, 3) and D (3, 3). Join them in order. Name the figure thus obtained. 8. Plot the points P (7, 3), Q (3, 0), R (0, 4) and S (4, 1). Join them in order. Name the figure thus obtained. 9. Plot the points A (0, 3), B (4, 1), C (0, 6) and D (4, 1). Join them in order. Name the figure thus obtained. 10. Graph the folloiwng equations : (i) x = 4 (ii) y = 3 (iii) y = x (iv) x + y = 3

PRACTICE TEST
M.M : 15 General Instructions : All questions carry 3 marks each. 1. Name the quadrant in which the following points lie : (i) A (7, 9) (ii) B (10, 25) (iii) C (7, 9) (iv) D (11, 7) 2. Plot A (6, 3), B (6, 0) and C (4, 5). Join these points in order. Name the figure thus obtained. 3. Look at the figure and write the following : (i) The co-ordinate of A. (ii) The co-ordinate of B. (iii) The abscissa of C. (iv) The point whose co-ordinates are (4, 3)
Y 6 5 4 D 3 2 1 7 6 5 4 3 2 1 1 2 C 3 4 5 B 0 1 2 3 4 5 6 7 8 X A

Time : 1/2 hour

32

CO-ORDINATE GEOMETRY

MATHEMATICSIX

4. Mark the points P (4, 2), Q (4, 4), R (3, 4) and S (3, 2) on the graph paper. Join these points in order. Name the figure obtained. Also, find area of the figure obtained. 5. Draw the graph of y = x + 1. Does the point (7, 6) lie on this line?

ANSWERS OF PRACTICE EXERCISE


1. (i) IVth quadrant (v) IInd qudrant 2. (i) P (5, 4) (v) S 3. yes (ii) IInd qudrant (vi) IIIrd quadrant (ii) Q (4, 2) (vi) T (4, 3) (iii) IIIrd quadrant (vii) Ist quadrant (iii) 6 (iv) Ist quadrant (viii) IVth qudrant (iv) T

4. Triangle, Area = 20 sq. units

MATHEMATICSIX

CO-ORDINATE GEOMETRY

33

5. Rectangle

6. Square

34

CO-ORDINATE GEOMETRY

MATHEMATICSIX

9. Kite

10.

(i)

(ii)

MATHEMATICSIX

CO-ORDINATE GEOMETRY

35

(iii)

(iv)

ANSWERS OF PRACTICE TEST


1. (i) IInd quadrant 2. Triangle (ii) IIIrd qudrant (iii) IVth quadrant (iv) Ist quadrant

3. (i) A (7, 6)

(ii) B (2, 5)

(iii) 4

(iv) D

4. Rectangle, Area = 42 square units 36 CO-ORDINATE GEOMETRY MATHEMATICSIX

5. No

MATHEMATICSIX

CO-ORDINATE GEOMETRY

37

CHAPTER

4
LINEAR EQUATIONS IN TWO VARIABLES

Points to Remember : 1. An equation of the form ax + by + c = 0, where a, b, c are real numbers, such that a and be are not both zero, is known as linear equation in two variables. 2. A linear equation in two variables has infinitely many solutions. 3. The graph of linear equation in two variables is always a straight line. 4. y = 0 is the equation of x-axis and x = 0 is the equation of y-axis. 5. The graph of x = a is a straight line parallel to the y-axis. 6. The graph of y = b is a straight line parallel to the x-axis. 7. The graph of y = kx passes through the origin. 8. Every point on the graph of a linear equation in two variables is a solution of the linear equation. Also, every solution of the linear equation is a point on the graph of the linear equation.

ILLUSTRATIVE EXAMPLES
Example 1. Express the following linear equations in the form ax + by + c = 0 and indicate the values of a, b and c in each case. (i) 4x 7y = 10 Solution. (ii) x +

y =5 2

(iii) x = 4y

(iv) 4y 9 = 0 (v) x = 5 (i) 4x 7y = 10 4x 7y 10 = 0 comparing with ax + by + c = 0, a = 4, b = 7, c = 10 (ii) x


y y 5 x 5 0 = ax + by + c 2 2

1 ,c=5 2 (iii) x = 4y x + 4y + 0 = 0 = ax + by + c on comparing, a = 1, b = 4, c = 0 (iv) 4y 9 = 0 0.x + 4.y 9 = 0 = ax + by + c on comparing, a = 0, b = 4, c = 9 (v) x = 5 x 5 = 0, x + 0.y 5 = 0 on comparing, a = 1, b = 0, c = 5 Example 2. Which one of the following options is true, and why? y = 3x + 5 has (i) a unique solution, (ii) only two solutions, (iii) infinitely many solutions. NCERT Solution. Given equation is y = 3x + 5 when x = 0, y = 3(0) + 5 = 5 (0, 5) is one of its solution.
on comparing, a = 1, b when y = 0, then x 38
5 3

LINEAR EQUATIONS IN TWO VARIABLES

MATHEMATICSIX

5 , 0 is another solution. 3 when x = 1, then y = 3(1) + 5 = 8 (1, 8) is also its solution. So, its clear that for every infinitely values we can give to x, we have corresponding value of y. This equation has infinitely many solutions. Example 3. Bhavya and Anisha have a total of Rs. 100. Express this information in the form of an equation. Solution. Let total amount with Bhavya = Rs x and total amount with Anisha = Rs. y according to the question, x + y = 100, which is required equation. Example 4. Give five integer solutions of the equation 3x + y = 8. Solution. 3x + y = 8 y = 8 3x Now, when x = 0, y = 8 3 (0) = 8 when x = 1, y = 8 3 (1) = 5 when x = 2, y = 8 3 (2) = 2 when x = 1, y = 8 3 (1) = 11 when x = 2, y = 8 3 (2) = 14 Solutions can be represented in the tabular form as follows :

x 0 1 2 1 2 y 8 5 2 11 14

Example 5. Write four solutions for each of the following equations : (i) 2x + y = 7 (ii) x + y = 9 (iii) x = 4y NCERT Solution. (i) 2x + y = 7 y = 7 2x For x = 0, y = 7 2(0) = 7 For x = 1, y = 7 2(1) = 7 2 = 5 For x = 2, y = 7 2(2) = 7 4 = 3 Four solutions of the given equation are (0, 7), (1, 5), (2, 3), (1, 9). (ii) Given equation x + y = 9 y = 9 x For x = 0, y = 9 (0) = 9 For x = 1, y = 9 (1) = 9 For x = 2, y = 9 (2) = 9 2 For x = 1, y = 9 (1) = 9 + The four solutions of the given equation are (0, 9), (1, 9 ), (2, 9 2), (1, 9 + ). x (iii) Given equation x = 4y y 4 0 For, x = 0, y 0 4 4 For x = 4, y 1 4 4 1 For x = 4, y 4 8 For x = 8, y 2 4 The four solutions of a given equation are (0, 0), (4, 1) (4, 1) and (8, 2). MATHEMATICSIX LINEAR EQUATIONS IN TWO VARIABLES 39

Example 6. Find a if x = 3, y = 1 is a solution of the equation 3x y = a. Solution. Given 3x y = a As x = 3, y = 1 is a solution of this given equation, it must satisfy it. 3 (3) 1 = a a = 9 1 a 8 Ans. Example 7. Give the equations of two lines passing through (2, 14). How many more such lines are there, and why? NCERT Solution. Equations of two lines passing through (2, 14) can be taken as 2x + y = 18 or 3x y = 8. Aslo, equations such as 7x y = 0, 5x + 2y = 38 etc. are also satisfied by the co-ordinates of the point (2, 14). So, any line passing through (2, 14) is an example of a linear equation for which (2, 14) is a solution. Thus, there are infinite number of lines through (2, 14). Example 8. Draw the graph of the following equations : (i) x = 3 (v) 2x + 5y = 10 Solution. (ii) y = 4 (vi) 2x y = 7 (ii) y = 4 As y is constant, x may take any value. (iii) y = x (iv) y = x

(i) x = 3 As x is constant, y may take any value.

x 3 3 3 3 3 y 2 1 0 1 2

x 2 1 0 1 2 y 4 4 4 4 4

(iii) y = x
x y 2 1 0 1 2 2 1 0 1 2

(iv) y = x
x 2 1 0 1 2 y 2 1 0 1 2

40

LINEAR EQUATIONS IN TWO VARIABLES

MATHEMATICSIX

(v) 2x + 5y = 10 10 2 x y 5
x 0 5 3 5 y 2 0 0.8 4

(vi) 2x y = 7 y = 2x 7
x 0 1 4 5 y 7 5 1 3

MATHEMATICSIX

LINEAR EQUATIONS IN TWO VARIABLES

41

Example 9. From the choices given, choose the equation whose graph is given (i) y = 2x (ii) y = 2x + 1 (iii) x + y = 0

(iv) y = 2x 4

Solution.

Given points on the graph are (2, 0), (1, 2) (0, 4) and (1, 6). By trial and error we observe that all these 4 given points satisfy the equation y = 2x 4, so, it is a graph of y = 2x 4.

Example 10. The taxi fare in a city is as follows. For the first km., the fare is Rs. 8 and for the subsequent distance it is Rs. 5 per km. Write a linear equation for this information and draw its graph. Solution. Let total distance covered is x km. Let total fare is Rs. y Since, fare for Ist km is Rs. 8 and for remaining (x1) km. is Rs. 5 per km. By given information, we have 8 1 + 5 (x 1) = y 5x y = 3 or
y 5x 3

Now, equation is y = 5x + 3. Giving different values to x, we get corresponding values of y. Let us represent this in a tabular form.
x 1 2 3 4 y 8 13 18 23

42

LINEAR EQUATIONS IN TWO VARIABLES

MATHEMATICSIX

Example 11. If the work done by a body on application of a constant force is directly proportional to the distance travelled by the body, express this in the form of an equation in two variables and draw the graph of the same by taking the constant force as 5 units. Also, read from the graph the work done when the distance travelled by the body is : (i) 2 units (ii) 0 units. NCERT Solution. Let x be the distance and y be the work done. Then, according to the given problem, we have y = 5x. ( 5 is the constant force). Let us not draw the graph of this linear equation in two varibles. Required table is :
x 0 1 2 y 0 5 10

MATHEMATICSIX

LINEAR EQUATIONS IN TWO VARIABLES

43

(i) From the graph, we see that, x = 2 units distance y = 10 units work done. (ii) From the graph, we see that, x = 0 unit distance y = 0 unit work done. Example 12. Yamini and Fatima, two students of class IX of a school, together contributed Rs. 100 towards the Prime Ministers Relief Fund to help the earthquake victims. Write a linear equation which satisfies this date. (You may take their contributions as Rs. x and Rs. y). Draw the graph of the same. NCERT Solution. Let, Yamini contributed Rs. x and Fatima contributed Rs. y. then, according to the given question, we have, x + y = 100 Required table is : x 20 40 60 y 80 60 40

Every point of the shaded portion including the line x + y = 100, x-axis and y-axis in the first quadrant represent the solution set. 44 LINEAR EQUATIONS IN TWO VARIABLES MATHEMATICSIX

Example 13. In countries like USA and Canada, temperature is measured in Fahrenheit, wheres in countries like India, it is measured in Celsius. Here is a linear equation that converts Fahrenheit to Celsius:
F 9 C 32 . 5 Draw the graph of the linear equation above using Celsius for x-axis and Fahrenheit for yaxis. If the temperature is 30C, what is the temperature in Fahrenheit? If the temperature is 95F, what is the temperature in Celsius? If the temperature is 0C, what is the temperature in Fahrenheit, and if the temperature is 0F, what is the temperature in Celsius? Is there a temperature which is numerically the same in both Fahrenheit and Celsius? If yes, find it. NCERT
9 C 32 . 5

(i) (ii) (iii) (iv) (v)

Solution.

Required table is :

C 10 20 30 F 50 68 86

MATHEMATICSIX

LINEAR EQUATIONS IN TWO VARIABLES

45

(i) Draw the graph of the linear equation above using Celsius for x-axis and Fahrenheit for yaxis. (i) The graph of the line F (ii) (iii) (iv) (v)
9 C 32 is shown in the figure. 5 From the graph : C = 30 F = 86 From the graph : F = 95 C = 35 If C = 0 F = 32 and, If F = 0 C = 17.8 Yes, clearly from the graph, the temperature which is numerically the same is both Fahrenheit and Celcius is 40 F = 40C.

PRACTICE EXERCISE
1. Which of the following are linear : (i) 3x (x 2) = 3x2 + 2x 7 (ii) (x + 1) (x 2) = 10 2 (iv) 2x 2 + 7x 3 = 2x (x + 1) (iii) x (x + 1) = 2x + 7x + 2 2. Write each of the following equation in the form ax + by + c = 0 (i) y 3 = 2 (x 1) (iii)
x 3 y6 2

(ii)

x y5 3

(iv) 2 (x 1) 3 (y + 1) = 1

x 1 3y 1 3 2 4 3. Write four solutions for each of the following : (i) 2x y = 3 (ii) x = 3y 4. Find out which of the following equations have x = 2, y = 1 as a solution :

(iii)

x y 6 2 3

(iv)

(i) 3x + 2y = 4 (iv) 4x y = 9

(ii) 4x y = 8 (v)
x y2 2

(iii) x

y 3 2

(vi) 7x 4y = 18

5. Find the value of k if the given point lies on the graph defined by each equation : (i) 3x + ky = 2 ; (1, 1) (ii) y + kx = 8 ; (3, 2) (iii) 2x 3y = k ; (1, 5) (iv) 2x k = 4y ; (2, 0) 6. Show that x = 1, y = 6 ; x = 2, y = 3 and x = 3, y = 0 are all solution of equation 3x y = 9. 7. The equation of a graph is given by 2x + y = 10. Indicate which of the following points lies on the graph. (i) (6, 2) (ii) (2, 8) (iii) (2, 6) (iv) (3, 16) (v) (6, 18) (vi) (6, 22) 8. Draw the graph of line 2x + 5y = 13. Is the point (9, 1) lies on the line? 9. Draw the graph of y = 2x + 4. Find co-ordinates of point at which it intersects the axes. 10. Draw the graph of y = 3x + 2 and y = 3x 1 using the same pair of axes. Are these two lines parallel? 11. Draw the graph of 3x + 2y = 7 and 2x 3y = 10 using the same pair of axes. Are these two lines perpendicular? 46 LINEAR EQUATIONS IN TWO VARIABLES

MATHEMATICSIX

12. Draw the graph of 3x y = 5 and 2x + 3y = 7 on the same axes. What is their point of intersection? 13. Graph the following equations : (i) 2y = x + 3 (ii)
x 1 y 2 3

(iii) 3x 4y = 12

(iv) y = |x|

14. Amit invests Rs. 100 at the rate of 5% p.a. simple interest. Assuming rate to be same, find graphically the interest he will earn after 5 years? 15. Unit of temperature measurement, Fahrenheit and celsius are related by relation F (i) (ii) (iii) (iv)
9 C 32 . 5

Draw graph of linear equation above, using celsius for x-axis and Fahrenheit for y-axis. If the temperature is 35C, what is the temperature in Fahrenheit? If the temperature is 86F, what is the temperature in celsius? Is there a temperature which is numerically the same in both units? If yes, find it.

PRACTICE TEST

M.M : 15 General Instructions : All questions carry 3 marks each.

Time : hour

1. Give three integral solutions for equation 5x y = 9. 2. Find the value of a so that the equation 4x ay = 7 have (2, 3) as a solution. 3. Give the geometrical representation of x = 3 as an equation. (i) In one variable (ii) In two variables 4. Yamini and Fatima, together contributed Rs. 100 towards PM relief fund. Write a linear equation which this data satisfies. Draw the graph of the same. 5. Draw the graph of 3x + y = 8. Is x = 1, y = 11, a solution of this equation. Also, shade the portion bounded by this line and both the axes.

ANSWERS OF PRACTICE EXERCISE


1. (i) and (iv) 2. (i) 2x y + 1 = 0 (ii) x 3y 15 = 0 (iii) x 2y 15 = 0 (iv) 2x3y6=0 (v) 3x 2y 36 = 0 (vi) 2x 3y 9 = 0 3. (i) (0, 3), (1, 1), (2, 1), (1, 5) 4. (i), (iv), (v), (vi) 5. (i) k = 1 (ii) k = 2 (iii) k = 17 (iv) k = 4 7. (i), (iii), (iv), (vi) (ii) (3, 1), (6, 2), (9, 3), (12, 4)

MATHEMATICSIX

LINEAR EQUATIONS IN TWO VARIABLES

47

8. Yes

9. (0, 4) and (2, 0)

48

LINEAR EQUATIONS IN TWO VARIABLES

MATHEMATICSIX

10. Yes

11. Yes

12. (2, 1)

13. (iv) y = | x |

MATHEMATICSIX

LINEAR EQUATIONS IN TWO VARIABLES

49

14.

15.

From graph, we observe that after 5 years, he will receive Rs. 25.

From graph, we have : (ii) 95F (iii) 30C (iv) Yes, 40

ANSWERS OF PRACTICE TEST


1. (1, 4), (2, 1) and (3, 6) 2. a
1 3

--3 3. (i) (ii)

50

LINEAR EQUATIONS IN TWO VARIABLES

MATHEMATICSIX

4. x + y = 100

where, x Rs. = contribution by Yamini and y Rs. = contribution by Fatima. 5. Yes

MATHEMATICSIX

LINEAR EQUATIONS IN TWO VARIABLES

51

CHAPTER

INTRODUCTION TO EUCLIDS GEOMETRY


Points to Remember : 1. A point, a line and a plane are concepts only and these terms are taken as undefined. 2. Axioms (or Postulates) are assumptions which are self evident truths. 3. Theorems are the statements which are proved, using axioms, previously proved statements and deductive reasoning. 4. Some of Euclids axioms were: (a) Things which are equal to the same thing are equal to one another. (b) If equals are added (or subtracted) to / from equals, the wholes / remainders are equal. (c) The whole is greater than the part. (d) Things which are double of the same things are equal to one another. 5. Euclids Five Postulates: Postulate 1: A straight line may be drawn from any one point to any other line. Postulate 2: A terminated line can be produced indefinitely. Postulate 3: A circle can be drawn with any center and any radius. Postulate 4: All right angles are equal to one another. Postulate 5: If a straight line falling on two straight lines makes the interior angles on the same side of it taken together less than two right angles, then the two straight lines, if produced indefinitely, meet on that side on which the sum of angles is less than two right angles. 6. Two equivalent versions of Euclids fifth Postulate: (a) Play fair axiom: Through a given point, not on the line, one and only one line can be drawn parallel to a given line. (b) Two distinct intersecting lines cannot be parallel to the same line.

ILLUSTRATIVE EXAMPLES
Example 1. Prove that an equilateral triangle can be constructed on any given line segment. Solution. Draw a line segment, says PQ. Using Euclids postulate 3, draw a circle with center P and radius PQ. Again, we draw another circle with center Q and radius QP. The two circles meet at a point, says R. Join P to R and Q to R. now, in PQR, PQ = PR and PQ = QR. So, by axiom 4, we get PQ = QR = PR. Hence, PQR is an equilateral triangle. 52 INTRODUCTION TO EUCLIDS GEOMETRY MATHEMATICSIX

Example 2. If a point C lies between two points A and B such that AC = BC, then prove that AC = Solution. Since, C lies between AB. AB = AC + BC ( by addition axiom) AB = AC + AC ( AC = BC) AB = 2AC AC =

1 AB. 2

1 AB. 2 which prove the result. Example 3. Let point C be a mid-point of line segment AB. Prove that every line segment has one and only one mid-point. Solution. Let if possible, D be another mid-point of AB. AD = DB ....(1) But, it is given that C is the mid-point of AB. AC = CB ...(2) Subtracting (1) from (2), we get AC AD = CB DB DC = DC 2DC = 0 DC = 0 C and D must coincides. Thus, every line segment has one and only one mid-point. Example 4. In figure, if AC = BD, then prove that AB = CD. Solution. AC = BD ...(1) ( given ) Also, AC = AB + BC ...(2) ( point B lies between A and C) and, BD = BC + CD ...(3) ( point C lies between B and D) Substituting for AC and BD from (2) and (3) in (1), We get, AB + BC= BC + CD AB + BC BC = BC + CD BC ( subtracting BC both sides) AB = CD Hence proved. Example 5. Which of the following statements are true and which are false? Give reasons for your answers. (i) Only one line can pass through a single point. (ii) There are infinite number of lines which pass through two distinct points. (iii) A terminate line can be produced indefinitely on both sides. (iv) If two circles are equal, then their radii are equal. (v) In figure, if AB = PQ and PQ = XY then AB = XY.

NCERT T Solution. (i) False, since, through a point infinite number of lines may be drawn. (ii) False, since one and only one line can pass through two distinct points. (iii) True, since a line can be produced infinitely on both the ends. (iv) True, since two circles will be equal only when their radii are the same. (v) True, since AB = PQ, PQ = XY AB = XY. (By transitive property) INTRODUCTION TO EUCLIDS GEOMETRY 53

MATHEMATICSIX

PRACTICE EXERCISE
1. 2. 3. 4. 5. 6. 7. What is the difference between axiom and a theorem? Give any three axioms given by Euclid. What are Euclids five postulates? State Play fairs axiom. Give two equivalent versions of Euclids fifth postulate. If P,Q and R are three points on a line, and Q lies between P and R, then prove that PQ + QR = PR. Prove that an equilateral triangle can be constructed on any given segment.

8. Prove that, Two distinct lines cannot have more than one point in common.

54

INTRODUCTION TO EUCLIDS GEOMETRY

MATHEMATICSIX

CHAPTER

6
LINES AND ANGLES

Points to Remember : 1. If a ray stands on a line, then the sum of the two adjacent angles so formed is 180 and vice-versa. This property is known as the Linear Pair Axiom. 2. If two lines intersect each other, then the vertically opposite angles are equal. 3. If a transversal intersects two parallel lines, then (a) each pair of corresponding angles is equal. (b) each pair of alternate interior angles is equal. (c) each pair of interior angles on the same side of the transversal is supplementary. 4. If a transversal intersects two line such that, either (a) any one pair of corresponding angles is equal, or (b) any one pair of alternate interior angles is equal, or (c) any one pair of interior angles on the same side of of the transversal is supplementary, then the lines are parallel. 5. Two intersecting lines cannot both be parallel to the same line. 6. Lines which are parallel to a given line are parallel to each other. 7. The sum of three angles of a triangle is 180. 8. If a side of a triangle is produced, the exterior angle so formed is equal to the sum of the two interior opposite angles.

ILLUSTRATIVE EXAMPLES
Example 1. If two lines intersects, prove that vertically opposite angles are equal. Solution. Given : Two lines AB and CD intersects at O. To prove : 1 = 3 and 2 = 4 Proof : OB stands on line COD. 1 + 2 = 180 ( linear pair) ...(1) Also, OD stands on line AOB 2 + 3 = 180 ( linear pair) ...(2) from (1) and (2) 1 + 2 = 2 + 3 1 = 3 Similarly, 2 = 4 Hence proved Example 2. The complement of an angle is half of itself. Find the angle and its complement. Solution. Let the given angle be x. Then, its complement is 90 x. According to given question, 90 x =
1 x 2 x x 90 2

MATHEMATICSIX

3x 2 90 90 x 2 3

LINES AND ANGLES

55

x = 60 Hence, the given angle is 60 and its complement is 90 60 i.e. 30. Example 3. Two supplementary angles are in the ratio 3 : 2. Find the angles. Solution. Let the angles be 3x and 2x. according to given question, 3x + 2x = 180 ( angles are supplementary) 5x = 180
x 180 36 5

Angles are 3 x 3 36 108 Ans. and 2 x 2 36 72

Example 4. What value of x would make AOB a line in given figure if AOC = (3x 10) and BOC = (7x + 30)?

Solution.

Since, OB and OA are opposite rays BOC + AOC = 180 7x + 30 + 3x 10 = 180 10x + 20 = 180 10x = 180 20 10x = 160 x
160 10

x 16 Ans.

Example 5. Rays OA, OB, OC, OD and OE have the common initial point O. Show that : AOB + BOC + COD + DOE + EOA = 360 Solution. Let us draw a ray OF oposite to ray OA.
C 3 4 D 2 0 5 1 6 E B

so, 1 + 2 + 3 = 180 and 4 + 5 + 6 = 180 ( linear pair axiom) adding (1) and (2), (1 + 2 + 3) + (4 + 5 + 6) = 180 + 180 1 + 2 + (3+ 4) + 5 + 6 = 360 AOB + BOC + COD + DOE + EOA = 360 Hence shown. 56 LINES AND ANGLES

...(1) ...(2)

MATHEMATICSIX

Example 6. Prove that if a transversal intersects two parallel lines, then each pair of interior angle on the same side of the transversal is supplementary. Solution. Given : A transversal l intersects two parallel lines AB and CD at P and Q.

4 + 5 = 180 and 3 + 6 = 180 Proof : Ray QD stands on line l 1 + 4 = 180 ...(1) ( linear pair axiom) also, 1 = 5 ...(2) ( corresponding angles) from (1) and (2) 4 + 5 = 180 Again, ray QC stands on line l 2 + 3 = 180 ...(3) ( linear pair axiom) and, 2 = 6 ...(4) ( corresponding angles) from (3) and (4) 3 + 6 = 180 Hence proved. Example 7. If two parallel lines are intersected by a transversal, prove that the bisectors of the two pairs of interior angles encloses a rectangle. Solution. Given : Two parallel lines AB and CD are intersected by a transversal at P and R respectively. PQ, RQ, RS and PS are bisectors of APR, PRC, PRD and BPR respectively. To prove :

To prove : PQRS is a rectangle Proof : Since AB || CD and l is a transversal APR = PRD ( alt. interior angles)
1 1 APR = PRD QPR = PRS 2 2

But, these are alternate interior angles. PQ || RS. Similarly QR || PS. PQRS is a parallelogram. MATHEMATICSIX LINES AND ANGLES 57

Now, ray PR stands on AB APR + BPR = 180

( linear pair)

1 1 APR + BPR = 90 2 2 QPR + SPR = 90 QPS = 90 Thus, PQRS is a parallelogram, one of whose angle is 90 PQRS is a rectangle. Hence proved. Example 8. In the given figure lines XY and MN intersect at O. If POY = 90 and a : b = 2 : 3, find c.

Solution.

NCERT Since XY is a line a + b + 90 = 180 ( linear pair) a + b = 180 90 a + b = 90 But a : b = 2 : 3. Let a = 2x and b = 3x. 2x + 3x = 90 5x = 90 x = 18 a = 2x = 2 18 = 36 and b = 3x = 3 18 = 54 Now, OM and ON are opposite rays. MON is a line. Since ray OX stands on MN MOX + XON = 180 ( linear pair) c + b = 180 c + 54 = 180 C = 180 154 = 126 c = 126 Ans. Example 9. It is given that XYZ = 64 and XY is produced to point P. Draw a figure from the given information. If ray YQ bisects ZYP, find XYQ and reflex QYP. NCERT Solution. Since XY is produced to point P, XP is a straight line. Since, YZ stands on XP, XYZ + ZYP = 180 ( linear pair) 64 + ZYP =1 80 ZYP = 180 64 = 116 Since ray YQ bisects ZYP
116 58 2 Now, XYQ = XYZ + ZYQ XYQ = 64 + 58 = 122 and reflex QYP = 360 QYP = 360 58 = 302 Ans.

QYP ZYQ

58

LINES AND ANGLES

MATHEMATICSIX

Example 10. In the given figure, if AB || CD, CD || EF and y : z = 3 : 7, find x.

NCERT

Since, AB || CD and CD || EF AB || CD || EF. Now, CD || EF and PR is a transversal. PQD = PRF 180 y = z y + z = 180 also, y : z = 3 : 7. Let y = 3a and z = 7a 3a + 7a = 180 10a = 180 a = 18 y = 3 18 = 54 and z = 7 18 = 126 Now, AB || CD and PQ is a transversal. x + y = 180 ( consecutive interior angles are supplementary) x + 54 = 180 x = 180 54 = 126 x = 126 Ans. Example 11. In the given figure, X = 62, XYZ = 54. If YO and ZO are the bisectors of XYZ and XZY respectively of XYZ, find OZY and YOZ. NCERT Solution.

Solution.

In XYZ, YXZ + XYZ + XZY = 180 ( angle sum property of a triangle) 62 + 54 + XZY = 180 XZY = 180 62 54 = 64 Since, YO and ZO are bisectors of XYZ and XZY,
OYZ 1 1 XYZ 54 27 2 2

1 1 XZY 64 32 2 2 In OYZ, we have YOZ + OYZ + OZY = 180 ( angle sum property of a triangle) YOZ + 27 + 32 = 180 YOZ = 180 27 32 = 121 Hence, OZY = 32 and YOZ = 121 Ans.

and, OZY

MATHEMATICSIX

LINES AND ANGLES

59

Example 12. In the given figure, if PQ PS, PQ || SR, SQR = 28 and QRT = 65, then find the values of x and y. NCERT

Solution.

QRT = RQS + QSR ( Exterior angle property in SRQ) 65 = 28 + QSR QSR = 65 28 = 37 Since, PQ || SR and the transversal PS intersects then at P and S respectively. PSR + SPQ = 180 ( sum of consecutive interior angles is 180) (PSQ + QSR) + 90 = 180 y + 37 + 90 = 180 y = 180 37 90 = 53 Now, in the right SPQ, we have PQS + PSQ = 90 x + 53= 90 x = 90 53 = 37 Hence, x = 37 and y = 53 Ans. Example 13. In each of the following figures, AB || CD, find x.
A B 50

O 25 D

C (i)

Solution.

(i) Through O, Draw EOF || AB || CD. then, 1 + 2 = x Now, EO || AB and BO is the transversal. 1 + 50 = 180 ( Interior angles on the same side of the transversal)
A B 50 1 O 2 C

F 25 D

60

LINES AND ANGLES

MATHEMATICSIX

1 = 180 50 1 = 130 Again, EO || CD and OD is a transversal 2 + 25 = 180 2 = 180 25 2 = 155 adding (1) and (2) 1 + 2 = 130 + 155 = 285 i.e. x 285 Ans. (ii) Produce AB to intersect CE and F.
C A B 125 F x E

...(1)

...(2)

D 105 G

Now, ABE + EBF = 180

( Linear pair)

125 + EBF = 180 EBF = 180 125 = 55 Again, CD || FG and CF is a transversal DCF + CFG = 180 ( Interior angles on same side of transversal) 105 + CFG = 180 CFG = 180 105 = 75 Also, BFE = CFG = 75 ( vertically opposite angles) Now, In BEF, BEF + EBF + BFE = 180 ( angle sum property) x + 55 + 75 = 180 x + 130 = 180 x = 180 130

x 50 Ans.

Example 14. The side QR of PQR is produced to a point S. If the bisectors of PQR and PRS meet at point 1 NCERT T, then prove that QTR = QPR. 2 Solution. Side QR of PQR is produced to S, Ext. PRS =P + Q ( Exterior angle sum property of a triangle)
1 1 1 Ext PRS = P + Q 2 2 2
P T

1 P + 1 ...(1) 2 Again, In QRT., Ext. TRS = T + 1 ( same as above) 2 = T + 1 ...(2)

2 =

1 1 Q

2 2 R S

MATHEMATICSIX

LINES AND ANGLES

61

Equating (1) and (2), we get 1 P + 1 = T + 1 2 1 1 T = P or QTR = QPR 2 2 Hence Proved. Example 15. Bisectors of exterior angles of ABC (obtained by producing sides AB and AC) meet at O. Prove 1 that BOC = 90 A. 2 Solution. Given : ABC in which bisectors of exterior angles meet at O (as shown) 1 To prove : BOC = 90 A 2 Proof : In OBC 1 + 2 + 0 = 180 ( angle sum property of a triangle) 1 1 CBD + BCE + O = 180 2 2 CBD + BCE + 2O = 360 (180 B) + (180 C) + 2O = 360 B + C = 2O 180 A = 2O 1 O = (180 A) 2 1 BOC = 90 A. Hence proved. 2

PRACTICE EXERCISE
1. 2. 3. 4. 5. 6. If an angle is 24 less than its complement. find its measure. An angle is 40 less than one-third of its supplement. Find the angle and its supplement. Two supplementary angles are in the ratio 11 : 7. Find them. Find the angle whose supplement is four times its complement. Find the measure of an angle if, three times its supplement is 60 more than six times its complement. In the following figure, it is given that 2a 5b = 10 find a and b.

a A

b B

7. In the following figures, what value of x will make AOB a straight line?

62

LINES AND ANGLES

MATHEMATICSIX

8. In thefollowing figure, COA = 90 and AOB is a straight line. Find x and y.

9. Prove that the bisectors of the angles of a linear pair are at right angles. 10. If the bisectors of two adjacent angles form a right angle then prove that their non-common arms are in the same straight line. 11. In given figure, AOB is a line. Ray OD is perpendicular to AB . OC is another ray lying between OA and OD. Prove that DOC =
1 (BOC AOC) 2
D

90 A B

12. In the following figure, AB and CD intersects at O and BOE = 70. Find value of a, b and c.

13. In the given figure 1 : 2 = 5 : 4 and AB || CD. Find all the labelled angles.

MATHEMATICSIX

LINES AND ANGLES

63

14. Find missing x in the following diagrams. Given AB || CD.

B 140 85 x E

(v) 15. In the following figure AB || CD. Find the values of a, b and c.

16. In the following figure, show that AB || CD.


E 50 C 160 D F

30 20

64

LINES AND ANGLES

MATHEMATICSIX

17. In the following figure, AB || CD and CD || EF. Also EA AB. If BEF = 65, find the value of a, b and c.

x x 18. The angles of a triangle are 50 , 60 and (2x 15), find the angles. 2 3

19. If a transversal cuts two parallel lines and is perpendicular to one of them, show that it will be perpendicular to the other also. 20. If two parallel lines are intersected by a transversal, show that the bisectors of any corresponding angles are parallel. 21. If two lines are intersected by a transversal in such a way that the bisectors of a pair of corresponding angles are parallel, show that the lines are parallel to each other. NCERT 22. Prove that the bisectors of a pair of alternate angles of two parallel lines are themselves parallel. 23. Prove that if the arms of an angle are, respectively, parallel to the arms of another angle, then the angles either have equal measure or they are supplementary. 24. PQ and RS are two mirrors placed parallel to each other. An incident ray AB strikes the mirror PQ at B, the reflected ray moves along the path BC and strikes the mirror RS at C and again reflects back along CD. Prove that AB || CD. NCERT
P B Q D

A R C

25. If two straight lines are perpendicular to the same line, prove that they are parallel to each other. 26. One of the angles of a triangle is 55. Find the remaining two angles, if their diffrence is 35. 27. In the given figures find x :

MATHEMATICSIX

LINES AND ANGLES

65

28. Sides QP and RQ of PQR are produced to points S and T respectively. If SPR = 140 and POT = 105, find PRQ.

29. A square ABCD is surmounted by an equilateral triangle EDC. Find x.


E x

30. In the following figure, AB || DC. If x


B y x

y 4 and y z . Find the values of x, y and z. 2 9


z C

D A

PRACTICE TEST
MM : 30 General Instructions : Q. 1-4 carry 2 marks, Q. 5-8 carry 3 marks and Q. 9-10 carry 5 marks each. 1. What value of x would make AOB a straight line?
C D

Time : 1 hour

(2x15) (3x + 3) A O x

66

LINES AND ANGLES

MATHEMATICSIX

2. If two parallel lines are intersected by a transversal, prove that the bisectors of two interior alternate angles are parallel. 3. Find the value of x.

4. Find the value of x, it is given that AB || CD.

5. If two straight lines are perpendicular to the same line, prove that they are parallel to each other. 6. In the given figure, show that AB || CD.
A 80 C 145 D B

45 35

7. In the following figure, AB || CD, find a and b.

A 55

E b

C F

135 G D

8. Angles A, B and C of a triangle satisfy B A = 30 and C B = 45. Find all the angles. 9. Prove that the sum of three angles of a triangle is 180. MATHEMATICSIX LINES AND ANGLES 67

10. In the given figure, the sides AB and AC of ABC are produced to points E and D respectively. If bisectors BO and CO of CBE and BCD respectively meet at O, then prove that : BOC = 90

1 BAC 2
A

ANSWERS OF PRACTICE EXERCISE


1. 33 6. 130, 50 2. 15, 165 3. 110, 70 4. 60 8. x = 33, y = 11 (v) x = 135 5. 20 7. (i) x = 20 (ii) x = 33 (iii) x = 15

12. a = 22, b = 44, c = 92 15. a = 100, b = 35, c = 45 18. 65, 70, 45 27. (i) 65 (ii) 95 29. x = 45

13. 1 = 3 = 5 = 7 = 80, 2 = 4 = 6 = 8 = 100 17. a = 25, b = 115, c = 115 26. a = 80, b = 45 28. 65 30. 24, 48, 108

14. (i) x = 115 (ii) x = 40 (iii) x = 55 (iv) x = 80

ANSWERS OF PRACTICE TEST


1. x = 32 7. a = 55, b = 80 3. x = 110 4. 290

8. A = 25, B = 55, C = 100

68

LINES AND ANGLES

MATHEMATICSIX

CHAPTER

7
TRIANGLE

Points to Remember : 1. Two figures are congruent, if they are of same shape and same size. 2. If two triangles ABC and XYZ are congruent under the correspondence A X, B Y and C Z, then symbolically, ABC XYZ 3. SAS Congruence Rule : If two sides and the included angle of one triangle are equal to two sides and the included angle of the other triangle, then the two triangles are congruent. 4. ASA Congruence Rule : If two angles and the included side of one triangle are equal to two angles and the side of the other triangle, then the two triangles are congruent. 5. AAS Congruence Rule : If two angles and one side of one triangle are equal to two angles and the corresponding side of the other triangle, then the two triangles are congruent. 6. RHS congruence Rule : If in two right triangles, hypotenuse and one side of a triangle are equal to the hypotenuse and one side of other triangle, then the two triangles are congruent. 7. SSS Congruence rule : If three sides of one triangle are equal to the three sides of anoter triangle, then the two triangles are congruent. 8. Angles opposite to equal sides of a triangle are equal. 9. Sides opposite to equal angles of a triangle are equal. 10. Each angle of an equilateral triangle is 60. 11. Of all the line segments that can be drawn to a given line from a point not lying on it, the perpendicular line segment is the shortest. 12. In a triangle, angle opposite to the longer side is greater. 13. In a triangle, side opposite the greater angle is longer. 14. Sum of any two sides of a triangle is greater than the third side. 15. Difference between any two sides of a triangle is less than its third side.

ILLUSTRATIVE EXAMPLES
Example 1. In the given figure, AB = AC and ACD = 110, find A.
A

110 B

Solution.

Since AB = AC B = C Now ACB = 180 110 ACB = 70 B = ACB = 70 Now, A B + C = 180

(angles opposite to equal sides are equal) ( linear pair)

(angle sum property of a triangle) TRIANGLES 69

MATHEMATICSIX

A + 70 + 70 = 180 A = 180 140 A 40 Ans.

Example 2. In given figure, AB = CF, EF = BD and AFE = DBC. Prove that AFE CBD. Solution. Given, AB = CF D Add BF both sides, we get AB + BF = CF + BF AF = CB A Now, In AFE and CBD, F B AF = BC (proved above) AFE = CBD (given) and EF = BD (given) E So, by SAS congruence rule, AFE CBD

Example 3. In quadrilateral ABCD, AC = AD and AB bisects A. (see figure). Show that ABC ABD. What can you say about BC and BD? NCERT Solution. In ABC and ABD, we have AC = AD (given) CAB = BAD ( AB bisects A) and, AB = AB (common side) ABC ABD (SAS congruence condition) also, BC = BD (cpct) Example 4. ABCD is a quadrilateral in which AD = BC and DAB = CBA (see figure). Prove that : (i) ABD BAC (ii) BD = AC (iii) ABD = BAC Solution. In ABD and BAC, we have AD = BC (given) DAB = CBA (given) AB = AB (common side) ABD BAC, (SAS congruence condition) which proves (i) part BD = AC and ABD = BAC, which proves (ii) and (iii) (cpct)

NCERT

Example 5. If the bisector of the vertical angle of a triangle bisects the base, prove that the triangle is isosceles. Solution. Given ABC in which AD is the bisector of A which meets BC in D such that BD = DC. To prove : AB = AC Construction : Produce AD to E, such that AD = DE. Join E to C 70 TRIANGLES MATHEMATICSIX

A Proof : In ABD and ECD BD = DC (given) AD = DE (by construction) 1 2 ADB = EDC (vertically opp. angles) ABD ECD (by SAS congruence condition) AB = EC and 1 = 3 (cpct) C B D also, 1 = 2 (AD is angle bisector) 2 = 3 EC = AC (sides opp. to equal angles) 3 AB = AC ( EC = AB) E Hence, ABC is isosceles. Example 6. In a right angled triangle, one acute angle is double the other. Prove that the hypotenuse is double the smallest side. Solution. Given PQR in which Q = 90 and QRP = 2QPR To prove : PR = 2QR Construction : Produce RQ to S such that RQ = QS. Join P to S. Proof : In PSQ and PQR PQ = PQ (common side) PQS = PQR (= 90 each) QS = QR (by construction) PSQ PQR (SAS congruence condition) PS = PR and SPQ = RPQ = x (say) (cpct) SPR = 2x = PRQ = PRS Now, In PSR, SPR = PRD = 2x PS = SR PR = SR ( PS = PR) PR = 2QR ( SR = 2QR) Hence proved. Example 7. In the given figure, PQRS is a quadrilateral and T and U are respectively points on PS and RS such that PQ = QR, PQT = RQU and TQS = UQS. Prove that QT = QU. Solution. PQT = RQU (given) ....(1) TQS = UQS (given) .....(2) adding, (1) and (2) we getPQT + TQS = RQU + UQS or PQS = RQS Now, In PQS and RQS P PQ = RQ (given) T PQS = UQS (proved above) QS = QS (common side) 1 S PQS RQS (SAS congruence condition) Q 2 So, 1 = 2 (cpct) Again, In TQS and UQS U TQS = UQS (given) R 1 = 2 (proved above)

MATHEMATICSIX

TRIANGLES

71

QS = QS (common side) TQS UQS (ASA congruence condition) Hence, QT = QU (cpct) Example 8. Line l is the bisector of an A, and B is any point on . BP and BQ are perpendiculars from B to the arms of A (see figure). NCERT Show that : (i) APB AQB (ii) BP = BQ or B is equidistant from the arms of A. Solution. In APB and AQB, we have APB = AQB ( each = 90) PAB = QAB ( AB bisects PAQ) AB = AB ( common side) APB AQB, (AAS congruence condition) which proves (i) part. and, BP = PQ (cpct) which proves (ii) part. Example 9. In the given figure, AC = AE, AB = AD and BAD = EAC. Show that BC = DE.

NCERT

In ABC and ADE, we have AB = AD (given) BAC = DAE ( BAD = EAC BAD + DAC = EAC + DAC BAC = DAE) and, AC = AE (given) ABC ADE (SAS congruence condition) BC = DE (cpct) Hence shown. Example 10. In right triangle ABC, right angled at C, M is the mid-point of hypotenuse AB. C is joined to M and produced to a point D such that DM = CM. Point D is joined to point B (see figure). Show that (i) AMC BMD (ii) DBC is a right angle (iii) DBC ACB Solution.
1 (iv) CM .AB 2

NCERT Solution. (i) In AMC and BMD, we have AM = BM ( M is the mid-point of AB) AMC = BMD (Vertically opp. angles) CM = MD (given) AMC BMD (SAS congruence condition) TRIANGLES MATHEMATICSIX

72

(ii) Now, AMC BMD BD = CA and BDM = ACM ...(1) (cpct) Thus, transversal CD cuts CA and BD at C and D respectively such that the alternate angles BDM and ACM are equal. BD || CA. CBD + BCA = 180 ( consecutive interior angles on the same side of a transversal are supplementary). CBD + 90 = 180 CBD = 90 or, DBC = 90 (iii) Now, in DBC and ACB, we have BD = CA (from (i)) DBC = ACB ( each = 90) BC = BC (common side) DBC ACB (SAS congruence condition) (iv) CD = AB (cpct) 1 1 CD AB 2 2 1 CM AB. Hence proved. 2 Example 11. In the given figure, AB = AC, DB = DC. Prove that ABD = ACD. Solution. Join A to D A In ABD and ACD, we have AB = AC (given) BD = CD (given) AD = AD (common side) D ABD ACD (SSS congruence condition) C B So, ABD = ACD (cpct) Example 12. In ABC, D is the mid point of base BC. DC and DF are perpendiculars to AB and AC respectively, such that DE = DF. Prove that B = C. Solution. In BED and CFD, we have DE = DF (given) BD = DC (D is mid point of BC) BED = CFD (= 90 each) BED CFD (RHS congruence condition) So, B = C (cpct) Example 13. In an isosceles triangle ABC, with AB = AC, the bisectors of B and C intersect each other at O. Join A to O. Show that : NCERT (i) OB = OC (ii) AO bisects A. Solution. (i) In ABC, we have AB = AC B = C ( angles opposite to equal sides are equal)

1 1 B C 2 2 OBC = OCB
( OB and OC bisect B and C respectively. OBC MATHEMATICSIX
1 1 B and OCB C ) 2 2 OB = OC ( sides opposites to equal angles are equal).

TRIANGLES

73

(ii) Now, In ABO and ACO, we have, AB = AC (given) OBC = OCB (proved above) OB = OC (proved above) ABO ACO (SAS congruence condition) BAO = CAO (cpct) AO bisects BAC. Hence proved. Example 14. ABC is an isosceles triangle in which altitudes BE and CF are drawn to sides AC and AB respectively (see figure). Show that these altitudes are equal. NCERT Solution. In ABC, AB = AC ACB = ABC ECB = FBC Now, In BEC and CFB, we have BEC = CFB (each = 90) ECB = FBC (proved above) BC = BC (common side) BEF CFB (AAS congruence condition) BE = CF (cpct) Hence shown. Example 15. ABC and DBC are two isosceles triangles on the same base BC (see figure). Show that ABD = ACD. NCERT Solution. In ABC, we have AB = AC ABC = ACB ...(1) ( angles opposite to equal sides are equal) Also, In BCD, we have BD = CD DBC = DCB ...(2) ( same reason as above) Adding (1) and (2), we get, ABC + DBC = ACB + DCB ABD = ACD. Hence shown. Example 16. ABC is an isosceles triangle in which AB = AC. Side BA is produced to D such that AD = AB (see figure). Show that BCD is a right angle. NCERT Solution. In ABC, we have AB = AC ACB = ABC ...(1) ( angles opp. to equal sides are equal). Now, AB = AD (given) AD = AC ( AB = AC). Thus, In ADC, we have AD = AC ACD = ADC ...(2) Adding (1) and (2), we get ACB + ACD = ABC + ADC BCD = ABC + BDC ( ADC = BDC) BCD + BCD = ABC + BDC + BCD ( adding BCD both sides) 2BCD = 180 ( angle sum property) BCD = 90 Hence, BCD is a right angle. 74 TRIANGLES MATHEMATICSIX

Example 17. ABC and DBC are two isosceles triangles on the same base BC and vertices A and D are on the same side of BC. (see figure). If AD is extended to intersect BC at P, show that : NCERT (i) ABD ACD (ii) ABP ACP (iii) AP bisects A as well as D. (iv) AP is the perpendicular bisector of BC. Solution. (i) In ABD and DBC, we have AB = AC (given) BD = DC (given) AD = AD (common side) ABD DBC (SSS congruence condition) (ii) In ABP and ACP, we have AB = AC (given) BAP = CAP ( ABD ACD BAD = DAC BAP = PAC) AP = AP (common side) ABP ACP (SAS congruence condition) (iii) Since ABD ACD BAD = DAC AD bisects A AP bisects A ...(1) In BDP and CDP, we have BD = CD (given) BP = PC ( ABP ACP BP = PC) DP = DP (Common side) BDP CDP (SSS congruence condition) BDP = PDC DP bisects D AP bisects D ...(2) from (1) and (2), we get AP bisects A as well as D. (iv) Since AP stands on BC. APB + APC = 180 ( linear pair) But, APB = APC 180 90 APB = APC 2 Also, BP = PC (proved above) AP is perpendicular bisector of BC. Example 18. Two sides AB and BC and median AM of one triangle ABC are respectively equal to sides PQ and QR and median PN of PQR (see figure). Show that : NCERT (i) ABM PQN (ii) ABC PQR

MATHEMATICSIX

TRIANGLES

75

Solution.

Since, BC = QR

1 1 BC QR BM QN ...(1) 2 2 Now, In ABM and PQN, we have AB = PQ (given) BM = QN (from (1)) AM = PN (given) ABM PQN, (SSS congruence condition) which proves (i) part. B = Q (cpct) ...(2) Now, In ABC and PQR, we have AB = PQ (given) B = Q (from (2)) BC = QR (given) ABC PQR (SAS congruence condition) which proves (ii) part. Example 19. BE and CF are two equal altitudes of a triangle ABC. Using RHS congruence rule, prove that the triangle ABC is isosceles. NCERT Solution. In BCF and CBE, we have BFC = CEB ( each = 90) hyp. BC = hyp. BC (common side) FC = EB BCF CBE ( RHS congruence condition) FBC = ECB (cpct) Now, In ABC ABC = ACB ( FBC = ECB) AB = AC ABC is an isosceles triangle. Example 20. In the given figure, the line segment joining the mid points M and N of opposite sides AB and DC of quadrilateral ABCD is perpendicular to both these sides. Prove that the other two sides of quadrilateral are equal. Solution. Join M to D and M to C. In MND and MNC M DN = CN (given) A B MND = MNC (each = 90) MN = MN (common side) MND MNC (RHS congruence condition) DM = CM and DMN = CMN ....(1) Now, AMN = BMN (= 90 each) ....(2) Subtracting (1) from (2), we get D C N AMN DMN = BMN CMN AMD = BMC Now, In AMD and BMC, we have AM = BM (given)

76

TRIANGLES

MATHEMATICSIX

MD = MC (proved above) AMD = BMC (proved above) AMD BMC (SAS congruence condition) So, AD = BC (c.p.c.t.) i.e. other two sides of the quadrilateral are equal. Example 21. In figure, B < A and C < D. Show that AD < BC.

NCERT

Since, B < A and C < D AO < BO and OD < OC ( side opp. to greater angle is larger) Adding these results, we get AO + OD < BO + OC AD < BC. Hence shown. Example 22. AB and CD are respectively the smallest and longest sides of a quadrilateral ABCD (see figure). Show that A > C and B > D. NCERT Solution. Join A to C and B to D. Since, AB is the smallest side of quadrilateral ABCD. In ABC, we have BC > AB 8 > 3 ...(1) ( angle opp. to longer side is greater). Since, CD is the longest side of quadrilateral ABCD, In ACD, we have CD > AD 7 > 4 ...(2) ( angle opp. to longer side is greater). Adding (1) and (2), 8 + 7 > 3 + 4 A > C Again, In ABD, we have AD > AB ( AB is the shortest side) 1 > 6 ...(3) In BCD, we have CD > BC ( CD is the longest side) 2 > 5 ...(4) Adding (3) and (4), we get 1 + 2 > 5 + 6 B > D Thus, A > C and B > D. Hence shown. Example 23. In the given figure, PR > PQ and PS bisects QPR. Prove that PSR > PSQ. NCERT Solution.

MATHEMATICSIX

TRIANGLES

77

Solution.

In PQR, we have PR > PQ (given) PQR > PRQ ( Angle opposite to larger side is greater) PQR + 1 > PRQ + 1 (adding 1 both sides) PQR + 1 > PRQ + 2 ...(1) ( PS is the bisector of P 1 = 2) Now, In PQS and PSR, we have PQR + 1 + PSQ = 180 and PRQ + 2 + PSR = 180 PQR + 1 = 180 PSQ and PRQ + 2 = 180 PSR 180 PSQ > 180 PSR (Using (1)) PSQ > PSR PSQ < PSR i.e. PSR > PSQ. Hence proved. Example 24. In the given figure, ABC is a triangle and D is any point in its interior. Show that : BD + DC < AB + AC Solution. Extend BD to meet AC in E. A In ABE, AB + AE > BE ( sum of any two sides is greater than the third side) AB + AE > BD + DE ....(1) E In CDE, DE + EC > DC ....(2) D from (1) and (2), we get AB + AE + DE + EC > BD + DE + DC B C AB + (AE + EC) > BD + DC AB + AC > BD + DC BD + DC > AB + AC Hence shown. Example 25. In the given figure PQRS is a qudrilateral in which diagonals PR and QS intersect in O. Show that (i) PQ + QR + RS + SP > PR + QS (ii) PQ + QR + RS + SP < 2 (PR + QS) Solution. In PSR, we have PS + SR > PR In PQR, PQ + QR > PR adding (1) and (2), we get PQ + QR + PS + SR > 2PR In PSQ, PS + PQ > SQ In RSQ, RS + RQ > SQ adding (4) and (5), we get PQ + RQ + RS + SP > 2SQ adding (3) and (6), we get 2 (PQ + QR + RS + SP) > 2 (PR + SQ) PQ + QR + RS + SP > PR + SQ which proves (i) part 78 ....(7) MATHEMATICSIX ....(6) ....(3) ....(4) ....(5)
P Q

....(1)
S R

( sum of any two sides is greater than the third side) ....(2)

TRIANGLES

Again, In OPQ, OP + OQ > PQ In OQR, OR + OQ > RQ In OSR, OS + OR > SR In OPS, OP + OS > PS adding (8), (9), (10), (11), we get 2 (OP + OQ + OR + OS) > PQ + QR + RS + SP or or 2 (PQ + QS) > PQ + QR + RS + SP PQ + QR + RS + SP < 2 (PR + QS)

....(8) ....(9) ....(10) ....(11)

which proves (ii) part.

PRACTICE EXERCISE
1. In PQR, P = 80 and PQ = PR. Find Q and R. 2. Prove that the measure of each angle of an equilateral triangle is 60. 3. In ABC, BAC = 50 and AB = AC. If ACD = x, find the value of x.
A 50

(Ans. 50, 50) (Ans. x = 115)

x C D

4. If the altitude from one vertex of a triangle bisects the opposite side, prove that the triangle is isosceles. 5. Prove that the perpendiculars drawn from the vertices of equal angles of an isosceles triangle to the opposite sides are equal. 6. Prove that medians of an equilateral triangle are equal. 7. If S is the mid-point of the hypotenuse PR of a right-angled PQR. Prove that QS =
A

1 PR 2

8. In the given figure, it is given that AE = AD and CE = BD. Prove that ABE ACD.

E O B

9. In the given figure, D and E are the points on the base BC of ABC such that BD = CE, AD = AE and ADE = AED. Prove that ABE ACD.
A

MATHEMATICSIX

TRIANGLES

79

10. Equilateral triangles ABD and ACE are drawn on the sides of a ABC. Prove that CD = BE. 11. BD is the bisector of ABC. P is any point on BD. Prove that the perpendiculars drawn from P to AB and BC are equal. 12. In the given figure, AD is the median and BE and CF are drawn perpendicular on AD and AD produced from B and C respectively. Prove that BE = CF.

13. In an isosceles ABC, in which AB = AC, the bisector of B and C meet at I. Show that (i) BI = CI (ii) AI is the bisector of A 14. If the altitudes AD, BE and CF of a ABC are equal, prove that ABC is equilateral. 15. Prove that diagonals of a rhombus bisect each other at right angle. 16. A point O is taken inside a rhombus ABCD such that its distances from the angular points A and C are equal. Show that BO and DO are in one and the same straight line. 17. In the given figure, m || n and E is the mid-point of AB. Prove that E is also the mid-point of any line segment CD having its end points at m and n respectively.

18. In the given figure, C is the mid-point of AB. If ACD = BCE and CBD = CAE, prove that DC = EC.

80

TRIANGLES

MATHEMATICSIX

19. In the given figure, EDC is an equilateral triangle and DABC is a square. Prove that : (i) EA = EB (ii) EBC = 15
E

20. In the given figure, CE AB and DF AB, and CE = DF. Prove that OC = OD.
C

F O E

21. Prove that in a right triangle, the hypotenuse is the longest side. NCERT 22. In the given figure, D is a point on the side of ABC such that AD = AC. Prove that AB > AD.
A

23. Prove that the sum of three altitudes of a triangle is less than the sum of the three sides of a triangle. 24. AD is a median of ABC. Prove that AB + AC > 2AD. 25. In the given figure, D is a point on side AB of ABC and E is a point such that BD = DE. Prove that : AC + BC > AE
A

MATHEMATICSIX

TRIANGLES

81

PRACTICE TEST
M.M : 30 General Instructions : Q. 1-4 carry 2 marks, Q. 5-8 carry 3 marks and Q. 9-10 carry 5 marks each. 1. Prove that the angles opposite to the equal sides of a triangle are equal. 2. In ABC, D is the mid-point of BC. If DL AB and DM AC such that DL = DM, prove that AB = AC. 3. In the given figure, CPD = BPD and AD is the bisector of BAC, prove that CAP BAP.
C P D

Time : 1 hour

4. In the given figure, ABCD is a square. E and F are the mid-points of the sides AD and BC respectively. Prove that BE = DF.
D C

F E

5. In the given figure AB and CD are respectively the smallest and the longest sides of a quadrilateral ABCD. Prove that A > C and B > D.

6. ABC is an isosceles triangle in which AB = AC. Side BA is produced to D such that AD = AB. Show that BCD = 90
D

82

TRIANGLES

MATHEMATICSIX

7. Show that of all the line segments drawn from a given point not on it, the perpendicular line segment is the shortest. 8. AD and BE are respectively altitudes of an isosceles triangle ABC with AC = BC. Prove that AE = BD. 9. ABC is a right angled triangle at C. M is the mid-point of AB. Prove that CM =

1 AB 2

10. Prove that Two triangles are congruent if any two angles and the included side of one triangle is equal to any two angles and the included side of the other triangle.

MATHEMATICSIX

TRIANGLES

83

CHAPTER

8
QUADRILATERALS

Points to Remember : 1. The sum of the angles of a quadrilateral is 360. 2. A diagonal of a parallelogram divides it into two congruent triangles. 3. In a Parallelogram: (i) opposite sides are equal (ii) opposite angles are equal (iii) diagonals bisect each other 4. A quadrilateral is a parallelogram, if (i) opposite sides are equal, or (ii) opposite angles are equal, or (iii) diagonals bisect each other, or (iv) a pair of opposite sides is equal and parallel. 5. Diagonals of a rhombus bisect each other at a right angle and vice-versa. 6. Diagonals of a rectangle bisect each other and are equal, and vice-versa. 7. Diagonals of a square bisect each other at right angles and are equal, and vice-versa. 8. Mid-point Theorem : The line segment joining the mid-points of any two sides of a triangle is parallel to the third side and is half of it. 9. A line through the mid-point of a side of a triangle parallel to another side bisects the third side. 10. The quadrilateral formed by joining the mid-points of hte sides of a quadrilateral, in order, is a parallelogram.

ILLUSTRATIVE EXAMPLES
Example 1. Four angles of a quadrilateral are in the ratio 1 : 2 : 3 : 4. Find them. Solution. Let the four angles be x, 2x, 3x and 4x. Since, sum of four angles of a quadrilateral is 360, x + 2x + 3x + 4x = 360 10x = 360, or x = 36 four angles are 36, 2 36, 3 36, 4 36 i.e. 36, 72, 108, 144 Ans. Example 2. In the given figure, PQRS is a trapezium in which PQ || SR. If P = 60 and Q = 75, find S and R.

84

QUADRILATERALS

MATHEMATICSIX

P + S = 180 and Q + R = 180 ( interior angles on the same side of transversal are supplementarly) 60 + S = 180 and 75 + R = 180 S = 180 60 and R = 180 75 S = 120 and R = 105 Ans. Example 3. Show that the diagnomals of a rhombus are perpendicular to each other. Solution. Let ABCD be a given Rhombus. here, AB = BC = CD = DA Now, In AOD and COD, OA = OC ( diagonals of parallelogram bisect each other) OD = OD (common side) AD = CD Solution. AOD COD (SSS congruence condition) AOD = COD (c.p.c.t.) But, AOD + COD = 180 (linear pair) 2AOD = 180 AOD = 90 So, the diagonals of a rhombus are perpendicular to each other. Example 4. If the diagonals of a parallelogram are equal, then show that it is a rectangle. NCERT Solution. Given : A parallelogram ABCD in which AC = BD. To prove : ABCD is a rectangle. Proof : In ABC and DCB, we have AB = DC (opp. sides of parallelogram) BC = BC (common side) AC = DB (given) ABC DCB (SSS congruence condition) ABC = DCB ...(1) (cpct) But AB || DC and BC cuts them. ACB + DCB = 180 (sum of consecutive interior angles is 180) 2ABC = 180 ABC = 90 Thus ABC = DCB = 90 ABCD is a parallelogram one of whose angle is 90. Hence, ABCD is a rectangle. Example 5. Show that if the diagonals of a quadrilateral bisect each other at right angles, then it is a rhombus. NCERT Solution. Given : A quadrilateral ABCD in which the diagonals AC and BD intersect at O such that AO = OC, BO = OD and AC BD. To prove : ABCD is a rhombus. Proof : In AOD and COB, we have AO = OC (given) OD = OB given) AOD = COB (vertically opp. angles) AOD COB (SAS congruence condition) OAD = OCB ...(i) (cpct) MATHEMATICSIX QUADRILATERALS 85

Now, line AC intersects AD and BC at A and C respectively such that AOD = OCB (proved in (i)) i.e. alternate interior angles are equal. AD || BC. Similarly, AB || CD Hence, ABCD is a parallelogram. Again, In AOD and COD, we have OA = OC (given) AOD = COD (each = 90) OD = OD (common side) AOD COD (SAS congruence condition) AD = CD ...(2) (cpct) Now, ABCD is a parallelogram, AB = CD and AD = BC (opp. sides of a parallelogram are equal) AB = CD = AD = BC ( using (2)) Hence, quadrilateral ABCD is a rhombus. Example 6. Show that the diagonals of a square are equal and bisect each other at right angles. NCERT Solution. Given : A square ABCD. To prove : AC = BD, AC BD and OA = OC, OB = OD. Proof : Since ABCD is square, AB || DC and AD || BC. Now, AB || DC and transversal AC intersects them at A and C respectively. BAC = DCA ( alternate interior angles are equal) BAO = DCO ...(1) Again AB || DC and BD intersects them at B and D respectively. ABD = CDB ( alternate interior angles are equal) ABO = CDO ...(2) Now, In AOB and COD, we have BAO = DCO (from (1)) AB = CD (opp. sides of a parallelogram are equal) ABO = CDO (from (2)) AOB COD (ASA congruence condition) OA = OC and OB = OD (cpct) Hence, the diagonals bisect each other. Again, In ADB and BCA, we have AD = BC (sides of a square are equal) BAD = ABC (each 90) AB = AB (common side) ADB BCA (SAS congruence condition) AC = BD (cpct) Hence, diagonals are equal. Now, In AOB and AOD, we have, OB = OD (diagonals of a parallelogram bisect each other) AB = AD ( sides of a square are equal) AO = AO (common side) AOB AOD (SSS congruence condition) AOB AOD (cpct) 86 QUADRILATERALS MATHEMATICSIX

but, AOB AOD = 180 AOB AOD =


180 = 90 2

AO BD AC BD. Hence, diagonals intersect at right angles. Hence proved. Example 7. Show that if the diagonals of a quadrilateral one equal and bisect each other at righ angles, then it is a square. NCERT Solution. Given : A quadrilateral ABCD in which the diagonals AC = BD, AO = OC, BO = OD and AC BD. To prove : Quadrilateral ABCD is a square. Proof : In AOD and COB, we have AO = OC (given) OD = OB (given) AOD = COB (vertically opp. angles) AOD COB (SAS congruence condition) OAD = OCB ...(1) (cpct) Now, line AC intersects AD and BC at A and C respectively such that OAD = OCB, i.e. alternate interior angles are equal. AD || BC Similarly, AB || CD. Hence, ABCD is a parallelogram. Now, In AOB and AOD, we have AO = AO (common side) AOB = AOD (each = 90) OB = OD ( diagonals of a parallelogram bisect each other) AOB AOD (SAS congruence condition) AB = AD (cpct) But, AB = CD and AD = BC (opposite sides of a parallelogram are equal) AB = BC = CD = AD ...(2) Now, In ABD and BAC, we have AB = AB (common side) AD = BC (opp. sides of parallelogram are equal) BD = AC (given) ABD BAC (SSS congruence condition) DAB = CBA (cpct) But, DAB + CBA = 180 DAB CBA =
180 = 90 2

Thus, ABCD is a parallelogram whose all the sides are equal and one of the angle is 90. ABCD is a square. Hence proved. MATHEMATICSIX QUADRILATERALS 87

Example 8. Prove that bisectors of a parallelogram form a rectangle. Solution. Given : A paralleogram ABCD in which AR, BR, CP and DP are the bisectors of A, B, C and D respectively forming quadrilateral PQRS.

To prove : PQRS is a rectangle. Proof : DCB + ABC = 180 (co-interior angles of parallelogram are supplementary)
1 1 DCB ABC 90 2 2

1 2 90 ...(1) Also, In CQB, 1 + 2 + CQB = 180 ...(2) from (1) and (2), we get CQB = 180 90 = 90 RQP = 90 ( CQB = RQP, vertically opp. angles) Similarly, it can be shown, QRP = RSP = SPQ = 90 So, Quadrilateral PQRS is a rectangle. Example 9. In the given figure, ABCD is a parallelogram in which P and Q are the mid points of AB and CD respectively. If AQ and DP intersects at S and PC and BQ intersects at R, show that the quadrilateral PRQS is a parallelogram.

Solution.

AB || DC AP || QC Also, AB = DC
1 1 AB DC AP QC 2 2 AP || QC and AP = QC APCQ is a parallelogram. So, AQ || PC or SQ || PR. Similarly, it can be easily shown that PS || QR. Thus, In quadrilateral PRQS, SQ || PR and PS || RQ. So, PRQS is a parallelogram.

88

QUADRILATERALS

MATHEMATICSIX

Example 10. ABCD is a parallelogram and line segments AP and CQ bisects the A and C respectively. Show that AP || CQ. Solution.

here, A = C ( opp. angles of a parallelogram are equal)


1 1 A C 2 2 1 = 2 ...(1) Now, AB || DC and CQ is a transversal 2 = 3 ...(2) From (1) and (2), 1 = 3 Thus, transversal AB intersects AP and CQ at A and Q, such that 1 = 3 i.e. corresponding angles are equal. Hence, AP || CQ. Example 11. Diagonals AC of a parallelogram ABCD bisects A (see figure). Show that : (i) it bisects C also. (ii) ABCD is a rhombus.

Solution.

(i) Given : A parallelogram ABCD, in which diagonal AC bisects A. To prove : (i) AC bisects C i.e. 3 = 4 (ii) ABCD is a rhombus Proof : Since ABCD is a parallelogram, AB || DC. Now, AB || DC and AC intersects them 1 = 3 ...(1) ( alternate interior angles) Again, AD || BC and AC intersects them. 2 = 4 ...(2) ( alternate interior angles) but, 1 = 2 ...(3) (given) from (1), (2) and (3), we get 3 = 4 Hence, AC bisects C. (ii) To prove : ABCD is a rhombus. from (i) part, We have 1 = 2 = 3 = 4 Now, in ABC, 1 = 4 AB = BC (sides opp. to equal angles in a triangle are equal) Similarly, In ADC, we have 2 = 3 AD = DC. Also, ABCD is a parallelogram. AB = CD and AD = BC combining these, we get AB = BC = CD = DA. Hence, ABCD is a rhombus. QUADRILATERALS 89

MATHEMATICSIX

Example 12. ABCD is a rhombus. Show that diagonal AC bisects A as well as C and diagonal BD bisects B as well as D. NCERT Solution. Given : A rhombus ABCD To prove : (i) Diagonal AC bisects A as well as C. (ii) Diagonal BD bisects B as well as D. Proof : ADC, AD = DC (sides of a rhombus are equal) DAC = DCA ...(1) (angles opp. to equal sides of a triangle are equal) Now, AB || DC and AC intersects them BCA = DAC ...(2) (alternate angles) from (1) and (2), we get DCA = BCA ...(3) AC bisects C. In ABC, AB = BC (sides of a rhombus are equal) from (3) and (4), we get BAC = DAC AC bisects A. Hence, diagonal AC bisects A as well as C. Similarly, diagonal BD bisects B as well as D. Example 13. In parallelogram ABCD, two points P and Q are taken on diagonal BD such that DP = BQ (see figure). Show that : (i) APD CQB (ii) AP = CQ (iii) AQB CPD (iv) AQ = CP (v) APCQ is a parallelogram. NCERT Solution. Given : ABCD is a parallelogram P and Q are points on the diagonal BD such that DP = BQ. To prove : (i) APD CQB (ii) AP = CQ (iii) AQB CPD (iv) AQ = CP (v) APCQ is a parallelogram. Construction : Join A to C to meet BD in O. Proof : We know that the diagonals of parallelogram bisect each other. Now, AC and BD bisect each other at O. OB = OD But BQ = DP (given) OB BQ = OD DP OQ = OP Thus, in quadrilateral APCQ, diagonals AC and PQ are such that OQ = OP and OA = OC i.e., the diagonals AC and PQ bisects each other. Hence, APCQ is a parallelogram, which proves (v) part. (i) Now, In APD and CQB, we have 90 QUADRILATERALS MATHEMATICSIX

AD = CB (opp. sides of a a parallelogram ABCD) AP = CQ (opp. sides of a a parallelogram APCQ) DP = BQ (given) APD CQB (SSS congruence condition) (ii) AP = CQ (cpct) (iii) In AQB and CPD, we have AB = CD (opp. sides of a a parallelogram ABCD) AQ = CP (opp. sides of a a parallelogram APCQ) BQ = DP (given) AQB CPD (SSS congruence condition) (iv) Since, AQB CPD AQ = CP (cpct) Example 14. ABCD is a parallelogram and AP and CQ are perpendiculars from vertices A and C on diagonal BD respectively. (see figure). Show that : (i) APB CQD (ii) AP = CQ NCERT

Solution.

(i) Since, ABCD is a parallelogram, DC || AB. Now, DC || AB and transversal BD intersects them at B and D respectively. ABD = BDC (alt. int. angles) Now, In APB and CQD, we have ABP = QDC ( ABD = BDC) APB = CQD (each = 90) AB = CD (opp. sides of a parallelogram) APB CQD (AAS congruence condition) (ii) Since, APB CQD AP = CQ (cpct)

Example 15. ABCD is a trapezium in which AB || CD and AD = BC (see figure). Show that : (i) A = B (ii) C = D (iii) ABC BAD (iv) diagonal AC = diagonal BD Solution. NCERT Given : ABCD is a trapezium, in which AB || CD and AD = BC. To prove : (i) A = B (ii) C = D (iii) ABC BAD (iv) diagonal AC = diagonal BD Construction : Produce AB and draw a line CE || AD. Proof : (i) Since AD || CE and transversal AE cuts them at A and E respectively. A + E = 180 ...(1) QUADRILATERALS 91

MATHEMATICSIX

Since AB || CD and AD || CE, AECD is a parallelogram. AD = CE BC = CE (AD = BC (given)) Thus, In BCE, we have BC = CE CBE = CEB 180 B = E B + E = 180 ...(2) from (1) and (2), we get, A + E = B + E A = B (ii) Since A = B BAD = ABD 180 BAD = 180 ABD ADB = BCD D = C i.e. C = D (iii) In ABC and BAD, we have BC = AD (given) AB = AB (common) A = B (proved above) ABC BAD (SAS Congruence condition) (iv) Since, ABC BAD AC = BD (cpct) Hence proved. Example 16. Show that the quadrilateral formed by joining the mid-points of the adjacent sides of a quadrilateral is a parallelogram. Solution. Given : A quadrilateral ABCD, in whcih P, Q, R, S are the mid-points of AB, BC, CD and DA respectively. To prove : Quadrilteral PQRS is a parallelogram. Construction : Join A to C. Proof : In ABC, P and Q are mid-points of AB and BC respectively.
1 AC ( mid-point theorem) 2 Again, In DAC, R and S are mid-points of sides CD and AD respectively.

PQ || AC and PQ

SR || AC and SR

1 AC [ mid-point theorem] 2 Now, PQ || AC and SR || AC PQ || SR

1 AC SR PQ SR 2 PQ || SR and PQ = SR Hence, PQRS is a parallelogram.

Again, PQ

92

QUADRILATERALS

MATHEMATICSIX

Example 17. ABCD is a rhombus and P, Q, R and S are respectively the mid-points of the sides AB, BC, CD and DA respectively. Show that the quadrilateral PQRS is a rectangle. NCERT Solution. Given : ABCD is a rhombus in which P, Q, R and S are the mid-points of AB, BC, CD and DA respectively. PQ, QR, RS and SP are joined to obtain a quadrilateral PQRS. To prove : PQRS is a rectangle. Construction : Join A to C. Proof : In ABC, P and Q are the mid-points of AB and BC respectively. PQ || AC and PQ =
1 AC ...(1) 2 ( mid-point theorem) Similarly, In ADC, R and S are the mid-points of CD and DA respectively.

SR || AC and SR =

1 AC 2 ( mid-point theorem) from (1) and (2), we get PQ || SR and PQ = SR PQRS is a parallelogram. Now, AB = BC

...(2)

(sides of rhombus are equal)

1 1 AB = BC PB = BQ 3 = 4 ( angles opp. to equal sides of a triangle are equal) 2 2 Now, In APS and CQR, we have AP = CQ (half of equal sides AB and BC) AS = CR (half of equal sides AD and CD) PS = QR (opp. sides of parallelogram PQRS) APS CQR (SSS congruence condition) 1 = 2 (cpct) Now, 1 + SPQ + 3 = 180 ( linear pair) 1 + SPQ + 3 = 2 + PQR + 4 but, 1 = 2 and 3 = 4 (proved above) SPQ = PQR ...(3) Now, SP || RQ and PQ intersects them, SPQ + PQR = 180 ...(4) from (3) and (4), we get 2SPQ = 180 SPQ = 90 Thus, PQRS is a parallelogram whose one angle is 90. Hence, PQRS is a rectangle. Example 18. ABCD is a rectangle and P, Q, R, S are mid-points of the sides AB, BC, CD and DA respectively. Show that the quadrilateral PQRS is a rhombus. NCERT

MATHEMATICSIX

QUADRILATERALS

93

Solution.

Given : ABCD is a rectangle, in which P, Q, R and S are the mid-points of sides AB, BC, CD and DA respectively. PQ, QR, RS and SP are joined to obtain a quadrilateral PQRS. To prove : PQRS is a rhombus. Construction : Join A to C. Proof : In ABC, P and Q are the mid-points of sides AB and BC.
1 AC ...(1) ( mid-point theorem) 2 Similarly, In ACD, R and S are the mid-points of sides CD and DA.

PQ || AC and PQ =

1 AC ...(2) 2 from (1) and (2), we get PQ || SR and PQ = SR PQRS is a parallelogram. Now, AD = BC (opp. sides of a rectangle ABCD)

RS || AC and RS =

( mid-point theorem)

1 1 AD BC AS BQ 2 2 Now, In APS and BPQ, we have AP = BP ( P is the mid-point of AB) PAS = PBQ (each = 90) AS = BQ (proved above) APS BPQ (SAS congruence condition) PS = PQ (cpct) Thus, PQRS is a parallelogram in which adjacent sides are equal. PQRS is a rhombus. Example 19. ABCD is a trapezium in which AB || DC, BD is a diagonal and E is the mid-point of AD. A line is drawn through E parallel to AB intersecting BC at F (see figure). Show that F is the mid-point of BC. NCERT Solution. Given : A trapezium ABCD, in which AD || BC. E is the mid-point of AD and EF || AB.

To prove : F is the mid-point of BC. Construction : Join B to D. Let it intersect EF in G. Proof : In DAB, E is the mid-point of AD (given) EG || AB ( EF || AB) By converse of mid-point theorem, G is the mid-point of DB. Now, In BCD, G is the mid-point of BD ( proved above) GF || DC ( AB || DC, EF || AB DC || EF) By converse of mid-point theorem, F is the mid point of BC. 94 QUADRILATERALS MATHEMATICSIX

Example 20. ABC is a triangle right angled at C. A line through the mid-point M of hypotenuse AB and parallel to BC intersects AC at D. Show that : (i) D is the mid-point of AC Solution. (ii) MD AC (iii) CM MA
1 AB 2

NCERT Given : ABC is right angled at C, M is the mid-point of hyp. AB . MD || BC. To prove : (i) D is the mid-point of AC (ii) MD AC
1 AB 2 Proof : (i) In ABC, M is the mid-point of AB and MD || BC. D is the mid-point of AC i.e. AD = DC ...(1) (ii) Since MD || BC ADM = ACB (corresponding angles) ADM = 90 ( ACB = 90, MD || BC) But, ADM + CDM = 180 ( linear pair) 90 + CDM = 180 CDM = 90 Thus, ADM = CDM = 90 ...(2) AC. MD (iii) Now, In AMD and CMD, we have AD = CD (from (1)) ADM = CDM (from (2)) MD = MD (common side) AMD CMD (SAS congruence condition) MA MC (cpct)

(iii) CM MA

Also, Hence,

MA

1 AB, 2 1 AB. 2

Since M is the mid-point of AC.

CM = MA

PRACTICE EXERCISE
1. In the given figure, PQRS is a rectangle whose diagonals PR and QS intersect at O. If OPQ = 32, find OQR. (Ans. : 58)

2. Prove that, if in a parallelogram diagonals are equal and perpendicular to each other, then it is a squqre. 3. In the given figure, PQRS is a parallelogram and X and Y are the points on the diagonal QS such that SX = QY. Prove that PYRS is a parallelogram. MATHEMATICSIX QUADRILATERALS 95

4. Let ABC and DEF be two triangles drawn in such a way that AB || DE ; AB = DE; BC || EF and BC = EF. Show that AC || DF and AC = DF.

5. In a parallelogram ABCD, the bisectors of consecutive angles A and B intersect at O. Prove that AOB = 90. 6. In the given figure, ABCD is a parallelogram and P is the mid-point of AD. A line through D, drawn parallel to PB, meets AB produced at Q and BC at R, prove that : (i) AQ = 2DC (ii) DQ = 2DR

7. In a parallelogram ABCD, if A = (4x + 20) and B = (3x 15), find the value of x and the measure of ecah angle of a parallelogram. (Ans. x = 25 and angles are 60, 120, 60, 120) 8. A ABC is given. If lines are drawn through A, B, C parallel respectively to the sides BC, CA and AB forming XYZ, as shown. Show that BC
1 YZ. 2

9. In a parallelogram ABCD, the bisector of A also bisects the side BC at E. Show that AD = 2AB. 10. If a transversal cuts two parallel lines, prove that the bisectors of the interior angles form a rectangle. 96 QUADRILATERALS MATHEMATICSIX

11. ABC is a triangle right angled at B and D is the mid-point of AC. DE is drawn perpendicular to BC. Prove that BD
1 AC. 2

12. If P, Q and R are respectively the mid-points of sides BC, CA and AB of an equilateral triangle ABC, prove that PQR is also equilateral triangle. 13. ABCD is a parallelogram in which E and F are mid-points of the sides AB and CD respectively. Prove that the line segments CE and AF trisect the diagonal BD. 14. Let ABCD be a trapezium in which AB || DC and let E be the midpoint of AD. Let F be a point on BC such that EF || AB. Prove that: (i) F is the mid-point of BC
1 (AB DC) 2 Prove that the line segment joining the mid-points of the diagonals of a trapezium is parallel to the parallel sides and is equal to half their difference. Show that the quadrilateral formed by joining the midpoints of pairs of adjacent sides of a rhombus is a rectangle. Show that the quadrilateral formed by joining the mid-points of the pairs of adjacent sides of a rectangle is a rhombus. Show that the line segments joining the mid-points of opposite sides of a quadrilateral bisect each other. Show that the four triangles formed by joining the mid-points of the three sides of a triangle are congruent to each other. In ABC, AD is the median, and E is the mid-point of AD . BE produced meets AC in F. Prove that

(ii) EF

15. 16. 17. 18. 19. 20.

AF

1 AC . 3

21. The diagonals of a quadrilateral are perpendicular. Prove that the quadrilateral, formed by joining the mid-points of its sides is a rectangle. 22. D, E and F are respectively the mid-points of the sides BC, AC and AB of an isosceles triangle ABC, in which AB = AC. Prove that AD is perpendicular to EF and is bisected by it. 23. In the given figure, ABC is a triangle and through A, B, C lines are drawn parallel to BC, CA and AB respectively intersecting at P, Q and R. Prove that perimeter of PQR is double the perimeter of ABC.

MATHEMATICSIX

QUADRILATERALS

97

24. ABC is any triangle. D is a point of AB such that AD


AE

1 AB and E is a point on AC such that 4

1 1 AC. Prove that DE BC. 4 4 25. BX and CY are perpendiculars to a line passing through the vertex A of a triangle ABC. If Z is the midpoint of BC. Prove that XZ = YZ.

PRACTICE TEST
MM : 30 General Instructions : Q. 1-4 carry 2 marks, Q. 5-8 carry 3 marks and Q. 9-10 carry 5 marks each. 1. In parallelogram ABCD, AB = 10 cm and BC = 8 cm. If B = 120, find : (i) DA (ii) C (iii) D 2. ABCD is a parallelogram and AB is produced to X so that AB = BX. Prove that DX and BC bisect each other. 3. Two adjacent angles of a parallelogram are (2x + 15) and (3x 25). Find the value of x and also the measure of the angles. 4. ABCD is a trapezium, in which AB || DC. X and Y are respectively mid-points of AD and BC. If AB = 13 cm and CD = 9 cm, find XY. 5. In a parallelogram, show that the angle bisectors of two adjacent angles intersect at right angle. 6. ABCD is a parallelogram whose diagonals intersect each other at O. Through O, XY is drawn as shown. Prove that OX = OY. Time : 1 hour

7. PQRS is a parallelogram. PO and QO are respectively the angle bisectors to P and Q. Line AOB is drawn parallel to PQ. Prove that : (i) PA = QB (ii) AO = OB

98

QUADRILATERALS

MATHEMATICSIX

8. Prove that in a right angled triangle, the median bisecting the hypotenuse is half of the hypotenuse. 9. ABC is a triangle right angled at C, A line through the mid-point M of hypotenuse AB and parallel to BC intersects AC at D. Show that : (i) D is the mid-point of AC. (ii) MD AC.
1 AB. 2 10. Prove that the quadrilateral formed by joining mid-points of the sides of a square is again a square.

(iii) CM MA

ANSWERS OF PRACTICE EXERCISE


1. (i) 8 cm (ii) 60 (iii) 120 3. x = 38, angles are 89, 91, 89, 91 4. 11 sq. units

MATHEMATICSIX

QUADRILATERALS

99

CHAPTER

AREAS OF PARALLELOGRAMS AND TRIANGLES


Points to Remember : 1. Two congruent figures must have equal areas. However, two figures having equal areas need not to be congruent. 2. Two figures are said to be on the same base and between the same parallels, if they have a common base and the vertices (or the vertex) opposite to the common base of each figure lie on a line parallel to the base. 3. Parallelograms on the same base and between the same parallels are equal in area. 4. Area of parallelogram = Base corresponding height. 5. Parallelograms on the same base (or equal bases) and having equal areas lie between the same parallels. 6. Two triangles on the same base (or equal bases) and between the same parallels are equal in area. 7. Two triangles having the same base (or equal bases) and equal areas lie betwen the same parallels. 8. Area of Triangle
1 Base corresponding height. 2

9. Area of a Rhombus

1 product of diagonals. 2

1 (sum of the parallel sides) (distance between them). 2 11. A median of a triangle divides it into two triangles of equal area. 12. The diagonals of a parallelogram divides it into four triangles of equal area.

10. Area of a Trapezium

ILLUSTRATIVE EXAMPLES
Example 1. In the given figure, ABCD is a parallelogram, AB = 12 cm, altitude DF = 7.2 cm and DE = 6 cm, find the perimeter of parallelogram ABCD. Solution. Area of parallelogram ABCD = AB DE ...(1) also, Area of parallelogram ABCD = BC DF ...(2) From (1) and (2), we get AB DE = BC DF 12 6 = BC 7.2
12 6 10 cm 7.2 Perimeter of parallelogram ABCD = 2 (AB + BC) = 2 (12 + 10) cm = 2 (22) cm = 44 cm Ans. BC

Example 2. P, Q, R, S are respectively, the mid-points of sides AB, BC, CD and DA of parallelogram ABCD. Show that the quadrilateral PQRS is a parallelogram and its area is half the area of the parallelogram ABCD. 100 AREAS OF PARALLELOGRAMS AND TRIANGLES MATHEMATICSIX

Solution.

Join A to C and Q to S

Now, In ABC, P and Q are respectively mid-points of AB and BC. PQ || AC and PQ


1 AC 2 1 AC 2

...(1) ( mid-point theorem) ...(2)

Similarly, SR || AC and SR

From (1) and (2), we get PQ || SR and PQ = SR So, PQRS is a parallelogram. Now, SPQ and parallelogram ABQS stand on same base SQ and between same parallels AB and QS, ar (PQS)
1 ar (parallelogram ABQS) 2

...(3)

1 ...(4) ar (parallelogram SQCD) 2 adding (3) and (4), we get, 1 ar (PQS) + ar (SRQ) [ar (parallelogram ABQS) ar (parallelogram SQCD)] 2 1 ar (parallelogram PQRS) ar (parallelogram ABCD) 2 which proves the result. Example 3. P and Q are any two points lying on the sides DC and AD respectively of a parallelogram ABCD. Show that ar (APB) = ar (BQC). NCERT Solution. APB and parallelogram ABCD stand on the same base AB and lie between the same parallels AB and DC.

Similarly, ar (SRQ)

1 ar(ABCD) ...(1) 2 Similarly, BQC and parallelogram stand on the same base BC and lie between the same parallels BC and AD.

ar (APB) =

1 ar (ABCD) ...(2) 2 from (1) and (2), we get ar (APB) = ar (BQC). Hence proved. Example 4. A farmer was having a field in the form of a parallelogram PQRS. She took any point A on RS and joined it to points P and Q. In how many parts the fields is divided? What are the shapes of these parts? The farmer wants to sow wheat and pulses in equal portions of the field seperately. How should she do it? (NCERT)

ar (BQC)

MATHEMATICSIX AREAS OF PARALLELOGRAMS AND TRIANGLES

101

Solution.

Clearly, the field parallelogram PQRS is divided into 3 parts. Each part is in shape of a triangle. Since APQ and parallelogram PQRS stand on the same base PQ and lie between the same parallels PQ and SR.
1 . ar (PQRS) ...(1) 2 Clearly, ar (APS) + ar (AQR) = ar (PQRS) ar (APQ)

ar (APQ) =

= ar (PQRS) =

1 ar (PQRS) ( using (1)) 2

1 ar (PQRS) ...(2) 2 From (1) and (2), we get ar (APS) + ar (AQR) = ar (APQ) Thus, the farmer should sow wheat and pulses either as [(APS and AQR) or APQ] or as [APQ or (APS and AQR)]. Example 5. Show that a median of a triangle divides it into two triangles of equal area. NCERT Solution. Let ABC be a given triangle and AD is a median. Draw AE BC. Since, D is mid point of BC, so we have, BD = DC ...(1)

Now, ar (ABD) and, ar (ADC)

1 BD AE 2

...(2)

1 CD AE 2

1 BD AE ...(3) 2 ( BD = DC) From (2) and (3), we get ar (ABD) = ar (ADC) which proves the desired result. Example 6. E is any point on median AD of a ABC. Show that ar (ABE) = ar (ACE) Solution. Given : AD is a median of ABC and E is any point on AD. To prove : ar (ABE) = ar (ACE) Proof : AD is the median of ABC ar (ABD) = ar (ACD) ...(1) also, ED is the median of EBC, ar (BED) = ar (CED) ...(2) Subtracting (2) from (1), we get ar (ABD) ar (BED) = ar (ACD) ar (CED) ar (ABE) = ar (ACE). Hence shown.

NCERT

Example 7. In a ABC, E is the mid-point of median AD. Show that ar (BED) = Solution. Given : ABC in which E is the mid-point of median AD. To prove : ar (BED) 102
1 . ar (ABC) 4

1 ar (ABC). 4

NCERT T

AREAS OF PARALLELOGRAMS AND TRIANGLES MATHEMATICSIX

Proof : Since AD is a median of ABC and median divides a triangle into two triangles of equal area. ar (ABD) = ar (ADC) ar (ABD) =
1 ar (ABC) 2

...(1)

In ABD, BE is a median, ar (BED) = ar (BAE)


1 ar (ABD) 2 1 1 ar (ABC) 2 2
1 ar (ABC) 4

ar (BED) =

( using (1))

= Hence shown.

Example 8. If the medians of a ABC intersect at G, show that ar (AGB) = ar (AGC) = ar (BGC) 1 ar (ABC) 3 We know that a median of a triangle divides it into two triangles of equal areas. In ABC, AD is the median. ar (ABD) = ar (ACD) ...(1) ...(2) Again, In GBC, ar (GBD) = ar (GCD) Subtracting (2) from (1), ar (ABD) ar (GBD) = ar (ACD) ar (GCD) ar (ABG) = ar (AGC) Similarly, ar (AGB) = ar (BGC) From (3) and (4), we get, ar (ABG) = ar (AGC) = ar (BGC) Now, ar (ABC) = ar (ABG) + ar (AGC) + ar (BGC) = 3 ar (ABG) , ar (ABC)
1 ar (ABC) 3

Solution.

...(3) ...(4)

Hence, ar(AGB) = ar (AGC) = ar (BGC) 1 ar ( ABC) 3 Example 9. PQRS and ABRS are parallelograms and X is any point on the side BR. Show that : (i) ar (parallelogram PQRS) = ar (parallelogram ABRS) (ii) ar (AXS) =
1 ar (parallelogram PQRS) 2

MATHEMATICSIX AREAS OF PARALLELOGRAMS AND TRIANGLES

103

Solution.

Since, parallelogram PQRS and ABRS stand on same base SR and between same parallels SR and PB, ar (parallelogram PQRS) = ar (parallelogram ABRS)
1 ar (parallelogram ABRS) 2 ( ASX and parallelogram ABRS stand on same base AS and between parallel lines AS and BR) But, ar (parallelogram ABRS) = ar (parallelogram PQRS)

Again, ar (ASX) =

ar (ASX)

1 ar (parallelogram PQRS) 2

Hence proved. Example 10. In ABC, D, E and F are mid-points of the sides BC, CA and AB respectively. Prove that : (i) BDEF is a parallelogram 1 (ii) ar (DEF) ar (ABC) 4 1 (iii) ar (BDEF) ar (ABC). 2 Solution. (i) FE || BC or FE || BD and DE || BF ( mid point theorem) BDEF is a parallelogram. (ii) Since a diagonal of a parallelogram divides it into two triangles equal in area. ar (BDF) = ar (DEF) Similarly, ar (DCE) = ar (DEF)

and ar (DEF) = ar (AFE) Combining all these, we get ar (BDF) = ar (DEF) = ar (DCE) = ar (AFE) But, ar (ABC) = ar (BDF) + ar (DEF) + ar (DCE) + ar (AFE) = 4 ar (DEF)
1 . ar (ABC) 4 (iii) Again, 4ar (DEF) = ar (ABC) 1 2 . ar (DEF) = ar (ABC) 2 1 ar (BDEF) = . ar (ABC) 2 Hence proved.

ar (DEF)

104

AREAS OF PARALLELOGRAMS AND TRIANGLES MATHEMATICSIX

Example 11. In the given figure, ABC and ABD are two triangles on the same base AB. If line segment CD is bisected by AB at O, show that ar (ABC) = ar (ABD). NCERT

Solution.

Given : ABC and ABD are two triangles on the same base AB. A line segment CD is bisected by AB at O. i.e. OC = OD. To prove : ar (ABC) = ar (ABD) Proof : In ACD, we have OC = OD (given) AO is a median. ar (AOC) = ar (AOD) ...(1) ( median divides a triangle in two triangles of equal areas). Similarly, In BCD, BO is the median. ar (BOC) = ar (BOD) ...(2) adding (1) and (2), we get ar (AOC) + ar (BOC) = ar (AOD) + ar (BOD) ar (ABC) = ar (ABD). Example 12. In the given figure, diagonals AC and BD of a quadrilateral ABCD intersect at O such that OB = OD. If AB = CD, then show that : (i) ar (DOC) = ar (AOB) (ii) ar (DCB) = ar (ACB) (iii) DA || CB or ABCD is a parallelogram. NCERT Solution. (i) Draw DN AC and BM AC. In DON and BOM DNO = BMO (each = 90) DON = BOM (vert. opp. angles) OD = OB (given) DON BOM (AAS congruence condition) ar (DON) = ar (BOM) ...(1) ( congruent triangles have equal area) Again, In DCN and BAM DNC = BMA (each = 90) DC = AB (given) DN = BM (DON BOM DN = BM) DCN BAM (RHS congruence condition) ar (DCN) = ar (BAM) ...(2) ( congruent triangles have equal areas) adding (1) and (2), we get ar (DON) + ar (DCN) = ar (BOM) + ar (BAM) ar (DOC = ar (AOB) (ii) Since, ar (DOC) = ar (AOB) ar (DOC) + ar (BOC) = ar (AOB) + ar (BOC) 105

MATHEMATICSIX AREAS OF PARALLELOGRAMS AND TRIANGLES

ar (DCB) = ar (ACB) (iii) DCB and ACB have equal areas and have the same base. So, these triangles lie between the same parallels. DA || CB. i.e. ABCD is a parallelogram. Example 13. A point O inside a rectangle ABCD is joined to the vertices. Prove that : ar (AOD) + ar (BOC) = ar (AOB) + as (COD) Solution. Draw POQ || AD and ROS || AB. Since, POQ || AD and DC cuts them. ADC = PQC = 90 ( corr. angles) i.e. OQ CD. Similarly, OR AD, OP AB and OS BC. Consider, ar (AOD) + ar (BOC)
1 1 AD OR BC OS 2 2 1 AD (OR OS) 2

( AD BC)

1 1 AD RS AD AB 2 2

1 ar (rect. ABCD) 2 Again, ar (AOB) + ar (COD) 1 1 AB OP CD OQ 2 2 1 AB (OP OQ) 2 1 1 AB PQ AB AD 2 2

...(1)

( AB = CD)

1 ar (rect. ABCD) ...(2) 2 From (1) and (2), we get ar (AOD) + ar (BOC) = ar (AOB) + ar (COD) Example 14. The side AB of a parallelogram ABCD is produced to any point P. A line through A and parallel to CP meets CB produced at Q and then parallelogram PBQR is completed (see figure). Show that ar (ABCD) = ar (PBQR).

Solution.

Join A to C and P and Q. Since AC and PQ are diagonals of parallelogram ABCD and parallelogram BPQR respectively. AREAS OF PARALLELOGRAMS AND TRIANGLES MATHEMATICSIX

106

ar (ABC) = and, ar (PBQ) =

1 ar (ABCD) 2

...(1)

1 ar (BPRQ) ...(2) 2 Now, ACQ and AQP are in the same base AQ and CP. ar (ACQ) = ar (AQP) ar (ACQ) ar (ABQ) = ar (AQP) ar (ABQ) ( Subtracting ar (ABQ) from both sides) ar (ABC) = ar (BPQ) 1 1 ar (ABCD) ar(BPRQ) ( using (1) and (2)) 2 2 ar (ABCD) = ar (BPRQ) Hence proved. Example 15. In the given figure, PQ is a line parallel to side BC of ABC. If BX || CA and CY || BA meet the line PQ produced in X and Y respectively, show that : ar (ABX) = ar (ACY).

Solution.

Parallelogram XBCQ and ABX stand on the same base BX and between the same parallels BX and CA, we have
1 ar (parallelogram XBCQ) ...(1) 2 Also, parallelogram BCYP and ACY stand on same base CY and between the same parallels CY and BA, we have

ar (ABX)

1 ar (parallelogram BCYP) ...(2) 2 But, parallelogram XBCQ and parallelogram BCYP stand on same base BC and between same parallels BC and XY, ar (parallelogram XBCQ) = ar (parallelogram BCYP) ...(3) from (1), (2) and (3), we get, ar (ABX) = ar (ACY) Hence proved. Example 16. In the given figure, ABCDE is a pentagon. A line through B parallel to AC meets DC produced at F. Show that (i) ar (ACB) = ar (ACF) (ii) ar (AEDF) = ar (ABCDE) NCERT Solution. (i) Since ACD and ACF are on the same base AC and betweeen the same parallels AC and BF, ar (ACB) = ar (ACF) (ii) Now, ar (ACB) = ar (ACF) adding ar (ACDE) both sides, we get

ar (ACY)

MATHEMATICSIX AREAS OF PARALLELOGRAMS AND TRIANGLES

107

ar (ACB) + ar (ACDE) = ar (ACF) + ar (ACDE) ar (AEDF) = ar (ABCDE) Hence proved. Example 17. Diagonals AC and BD of a quadrilateral ABCD intersect at O in such a way that ar (AOD) = ar (BOC). Prove that ABCD is a trapezium. NCERT Solution. Diagonals AC and BD of a quadrilateral ABCD intersect at O in such a way that, ar (AOD) = ar (BOC) ...(1) adding ar (ODC) on both sides, we get ar (AOD) + ar (ODC) = ar (BOC) + ar (ODC) ar (ADC) = ar (BDC)
1 1 DC AL DC BM 2 2 AL = BM AB || DC. (distance between two parallel lines is same) Hence ABCD is a trapezium. Example 18. In the given figure, ar (DRC) = ar (DPC) and ar (BDP) = ar (ARC). Show that both the quadrilaterals ABCD and DCPR are trapeziums. Solution. ar (BDP) = ar (ARC) ...(1) (given) and ar (DPC) = ar (DRC) ...(2) (given) subtracting (2) from (1), we get ar (BDP) ar (DPC) = ar (ARC) ar (DRC) ar (BDC) = ar (ADC) DC || AB Hence, ABCD is a trapezium. ar (DRC) = ar (DPC) (given) on subtracting ar (DLC) from both sides, we get ar (DRC) ar (DLC) = ar (DPC) ar (DLC) ar (DLR) = ar (CLP) on adding ar (RLP) to both sides, we get ar (DLR) + ar (RLP) = ar (CLP) + ar (RLP) ar (DRP) = ar (CRP) RP || DC Hence, DCPR is a trapezium.

Example 19. Let P, Q, R and S be respectively the mid-points of the sides AB, BC, CD and DA of quadrilateral ABCD. Show that PQRS is a parallelogram such that ar (parallelogram PQRS) Solution. ABCD). Join A to C and A to R. In ABC, P and Q are mid-points of AB and BC respectively.
1 AC. 2 In DAC, S and R are mid-points of AD and DC respectively. 1 ar (quad. 2

PQ || AC and PQ

SR || AC and SR

1 AC. 2 Thus, PQ || SR and PQ = SR.

108

AREAS OF PARALLELOGRAMS AND TRIANGLES MATHEMATICSIX

PQRS is a parallelogram. Now, median AR divides ACD into the triangles of equal area. ar (ARD) =
1 . ar (ACD) 2 median RS divides ARD into two triangles of equal area.

...(1)

ar (DSR) =

1 ar (ARD) 2

...(2)
1 1 1 1 ar (ARD) = ( ar (ACD)) = ar (ACD) 2 2 4 2

from (1) and (2), we get, ar (DSR) = Similarly, ar (BQP) =


1 ar (ABC). 4

ar (DSR) + ar (BQP) =

1 [ar (ACD) + ar (ABC)] 4 1 ar (quad. ABCD) 4 1 ar (quad. ABCD) 4

ar (DSR) + ar (BQP) =

...(3) ...(4)

Similarly, ar (CRQ) + ar (ASP) = Adding (3) and (4), we get

1 ar (quad. ABCD) 2 But, ar (DSR) + ar (BQP) + ar (CRQ) + ar (ASP) + ar (parallelogram ABCD) = ar (quad ABCD) Subtracting (5) from (6), we get

ar (DSR) + ar (BQP) + ar (CRQ) + ar (ASP) =

...(5) ...(6)

1 ar (quad. ABCD). Hence proved. 2 Example 20. Prove that, of all the parallelograms of given sides, the parallelogram which is a rectangle has the greatest area. Solution. In parallelogram ABCD of sides a and b, let h be the height corresponding to the base a.

ar (|| PQRS) =

Now, In DAE, h<b ( b, being the hypotenuse, is the longest side of the triangle) multiplying both sides by a, we get ah < ab i.e. ar (parallelogram ABCD) < ar (rect ABCD), which proves the result. MATHEMATICSIX AREAS OF PARALLELOGRAMS AND TRIANGLES 109

PRACTICE EXERCISE
1. PQRS is a parallelogram in which SX PQ and QY PS. If PS = 18 cm, SX =1 2 cm and QY = 8 cm, find the value of PQ. (Ans. PQ = 12 cm) 2. Show that the line segment joining the mid-points of a pair of opposite sides of a parallelogram divides it into two equal parallelograms. 3. In the given figure, O is a point in the interior of parallelogram ABCD. Show that : (i) ar (AOB) + ar (OCD)
1 ar (ABCD) 2

(ii) ar (AOD) + ar (OBC) = ar (AOB) + ar (OCD) 4. The diagonals of a parallelogram ABCD intersect at a point O. Through O, a line is drawn to intersect AD at X and BC at Y. Show that XY divides the parallelogram into two parts of equal area. 5. Find the area of rhombus, the lengths of whose diagonals are 8 cm and 6 cm. Also, find length of the side of the rhombus. (Ans. Area = 24 cm2, side = 5 cm) 6. Find the area of a trapezium, whose parallel sides are 11 cm and 7 cm respectively and the distance between these sides is 6 cm. (Ans. Area = 54 cm2) 7. BD is one of the diagonals of a quadrilateral ABCD. If AM BD and CN BD. Show that : ar (quad. ABCD)
1 BD (AM + CN) 2

8. In the given figure, ABCD is a trapezium in which AB || DC and its diagonals AC and BD intersects at O. Prove that ar (AOD) = ar (BOC).

9. In the given figure, D is the midpoint of side AB of ABC and X is any point on BC. If CY || XD meets AB in Y, prove that : ar (BXY) 110
1 ar (ABC) 2

AREAS OF PARALLELOGRAMS AND TRIANGLES MATHEMATICSIX

10. Show that the diagonals of a parallelogram divides it into four triangle of equal area. 11. If each diagonal of a quadrilateral seperates it into two triangles of equal area then show that the quadrilateral is a parallelogram. 12. E is any point on the median AD of ABC. Show that ar (ABE) = ar (ACE). 13. In a triangle ABC, E is the mid point of median AD. Show that ar (BED)
1 ar (ABC). 4 14. In the given figure, the side AB of parallelogram ABCD is produced to a point P and a line through A, parallel to CP, meets CB produced in Q and the parallelogram BQRP is completed. Show that ar (parallelogram ABCD) = ar (parallelogram BQRP).

15. Diagonal BD of a quadrilateral ABCD bisects the other diagonal AC in O. Prove that : ar (ABD) = ar (CBD) 16. Let P, Q, R and S be respectively the mid points of the sides AB, BC, CD and DA of quadrilateral ABCD. 1 Show that PQRS is a parallelogram such that ar (parallelogram PQRS) ar (quad. ABCD). 2 17. In the given figure, ABCD is a parallelogram and E is any point on BC. Prove that : ar (ABE) + ar (DEC) = ar (EDA)

18. The medians CF and BE intersect at G. Prove that ar (GBC) = ar (quad. AFGE). 19. D and E are points on sides AB and AC respectively of ABC such that ar (DBC) = ar (EBC). Prove that DE || BC. 20. ABCD is a parallelogram, E and F are the mid-points of BC and CD respectively. Prove that : ar (AEF) 3 ar (parallelogram ABCD). 8 MATHEMATICSIX AREAS OF PARALLELOGRAMS AND TRIANGLES

111

PRACTICE TEST
MM : 15 General Instructions : Each Questions carries 3 marks. 1. Show that a median of a triangle divides it into two triangles of equal areas. 2. ABCD is a parallelogram and O is any point in its interior. Prove that : ar (AOB) + ar (COD) = ar (BOC) + ar (AOD) 3. Triangles ABC and DBC are on the same base BC with A, D on opposite sides of line BC, such that ar (ABC) = ar (DBC). Show that BC bisects AD. 4. D, E and F are the mid-points of the sides BC, CA and AB respectively of ABC. Prove that BDEF is a parallelogram whose area is half of ABC. Also, show that ar (DEF) = 5. In the given figure, AP || BQ || CR. Prove that ar (AQC) = ar (PBR).
1 ar (ABC). 4

Time : 1 hour

112

AREAS OF PARALLELOGRAMS AND TRIANGLES MATHEMATICSIX

CHAPTER

10
CIRCLES

Points to Remember : 1. A circle is a collection of all the points in a plane, which are equidistant from a fixed point in the plane. 2. Equal chords of a circle (or of congruent circles) subtend equal angles at the centre. 3. If the angles subtended by two chords of a circle (or of congruent circles) at the centre (corresponding centre) are equal, the chords are equal. 4. The perpendicular from the centre of a circle to a chord bisects the chord. 5. The line drawn through the centre of a circle to bisect a chord is perpendicular to the chord. 6. There is one and only one circle passing through three non-collinear points. 7. Equal chords of a circle (or of congruent circles) are equidistant from the centre (or corresponding centres). 8. Chords equidistant from the centre (or corresponding centres) of a circle (or of congruent circles) are equal. 9. If two arcs of a circle are congruent, then their corresponding chords are equal and conversely, if two chords of a circle are equal, then their corresponding arcs (minor, major) are congruent. 10. Congruent arcs of a circle subtend equal angles at the centre. 11. The angle subtended by an arc at the centre is double the angle subtended by it at any point on the remaining part of the circle. 12. Angles in the same segment of a circle are equal. 13. Angle in a semicircle is a right angle. 14. If a line segment joining two points subtends equal angles at two other points lying on the same side of the line containing the line segment, the four points lie on a circle. 15. The sum of either pair of opposite angles of a cyclic quadrilateral is 180. 16. If the sum of a pair of opposite angles of a quadrilateral is 180, then the quadrilateral is cyclic.

ILLUSTRATIVE EXAMPLES
Example 1. Give a method to find the centre of given circle. Solution. Let A, B and C be any three distinct points on the given circle. Join A to B and B to C. Draw perpendicular bisectors PQ and RS of AB and BC respectively to meet at a point O. Then, O is the centre of the circle.

MATHEMATICSIX

CIRCLES

113

Example 2. Prove that if chords of congruent circles subtend equal angles at their centres, then the chords are equal. NCERT Solution. Given : Two congruent circle C(O, r) and C(O, r) such that AOB = COD. To prove : AB CD Proof : In AOB and COD OA = OC OB = OD AOB = COD AOB COD

(each = r) (each = r) (given) (SAS congruence condition)

(cpct) AB CD Example 3. If two circles intersect at two points, prove that their centres lie on the perpendicular bisector of the common chord. NCERT Solution. Given : Two circles, with centres O and O intersect at two points A and B so that AB is the common chord of the two circles and OO is the line segment joining the centres of the two circles. Let OO intersect AB at M. To prove : OO is the perpendicular bisector of AB. Construction : Draw line segments OA, OB, OA and OB. Proof : In OAO and OBO, we have OA = OB (Radii of same circle) OA = OB (Radii of same circle) OO = OO (Common side) OAO OBO (SSS congruence condition) AOO = BOO (cpct) AOM = BOM ...(1) Now, In AOM and BOM, we have OA = OB (Radii of same circle) AOM = BOM (from (1)) OM = OM (common side) BOM (SAS congruence condition) AM = BM and AMO = BMO (cpct) But, AMO + BMO = 180 2AMO = 180 AMO = 90 Thus, AM = BM and AMO = BMO = 90 Hence, OO is the perpendicular bisector of AB. Example 4. Find the length of a chord which is at a distance of 8 cm from the centre of a circle of radius 17 cm. Solution. Let AB be a chord of a circle with centre O and radius 17 cm. Draw OC AB. Join O to C. Then, OC = 8 cm . OA = 17 cm In right triangle OAC, using pythagoras theorem OA2 = OC2 + AC2 172 = 82 + AC2 AC2 = 172 82 AC2 = 225 AC = 15 cm Since, perpendicular from the centre of a circle to a chord bisects a chord, we have AB = 2 AC = 2 15 cm = 30 cm Ans. 114 CIRCLES MATHEMATICSIX

Example 5. Two circles of radii 5 cm and 3 cm intersect at two points and the distance between their centres is 4 cm. Find the length of the common chord. NCERT Solution. Clearly, the common chord AOB is the diameter of the cirlce with radius 3 cm.

Length of common chord AOB = 2 3 cm = 6 cm Ans. Example 6. AB and CD are two parallel chords of a circle which are on opposite sides of the centre such that AB = 10 cm, CD = 24 cm and the distance between them is 17 cm. Find the radius of the circle. Solution. Draw ON AB and OM CD. Since, ON AB, OM CD and AB || CD M, O, N are collinear points. MN = 17 cm Let ON = x cm, then OM = (17 x) cm. Now, we know that perpendicular from the centre of a circle to a chord bisects the chord,
AN CM 1 1 AB 10 cm 5 cm, and 2 2

1 1 CD 24 cm 12 cm 2 2 In ONA, OA2 = ON2 + AN2 r2 = x2 + (5)2 Again, In OCN, OC2 = OM2 + CM2 r2 = (17 x)2 + (12)2 from (1) and (2), we get

...(1) ...(2)

x 2 (5) 2 (17 x) 2 (12) 2


x 2 25 289 x 2 34 x 144 34x = 408 x = 12 Putting x = 12 in (1), r2 = (12)2 + (5)2 = 144 + 25 = 169 r = 13 Hence, radius of circle is 13 cm. Ans. Example 7. In a circle of radius 5 cm, AB and AC are two chords such that AB = AC = 6 cm. Find the length of chord BC. Solution. Given, OA = OC = 5 cm and AB = AC = 6 cm Since, points A and C are equidistant from A, so AO is the perpendicular bisector of BC. ADB = 90 Now, In right ADC, AC2 = AD2 + CD2 MATHEMATICSIX CIRCLES 115

(6)2 = AD2 + CD2 CD2 = 36 AD2 Also, In BDO, OB2 = BD2 + OD2 (5)2 = BD2 + (AO AD)2 25 = BD2 + (5 AD)2 BD2 = 25 (5 AD)2 CD2 = 25 (5 AD)2 ( BD = CD) from (1) and (2), we get, 36 AD2 = 25 (5 AD)2 36 AD2 = 25 25 AD2 + 10 AD AD = 3.6 cm using in eq. (1), CD2 = 36 (3.6)2 = 36 12.96 = 23.04

...(1)

...(2)

CD 23.04 4.8 cm BC = 2CD = 2 4.8 cm = 9.6 cm Ans. Example 8. Prove that the line joining the mid-points of two parallel chords of a circle passes through the centre of the circle. Solution. Given : M and N are the mid-points of two parallel chords AB and CD respectively of circle with centre O. To prove : MON is a straight line. Construction : Join OM, ON and draw OE || AB || CD. Proof : Since, the line segment joining the centre of a circle to the mid point of a chord is perpendicular to the chord OM AB and ON CD OM OE EOM = 90 Also, OM CD and CD || OE ON OE EON = 90 EOM + EON = 90 + 90 = 180 Hence, MON is a straight line. Example 9. In the given figure, there are two concentric circles with common centre O. l is a line intersecting these circles at A, B, C and D. Show that AB = CD. Solution. Draw OM l. We know that perpendicular from the centre of a circle to a chord bisects a chord. Now, BC is a chord of smaller circle and OM BC. BM = CM ...(1) Again, AD is a chord of bigger circle and OM AD. AM = DM ...(2) Subtracting (1) from (2), we get AM BM = DM CM AB = CD. Hence proved. Now, OM AB and AB || OE 116 CIRCLES MATHEMATICSIX

Example 10. If two equal chords of a circle intersect within the circle, prove that the segments of one chord are equal to corresponding segments of the other chord. NCERT Solution. Given : AB and CD are chords of a circle with centre O. AB and CD intersect at P and AB = CD. To prove : (i) AP = PD (ii) PB = CP Construction : Draw OM AB and ON CD. Join O to P. Proof : AM MB
1 AB 2 ( perpendicular from centre bisects the chord) 1 CD 2 ( perpendicular from centre bisects the chord)

also,

CN = ND =

1 1 AB = CD 2 2 AM = ND and MB = CN ...(1) Now, in OMP and ONP, we have OM = ON (equal chords of a circle are equidistant from the centre). OMP = ONP (each = 90) OP = OP (common side) OMP ONP (RHS congruence condition) MP = PN ...(2) (cpct) Adding (1) and (2), we get AM + MP = ND + PN AP = PD Subtracting (2) from (1), we get MB MP = CN PN PB = CP Hence proved. Example 11. A circular park of radius 20 m is situated in a colony. Three boys Ankur, Syed and David are sitting at equal distance on its boundary each having a toy telephone in his hands to talk each other. Find the length of the string of each phone. NCERT Solution. Let ABC is an equilateral triangle of side 2x metres.

But

AB = CD

BC 2 x metres x metres 2 2 In right ABM, AM2 = AB2 BM2

Clearly, BM

(2 x) 2 ( x) 2 4 x 2 x 2 3x 2
AM =

3x m

Now, OM = AM OA = ( 3 x 20) metres In right OBM, we have OB2 = BM2 + OM2 (20)2 = x2 + ( 3 x 20) 2 400 = x2 + 3x2 + 400 40 3 x 4x2 40 3 x = 0 4x (x 10 3 ) = 0 x = 0 or x 10 3 = 0 MATHEMATICSIX CIRCLES 117

But x 0 x 10 3 0 x 10 3 m Now, BC = 2 MB = 2x = 2 10 3 m = 20 3 m Hence, the length of each string = 20 3 m Ans. Example 12. If O is the centre of the circle, find the value of x, in each of the following figures:

Solution.

(i) (ii) (iii) (i) BAC = BDC = 25 ( angles in same segment are equal) Now, In BCD, DBC + BDC + x = 180 ( angles sum property of a triangle) 75 + 25 + x = 180 100 + x = 180 x = 180 100 = 80 Ans. (ii) Since, OB = OA (radii of same circle) OBA is an isosceles triangle OBA = BAO = 25 ...(1) Similarly, OAC is an isosceles. OCA = OAC = 30 ...(2) adding (1) and (2), we get OAB + OAC = 25 + 30 BAC = 55 Now, BOC = 2BAC ( The angle subtended by an arc at the centre is double the angle subtended by it at any point on the remaining part of the circle) x = 2 55 = 110 Ans. (iii) Reflex AOC = 360 120 = 240 1 ( same as above) ABC = .reflex AOC 2 1 240 120 2 x = 120 Ans. Example 13. In the given figure, AB is a diameter of a circle with centre O and chord CD = radius OC. If AC and BD when produced meet at P, prove that APB = 60. Solution. Join O to D and B to C. Now, CD = OC = OD (radii of same circle) OCD is equilateral COD = 60 and CBD
1 1 COD 60 30 2 2

( angle made by

at centre = 2 angle at any point on its remaining part). CIRCLES MATHEMATICSIX

118

Now, BCA + BCP = 180 ( linear pair) But, BCA = 90 ( angle in semi-circle) 90 + BCP = 180 BCP = 90 Now, in BCP, BCP + CBP + CPB = 180 90 + 30 + CPB = 180 CPB = 180 120 = 60 APB = 60 ( CPB = APB) Hence proved. Example 14. In the following figures, if O is the centre of the circle, find x.

ACB = 90 ( angle is a semi circle) ACB = 180 135 ( opposite angles of cyclic quadrilateral are supplementary) Now, In ABC, CAB + ACB + ABC = 180 ( angle sum property) x + 90 + 45 = 180 x = 45 Ans. (ii) Take any point P on the major arc. 1 Now, APC . AOC 2 ( The angle subtended by an arc at the centre is double the angle subtended by it at any point on the remaining part of the circle) 1 120 60 . 2 Also, APC + ABC = 180 ( opp. angles of cyclic quadrilateral are supplementary) 60 + ABC = 180 ABC = 180 60 = 120 Now, ABC + DBC = 180 ( linear pair) 120 + DBC = 180 DBC = 60 x = 60 Ans. Example 15. Prove that the quadrilateral formed by angle bisectors of a cyclic quadrilateral is also cyclic. Solution. Given : A cyclic quadrilateral ABCD in which AP, BP, CR and DR are the bisectors of A, B, C and D respectively, forming a quadrilateral PQRS. To prove : PQRS is a cyclic quadrilateral. Solution. (i)

MATHEMATICSIX

CIRCLES

119

Proof : In APB, APB + PAB + PBA = 180 ( angle sum property of a triangle) Also, InDRC, CRD + RCD + RDC = 180 ( same as above) and
APB CRD 1 1 A B 180 2 2

...(1) ...(2)

1 1 C D 180 2 2 Adding (1) and (2), we get APB CRD

1 (A B C D) 360 2

1 APB CRD (360) 360 (A + B + C + D = 360) 2 APB + CRD = 180 Thus, two opposite angles of quadrilateral PQRS are supplementary. Quadrilateral PQRS is cyclic. Example 16. If diagonals of a cyclic quadrilateral are diameters of the circle through the vertices of the quadrilateral, prove that it is a rectangle. NCERT Solution. Given : Diagonals AC and BD of a cyclic quadrilateral are diameter of the circle through the vertices A, B, C and D of the quadrilateral ABCD. To prove : ABCD is a rectangle. Proof : Since AC is a diameter. ABC = 90 ( angle in a semi-circles is 90) also, quadrilateral ABCD is a cyclic. ADC = 180 ABC ADC = 180 90 = 90 Similarly, BAC = BCD = 90. Now, each angle of a cyclic quadrilateral ABCD is 90. ABCD is a rectangle. Example 17. If the non-parallel sides of a trapezium are equal, prove that it is cyclic. NCERT Solution. Given : A trapezium ABCD in which AB || DC and AD = BC. To prove : ABCD is a cyclic trapezium.

Construction : Draw DE AB and CF AB. Proof : In order to prove that ABCD is a cyclic trapezium, it is sufficient to prove that B + D = 180. Now, In DEA and CFB, we have AD = BC (given) DEA = CFB (each = 90) DE = CF (distance between two parallel lines is always equal) DEA CFB (RHS congruence condition) A = B and ADE = BCF (cpct) Now, ADE = BCF 90 + ADE = 90 + BCF EDC + ADE = FCD + BCF ( EDC = 90, FCD = 90) ADC = BCD D = C Thus, A = B and C = D. 120 CIRCLES MATHEMATICSIX

Now, A + B + C + D = 360 ( sum of angles of a quadrilateral is 360) 2B + 2D = 360 360 B D 180 2 Hence, ABCD is a cyclic trapezium. Example 18. Two circles intersect at two points B and C. Through B, two line segments ABD and PBQ are drawn to intersect the circles at A, D and P, Q respectively (see figure). Prove that ACP = QCD. NCERT Solution. Since angles in the same segment of a circle are equal. ACP = ABP ...(1) and QCD = QBD ...(2) But, ABP = QBD ...(3) (vertically opposite angles) from (1), (2) and (3), we get ACP = QCD

Example 19. Two circles are drawn taking two sides of a triangle as diameters, prove that the point of intersection of these circles lie on the third side. NCERT Solution. Given : Two circles are drawn with sides AB and AC of ABC as diameters. The circles intersect at D. To prove : D lies on BC. Construction : Join A to D. Proof : Since AB and AC are diameters of the circles, ADB = 90 and ADC = 90 ( angles in a semi-circle is 90) Adding, we get, ADB + ADC = 90 + 90 = 180 BDC is a straight line. Hence, D lies on BC. Example 20. ABC and ADC are two right triangles with common hypotenuse AC. Prove that CAD = CBD. Solution. ABC and ADC are right angled triangles with common hypotenuse AC. Draw a circle with AC as diameter passing through B and D. Join B to D. NCERT

Clearly, CAD = CBD. ( angles in the same segment are equal) Hence proved. Example 21. Prove that a cyclic parallelogram is a rectangle. Solution. Given : ABCD is parallelogram inscribed in a circle. To prove : ABCD is a rectangle. Proof : Since ABCD is a cyclic quadrilateral, A + C = 180 ...(1) But, A = C ...(2) (opposite angles of a parallelogram are equal) MATHEMATICSIX CIRCLES

NCERT

121

from (1) and (2), we get, A + A = 180 2A = 180 A = 90 i.e. A = C = 90 Similarly, B = D = 90 ABCD is a parallelogram whose each angle is equal to 90. ABCD is a rectangle. Example 22. Prove that the circle drawn with any side of a rhombus as diameter, passes through the point of intersection of its diagonals. NCERT Solution. Given : ABCD is a rhombus. AC and BD are its two diagonals which bisect each other at right angles. To prove : A circle drawn on AB as a diameter will pass through O. Construction : From O, draw PQ || AD and EF || AB,
1 1 AB = DC 2 2 AQ = DP ( Q and P are mid-points of AB and DC respectively) Similarly, AE = OQ AQ = OQ = QB A circle drawn with Q as a centre and radius AQ passes through A, O and B, which proves the desired result. Example 23. ABCD is a parallelogram. The circle through A, B and C intersect CD (produced if necessary) at E. Prove that AE = AD. NCERT

Proof : Since, AB = DC

Since ABCE is a cyclic quadrilateral, AED + ABC = 180 ...(1) Now, CDE is a straight line. ADE + ADC = 180 ...(2) ( ADC and ABC are opposite angles of a parallelogram i.e. ADC = ABC) From (1) and (2), we get AED + ABC = ADE + ABC AED = ADE In AED, AED = ADE AD = AE (sides opposite to equal angles are equal) Hence proved. Example 24. AC and BD are chords of a circle which bisect each other. Prove that : (i) AC and BD are diameters (ii) ABCD is a rectangle. NCERT Solution. (i) Let AB and CD be two chords of a circle with center O. Let they bisect each other at O. Join AC, BD, AD and BC. Solution. 122 CIRCLES MATHEMATICSIX

Now, In AOC and BOD, we have OA = OB ( O is mid-point of AB) AOC = BOD (vertically opp. angles) OC = OD ( O is mid-point of CD) AOC BOD (SAS congruence condition) AC = BD (cpct) = ...(1) Similarly, from AOD and BOC, we have = Adding (1) and (2), we get, ...(2)

CD divides the circle into two equal parts CD is a diameter. Similarly, AB is a diameter. (i) Since, AOC BOD (proved above) OAC i.e. BAC = OBD i.e. ABD AC || BD. Again, AOD COB (proved above) AD || CB ABCD is a cyclic parallelogram. DAC = DBA ...(3) ( opp. angles of a parallelogram) also, ABCD is a cyclic quadrilateral, DAC + DBA = 180 ...(4) from (3) and (4), we get
180 90 2 Hence, ABCD is a rectangle. Example 25. Bisectors of angles A, B and C of a triangle ABC intersect the circumcircle at D, E and F respecDAC DBA

tively. Prove that the angles of the DEF are 90 Solution.

1 1 1 T A, 90 B and 90 C. NCERT 2 2 2

We have, D = EDF = EDA + ADF = EBA + FCA ( EDA and EBA are in the same segment are in the same segment of a circle) EDA = EBA. Similarly, ADF and FCA are the angles in the same segment, ADF = FCA
1 1 1 B C (B C) 2 2 2 1 (180 A) [ A + B + C = 180] 2

MATHEMATICSIX

CIRCLES

123

1 90 A 2 Similarly, other two angles of DEF are


1 1 90 B and 90 C. 2 2 Hence proved.

PRACTICE EXERCISE
1. Show how to complete a circle if an arc of the circle is given. 2. The radius of a circle is 13 cm and the length of one of its chord is 10 cm. Find the distance of the chord from the centre. 3. AB and CD are two parallel chords of a circle which are on the opposite sides of the centre such that AB = 8 cm and CD = 6 cm. Also, radius of circle is 5 cm. Find the distance between the two chords. 4. Two chords AB and AC of a circle are equal. Prove that the centre of the circle lies on the angle bisector of BAC. 5. If two circles intersect in two points, prove that the line through their centres is the perpendicular bisector of the common chord. 6. If a diameter of a circle bisects each of the two chords of the circle, prove that the chords are parallel. 7. In the given figure, two equal chords AB and CD of a circle with centre O, when produced meet at a point P. Prove that (i) BP = DP (ii) AP = CP.

8. Two circles whose centres are O and O intersects at P. Through P, a line l parallel to OO, intersecting the circles at C and D, is drawn. Prove that CD = 2.OO. 9. In the given figure, O is the centre of the circle and MO bisects AMC. Prove that AB = CD.

10. Show that if two chords of a circle bisect each other, they must be the diameters of the circle. 11. In the given figure, OD is perpendicular to the chord AB of a circle with centre O. If BC is a diameter, show that AC || OD and AC = 2OD.

124

CIRCLES

MATHEMATICSIX

12. Prove that two different circles cannot intersect each other at more than two points. 13. Two equal circles intersect in P and Q. A straight line through P meets the circle in X and Y. Prove that QX = QY.

14. Two circles of radii 5 cm and 3 cm intersect at two points and the distance between their centres is 4 cm. Find the length of the common chord. 15. AB and AC are two equal chords of a circle whose centre is O. If OD AB and OE AC, prove that ADE is an isosceles triangle. 16. Prove that angle is a semi-circle is a right angle. 17. Prove that the angles in the same segment of a circle are equal. 18. Prove that the angle formed by a chord in the major segment is acute. 19. Prove that the angle formed by a chord in the minor segment is obtuse. 20. If O is the centre of a circle, find the value of x in the following figures:

(i)

(ii)

(iii)

(iv) MATHEMATICSIX

(v) CIRCLES

(vi) 125

21. In the given figure, two circles intersect at P and Q. PR and PS are respectively the diameters of the circle. Prove that the points R, Q, S are collinear.

22. Prove that the circle drawn on any one of the equal sides of an isosceles triangle as diameter, bisects the third side of the triangle. 23. In the given figure, O is the centre of the circle. Prove that a + b = c.

24. In an isosceles triangle ABC with AB = AC, a circle passing through B and C intersects the sides AB and AC at D and E respectively. Prove that DE || BC.

25. In the given figure, PQ is a diameter of a circle with center O. If PQR = 65, SPR = 35 and PQT = 50, find : (i) QPR (ii) QPT (iii) PRS

126

CIRCLES

MATHEMATICSIX

26. In the given figure ABC is isosceles with AB = AC and ABC = 55. Find BDC and BEC.

27. Find the angles marked with a letter. O is the centre of the circle.

(i)

(ii)

(iii)

(iv) 28. In the following figure, find x and y.

(v)

(vi)

29. Prove that every cyclic parallelogram is a rectangle. 30. If two non-parallel sides of a trapezium are equal, prove that it is cyclic. 31. Prove that cyclic trapezium is always isosceles and its diagonals are equal. MATHEMATICSIX CIRCLES 127

32. In an isosceles ABC with AB = AC, a circle passing through B and C intersects the sides AB and AC at D and E respectively. Prove that DE || BC. 33. In the given figure, ABCD is a parallelogram. A circle through A, B, C intersects CD produced at E. Prove that AD = AE.

34. The bisectors of the opposite angles A and C of a cyclic quadrilateral ABCD intersect the circle at the points E and F respectively. Prove that EF is a diameter of the circle.

35. Prove that the angle bisectors of the angles formed by producing opposite sides of a cyclic quadrilateral (provided the are not parallel) intersect at a right angle.

128

CIRCLES

MATHEMATICSIX

PRACTICE TEST
General Instructions : MM : 30 Q. 1-4 carry 2 marks, Q. 5-8 carry 3 marks and Q. 9-10 carry 5 marks each. 1. In the given figure, A, B, C and D are four points on the circle. AC and BD intersect at a point E such that BEC = 120, and ECD = 20. Find BAC. Time : 1 hour

2. Prove that the line joining the mid-points of the two parallel chords of a circle passes through the centre of the circle. 3. Find the value of x and y:

4. If O is the centre of the circle, find the value of x.

5. If two intersecting circles have a common chord of length 16 cm, and if the radii of two circles are 10 cm and 17 cm, find the distance between their centres. 6. If two non-parallel sides of a trapezium are equal, prove that it is cyclic. 7. In the given figure, AB is a chord of a circle with centre O and AB is produced to C such that BC = OB. Also, CO is joined and produced to meet the circle in D. If ACD = b and AOD = a, prove that a = 3b.

MATHEMATICSIX

CIRCLES

129

8. In the given figure, ABCD is a cyclic quadrilateral. A circle passing through A and B meets AD and BC in E and F respectively. Prove that EF || DC.

9. Prove that the quadrilateral formed by angle bisectors of a cyclic quadrilateal is also cyclic. 10. The angle subtended by an arc of a circle at the centre is double the angle subtended by it at any point on the remaining part of the circle. Prove it.

ANSWERS OF PRACTICE EXERCISE


2. 12 cm 3. 7 cm 14. 6 cm 20. (i) 100 25. (i) 15 26. (i) 70 27. (i) a = 50 28. x = 40, y = 25 (ii) 50 (ii) 40 (ii) 110 (ii) b = 40 (iii) c = 35 (vi) a = 41, b = 41, c = 41 (iii) 55 (iii) 40 (iv) 35 (v) 30 (vi) 50

(iv) a = 45, b = 64, c = 58 (v) x = 40, y = 32, z = 40

ANSWERS OF PRACTICE TEST


1. 100 3. x = 75, y = 105 4. 55 5. 21 cm

130

CIRCLES

MATHEMATICSIX

CHAPTER

11
CONSTRUCTIONS

Points to Remember : Using a graduated scale and a compass, we can construct the following : 1. Perpendicular bisector of a line segement. 2. Bisector of an angle. 3. Angle of measures 60, 90, 45 etc. 4. A triangle given its base, a base angle and the sum of the other two sides. 5. A triangle give its base, a base angle and the difference of the other two sides. 6. A triangle given its perimeter and its base angles.

ILLUSTRATIVE EXAMPLES
Example 1. Draw a line segment AB = 7 cm. Construct its perpendicular bisector. Give step of constructions also. Solution.

1 (ii) Taking A and B as centres and radius more than AB, draw arcs on both sides of the line 2 segment AB. (iii) Let these arcs intersect each other at P and Q. Join P to Q. (iv) Let PQ intersect AB at M. The line PMQ is the reqired perpendicular bisector of AB. Example 2. Construct an angle of 60 at the initial point of a given ray. Solution. Step of constructions: (i) Draw the ray AB. (ii) Taking A as centre and some suitable radius, draw an arc of a circle, which intersects AB, say at a point D. (iii) Taking D as centre and with the same radius as before, draw an arc intersecting the previously drawn arc, say at a point E. (iv) Draw the ray AC passing through E. Then, CAB is the required angle of 60.

Step of constructions: (i) Draw a line segment AB = 7 cm

MATHEMATICSIX

CONSTRUCTIONS

131

Example 3. Construct the bisector of a given angle ABC = 70. Solution. Step of constructions: (i) Draw ABC = 70 using a protractor. (ii) Taking B as centre and any radius, draw an arc to intersect the rays BA and BC, say at E and D respectively. (iii) Taking E and D as centres and with the radius more than
1 DE, draw arcs to intersect each other, say 2

at F. (iv) Draw the ray BF. This ray BF is the required bisector of the ABC. Example 4. Construct a ABC in which BC = 5 cm, B = 60 and the sum of the other two sides is 7 cm. Solution.

Step of Constructions: (i) Draw BC = 5 cm. (ii) Draw XBC = 60 (iii) On the ray BX, mark off point D such that BD = 7 cm. (iv) Join D to C. (v) Draw perpendicular bisector EF of CD. Let it intersects BD at A. (vi) Join A to C. ABC is the required triangle. Example 5. Construct a ABC in which BC = 5.5 cm, B = 30 and AB AC = 2cm. Solution.

132

CONSTRUCTIONS

MATHEMATICSIX

Step of constructions: (i) Draw BC = 5.5 cm. (ii) Draw XBC = 30 (iii) On the ray BX, mark off point D such that BD = 2 cm. (iv) Join D to C. (v) Draw perpendicular bisector EF of CD. Let it intersects BX at A. (vi) Join A to C. ABC is the required triangle. Example 6. Construct a ABC in which B = 45, C = 60 and AB + BC + AC = 13 cm. Solution. Step of constructions:

(i) Draw XY = 13 cm. (ii) Draw MXY = 45 and NYX = 60 (iii) Draw angle bisectors of MXY and NYX, meeting at a point, say A. (iv) Draw perpendicular bisector of XA and YA, meeting XY at B and C respectively. (v) Join A to B and A to C. ABC is the required triangle.

PRACTICE EXERCISE
1. Construct the angles of the following measurements :
1 (ii) 22 (iii) 75 (iv) 15 (v) 135 2 Construct an equilateral triangle XYZ, in which XY = 5.4 cm. Construct a triangle ABC, in which BC = 5.8 cm, ACB = 45 and AB + AC = 7 cm. Construct a triangle ABC, in which AC = 5 cm, A = 60 and sum of the sides AB and BC is 8.5 cm. Construct a triangle ABC in which AC = 5.5 cm, A = 30 and BC AB = 1.6 cm. Construct a triangle ABC in which AB = 4.2 cm, B = 45 and BC AC = 2 cm. Construct a triangle ABC in which B = 60, C = 45 and AB + BC + AC = 11 cm. Construct a triangle PQR in which Q = 45, R = 30 and PQ + QR + PR = 9.5 cm. Construct a right-angled triangle whose perimeter is 12 cm and one acute angle is equal to 60. Construct a ABC such that BC = 6 cm, AB = 6 cm and median AD = 4 cm.

(i) 90

2. 3. 4. 5. 6. 7. 8. 9. 10.

MATHEMATICSIX

CONSTRUCTIONS

133

PRACTICE TEST
MM : 20 General Instructions : Each Questions carry 5 marks. 1. 2. 3. 4. Construct an angle of 90 at the initial point of a ray AB and justify your construction. Construct a ABC in which BC = 8 cm, B = 60, and AB + AC = 13.5 cm. Construct a ABC in which BC = 6 cm, B = 60 and AC AB = 2.2 cm. Construct a PQR in which Q = 30, R = 45 and PQ + QR + PR = 10 cm. Time :
1 2

hour

134

CONSTRUCTIONS

MATHEMATICSIX

CHAPTER

12
HERONS FORMULA

Points to Remember :
1 Base height 2 3 (side) 2 2. Area of an Equilateral triangle 4 a 3. Area of a isosceles triangle 4b 2 a 2 , where, a is base and b represents equal sides. 4 4. Herons Formula : If a, b, c denote the lengths of the sides of a triangle, then its Area,

1. Area of right triangle

s ( s a )( s b )( s c )

a bc 2 5. Area of a quadrilateral can be calculated by dividing the quadrilateral into triangles and using herons formula for calculating area of each triangle.

where,

ILLUSTRATIVE EXAMPLES
Example 1. Find the area of a triangle whose sides are 13 cm, 14 cm and 15 cm respectively. Solution. Let a = 13 cm, b = 14 cm, c = 15 cm a b c 13 14 15 21 cm s 2 2 Area s( s a )( s b )( s c)
21( 21 13)( 21 14)(21 15) sq cm.

21 8 7 6 sq. cm 3 7 2 2 2 7 2 3 sq. cm = 2 2 3 7 sq. cm. = 84 sq. cm. Area of triangle = 84 sq. cm. Ans. Example 2. A traffic signal board, indicating SCHOOL AHEAD is an equilateral triangle with side a. Find the area of the signal board, using herons formula if it perimeter is 180 cm. NCERT Solution. Given perimeter of equilateral triangle = 180 cm. Side of an equilateal triangle is a cm, then its perimeter is 3a cm. 3a = 180 a = 60 cm

Now, s a a a 60 60 60 180 90 cm. 2 2 2 Using herons formula, Area


s( s a )( s a )( s a ) 90(90 60)(90 60)(90 60) cm 2

90 30 30 30 cm 2 3 3 3 3 3 10000 cm 2 3 3 100 3 cm 2 900 3 cm 2 Ans.

MATHEMATICSIX

HERONS FORMULA

135

Example 3. A triangular park ABC has sides 120 m, 80 m and 50 m (see figure). A gardener Dhania has to put a fence all around it and also plant grass inside. How much area does she need to plant? Find the cost of fencing it with barbed wire at the rate of Rs 20 per metre leaving a space 3 m wide for a gate on one side. Solution.

(i) For Area of the park : Let a = 120 m, b = 80 m, c = 50 m


s a b c 120 80 50 125 m 2 2
125 (125 120)(125 80)(125 50) m 2

Area s( s a )( s b )( s c)

125 5 45 75 m 2 375 15 m 2 Also, Perimeter of the park = AB + BC + AC = 250 m length of wire needed for fencing = 250 m 3 m (to left for gate) = 247 m and cost of fencing = 247 20 Rs = Rs. 4940 Ans. Example 4. Triangular side walls of a flyover have been used for advertisements. The sides of the walls are 122 m, 22 m and 120 m (see figure). The advertisements yield an earning of Rs. 5000 per m2 per year. A company hired one of its walls for 3 months, how much rent did it pay? NCERT

136

HERONS FORMULA

MATHEMATICSIX

Solution.

We will calculate area of triangular wall here, a = 122 m, b = 22 m, c = 120 m


s a b c 122 22 120 264 m m 132 m 2 2 2

Area s( s a )( s b)( s c) 132 (132 122) (132 22) (132 120) m 2

132 10 110 12 m 2 11 12 10 10 11 12 m 2 10 11 12 m 2 1320 m 2 Now, Rent charges = Rs. 5000 per m2 per year Rent charged from company for 3 months
3 12 = Rs. 16,50,000 Ans. Example 5. There is a slide in a park. One of the its side walls has been painted, in colour with a message KEEP THE PARK GREEN AND CLEAN, (see figure). If the sides of the wall are 15 m, 11 m and 6 m, find the area painted in colour. NCERT Rs. 5000 1320

Solution.

here, Area

a = 15 m, b = 11m, c = 6m
s a b c 15 11 6 32 m m 16 m 2 2 2

s ( s a )( s b)( s c) 16(16 15)(16 11)(16 6) m 2 16 1 5 10 m 2

4 4 5 2 5 m 2 4 5 2 m 2 20 2 m 2 Area painted in a colour = Area of side wall

= 20 2 m 2 Ans. Example 6. An isosceles triangle has perimeter 30 m and each of the equal sides is 12 cm. Find the area of the triangle. NCERT Solution. here, a = b = 12 cm. also, a + b + c = 30 m 12 + 12 + c = 30 cm c = 6 cm MATHEMATICSIX HERONS FORMULA 137

abc 30 cm cm 15 cm 2 2 Area of the triangle s


s ( s a )( s b)( s c) 15(15 12)(15 12)(15 6) cm 2

15 3 3 9 cm 2 5 3 3 3 3 3 cm 2 3 3 5 3 cm 2 9 15 cm 2 Ans. Example 7. The perimeter of a triangle is 450 m and its sides are in the ratio of 13 : 12 : 5. Find the area of the triangle. Solution. Let the sides of the triangle by 13 x, 12 x and 5x. Given, perimeter of triangle = 450 m 13 x + 12 x + 5 x = 450 30x = 450 x = 15 Sides are 13 15 m, 12 15 m and 5 15 m i.e. 195 m, 180 m and 75 m. Let a = 195 m, b = 180 m, c = 75 m
s a b c 195 180 175 450 m 225 m 2 2 2

Area s( s a)(s b)(s c)


225(225 195)(225 180)(225 75) sq. m

225 30 45 150 sq. m.

15 15 2 15 3 15 5 5 2 3 sq. m.
= 15 15 2 3 5 sq. m. = 6750 sq. m Ans. Example 8. A park, in the shape of a quadrilateral ABCD, has C = 90, AB = 9 m, BC = 12 m, CD = 5m and AD = 8 m. How much area does it occupy? NCERT Solution. Area of BCD
1 1 BC CD 12 5 m 2 30 m 2 2 2 Now, In BCD, BD2 = BC2 + CD2

BD 12 2 52 m 144 25 m 169 m 13 m For ABD, let a = 13 m, b = 8 m, c = 9 m. Now, s


a b c 13 8 9 30 m m 15 m 2 2 2
s ( s a )( s b )( s c )

Area of ABD

15(15 13)(15 8)(15 9) m 2

138

HERONS FORMULA

MATHEMATICSIX

15 2 7 6 m 2 2 2 3 3 5 7 m 2 2 3 35 m 2 6 35 m 2 6 5.9 m 2 (approx.) = 35.4 m2 (approx.) Required Area = ar (ABD) + ar (BCD) = 35.4 m2 + 30 m2 = 65.4 m2 (approx.) Ans. Example 9. A parallelogram, the length of whose sides are 60 m and 25 m has one diagonal 65 m long. Find the area of the parallelogram. Solution. Let ABCD be the given parallelogram. Area of parallelogram ABCD = area of ABC + area of ACD = 2 area of ABC Let, a = 60 m, b = 65 m, c = 25 m

60 65 25 m 75 m 2

area of ABC s( s a )( s b )( s c)
75(75 60)(75 65)(75 25) m 2

75 15 10 50 m 2 = 5 3 5 2 5 m2 = 750 m2 Area of parallelogram ABCD = 2 area of ABC = 2 750 m2 = 1500 m2 Ans. Example 10. A triangle and a parallelogram have the same base and the same area. If the sides of the triangle are 26 cm, 28 cm and 30 cm, and the parallelogram stands on the base 28 cm, find the height of the parallelogram. NCERT Solution. For the triangle ; Let a = 26 cm, b = 28 cm, c = 30 cm

then, s

a b c 26 cm 28 cm 30 cm 84 cm 42 cm 2 2 2

Now, Area of triangle s( s a )( s b )( s c)


42(42 26)(42 28)(42 30) cm 2

42 16 14 12 cm 2 2 3 7 4 4 2 7 2 2 3 cm 2 2 2 2 2 3 3 4 4 7 7 cm 2 = 2 2 3 4 7 cm2 = 336 cm2 For the parallelogram : Area = base height

height

Area ( Area of parallelogram = Area of triangle) base 336 cm 12 cm. Ans. 28

MATHEMATICSIX

HERONS FORMULA

139

Example 11. Find the area of a quadrilateral ABCD, in which B = 90, AB = 15 cm, BC = 36 cm, CA = 24 cm and DA = 21 cm. Solution. Join A to C. In ABC, B = 90 Area of ABC
1 36 15 cm 2 2 = 270 cm2

also, In ABC, AC2 = AB2 + BC2 = 152 + 362 = 225 + 1296 = 1521
AC 1521 cm 39 cm Now, For area of ACD, we will use heros formula. let, a = 39 cm, b = 24 cm, c = 21 cm.

a b c 39 24 21 42 cm 2 2

Area of ACD s( s a )( s b )( s c)
42 (42 39)(42 24)(42 21) cm 2

42 3 18 21 cm 2 126 3 cm 2 218.232 cm 2 Area of ABCD = 270 cm2 + 218.232 cm2 = 488.232 cm2 Ans. Example 12. Radha made a picture of an aeroplane with coloured paper as shown in the figure. Find the total area of the paper used. NCERT

Solution.

Area I = area of isosceles triangle with a = 1 cm and b = 5 cm a 4b 2 a 2 4


1 4 25 (1) 2 cm 2 4

140

HERONS FORMULA

MATHEMATICSIX

99 cm 2 2.5 cm 2 (approx) 4 Area II = area of rectangle = length breadth = 6.5 1 cm2 = 6.5 cm2 Area III = Area of Trapezium = 3 area of equilateral triangle with a = 1 cm 3 3 1.732 5.196 (1) 2 cm 2 cm 2 cm 2 4 4 4 = 1.299 cm2 1.3 cm2 (approx)
= 3
1 2 2 Area of IV and V 2 6 1.5 cm 9 cm 2 Total area of paper used = area I + area II + area III + area IV + area V = 2.5 cm2 + 6.5 cm2 + 1.3 cm2 + 9 cm2 = 19.3 cm2 (approx) Ans. Example 13. An Umbrella is made by stitching 10 triangular pieces of cloth of two different colours (see figure), each piece measuring 20 cm, 50 cm and 50 cm. How much cloth of each colour is required for the umbrella? NCERT Solution. In one triangular piece, a = 20 cm, b = 50 cm, c = 50 cm.

Now, s

20 50 50 cm 120 cm 60 cm 2 2

Area s ( s a )( s b)( s c)
60 (60 20)(60 50)(60 50) cm 2

60 40 10 10 cm 2 200 6 cm 2

Area of 5, Ist coloured triangles = 5 200 6 cm2 = 1000 6 cm2 and area of 5, IInd coloured triangles = 5 200 6 cm2 = 1000 6 cm2 Example 14. A kite in the shape of a square with a diagonal 32 cm and an isosceles triangle of base 8 cm and sides 6 cm each is to be made of three different shades as shown in figure. How much paper of each shade has been used in it. NCERT Solution. Each diagonal of square = 32 cm 1 CO 32 cm 16 cm 2 Area of each shaded portion I and II 1 base height 2 1 32 16 cm 2 256 cm 2 2 And, ara of IIIrd portion with sides 6 cm, 6 cm and 8 cm. a 8 4b 2 a 2 4(6) 2 (8) 2 cm 2 4 4 MATHEMATICSIX HERONS FORMULA 141

2 144 64 cm 2 2 80 cm 2 8 5 cm 2 17.92 cm 2 Ans. Example 15. A floral design on a floor is made up of 16 tiles which are triangular, the sides of the triangle being 9 cm, 28 cm and 35 cm (see figure). Find the cost of polishing the tiles at the rate of 50 p per cm2. NCERT Solution. For each triangular tile, Let a = 9 cm, b = 28 cm, c = 35 cm

a b c 9 28 35 72 cm cm 36 cm 2 2 2
s ( s a )( s b )( s c )

Area of each triangular tile

36 (36 9)(36 28)(36 35) cm 2

36 4 2 3 9 cm 2 6 2 3 6 cm 2 36 6 cm 2

Total area of floral design 16 36 6 cm 2 Cost of polishing the 16 triangular tiles


1 16 36 6 Rs. 288 6 Rs. 705.45 Ans. 2 Example 16. Find the area of a trapezium ABCD in which AB || DC, AB = 77 cm, BC = 25 cm, CD = 60 cm and DA = 26 cm. Solution. Draw DE || BC and DF AB the, DE = BC= 25 cm. AE = AB EB = AB DC = 77 cm 60 cm = 17 cm. In DAE, Let a = 17 cm, b = 25 cm, c = 26 cm Rs. s a b c 17 25 26 cm 34 cm 2 2
s ( s a ) ( s b) ( s c)

Area of DAE

34 (34 17)(34 25)(34 26) cm 2

17 3 4 cm 2 204 cm 2
1 AE DF 2 1 17 DF cm 2 2 1 from (1) and (2), we get 17 DF 204 2 2 204 DF cm 24 cm 17 1 area of trapezium ABCD (AB DC) DF 2

...(1)

Also, area of DAE

...(2)

142

HERONS FORMULA

MATHEMATICSIX

1 (77 60) 24 cm 2 2 = 1644 cm2 Ans.

PRACTICE EXERCISE
1. Find the area of the triangle whose sides are : (i) 9 cm, 12 cm and 15 cm (ii) 150 cm, 120 cm and 2 m (iii) 6 m, 11 m and 15 m (iv) 26 cm, 28 cm, 30 cm 2. Find area of a equilateral triangle of side 16 cm. 3. An isosceles triangle has a perimeter 30 cm and each of the equal sides is 12 cm. Find the area of the triangle. 4. Calculate the area of the shaded portion, in the given figure.

5. The sides of a triangle are in the ratio 13:14:15 and its perimeter is 210 cm. Find its area. 6. If the area of an equilateral triangle is 49 3 cm2, find its perimeter. . 7. Two sides of a triangular field are 85 m and 154 m in length and its perimeter is 324 m. Find (i) The area of the field. (ii) the length of perpendicular from the opposite vertex on the side measuring 154 m. 8. ABC is a right triangle right angled at A with AB = 6 cm, AC = 8cm. A circle with centre O and radius r is inscribed in the triangle. Find r. 9. The perimeter of a triangular field is 240 m. If two of its sides are 78 m and 50 m, find the length of the perpendicular on the side of length 50 m from the opposite vertex. Calculate also the cost of watering it at Rs. 3.50 per 100 sq. m. 10. A triangle has sides 35 cm, 54 cm and 61 cm long. Find its area. Also, find the smallest altitude. 11. Find the area of a quadrilateral ABCD in which AB = 3 cm, BC = 4 cm, CD = 4 cm, DA = 5 cm and AC = 5 cm. 12. The area of a rhombus is 72 cm2. If one of the diagonals is 18 cm long, find the length of the other diagonal. 13. The diagonal of a four sided field is 40 m. The perpendiculars from the opposite vertices on this diagonal are 20 m and 14 m. Find the area of the field. 14. A field is in the shape of a trapezium whose parallel sides are 24 m and 52 m. The non-parallel sides are 26 m and 30 m. Find the area of the field. MATHEMATICSIX HERONS FORMULA 143

15. Find area of the following polygons :

(i)

(ii)

PRACTICE TEST
MM : 20 General Instructions : Each Question carry 4 marks. 1. The perimeter of a right triangle is 450 cm and its sides are in the ratio 13 : 12 : 5. Find the area of the triangle. 2. Find the area of a rhombus, one of whose sides is 25 cm and one of whose diagonal is 48 cm. 3. In a ABC, AB = 4 cm, BC = 9 cm and AC = 7 cm. Find the length of the perpendicular from A to BC. 4. A triangle and a parallelogram have the same base and the same area. If the sides of the triangle are 26 cm, 28 cm and 30 cm, and the parallelogram stands on the base 28 cm, find the height of the parallelogram. 5. A field is in the shape of a trapezium whose parallel sides are 55 m, 40 m and non-parallel sides are 20 m, 25 m. Find the area of the field. Time : 1 hour

ANSWERS OF PRACTICE EXERCISE


1. (i) 54 cm2 2.
64 3 cm 2

(ii) 8969 cm2 3. 9 15 cm 2 7. (i) 2772 m2 10. 939.14 cm2, 15.39 cm 14. 912 m2

(iii) 20 2 m 2 4. 384 cm2

(iv) 336 cm2 5. 2100 cm2

6. 42 cm 9. 67.2 cm, Rs. 58.80 13. 680 m2

(ii) 36 m 8. 2 cm 2 11. 15.2 cm 12. 8 cm 15. (i) 249.4 cm2 (ii) 444 m2

ANSWERS OF PRACTICE TEST


1. 9000 m2 2. 336 cm2 3.
4 5 cm 3

4. 12 cm

9. 950 m2

144

HERONS FORMULA

MATHEMATICSIX

CHAPTER

13
SURFACE AREAS AND VOLUMES

Points to Remember : 1. Cuboid (i) Volume = lbh (ii) Curved surface area = 2h (l + b) (iii) Total surface area = 2 (lb + bh + lh) (iv) Diagonal l 2 b 2 h 2

2. Cube (i) Volume = a3 (ii) Curved surface area = 4a2 (iii) Total surface area = 6a2 (iv) Diagonal 3 .a

3. Cylinder (i) Volume = r2h (ii) Curved surface area = 2rh (iii) Total surface area = 2r (r + h)

4. Cone (i) Volume


1 2 r h 3

(ii) slant height, l h 2 r 2 (iii) curved surface area r l (iv) Total surface area r (l r ) MATHEMATICSIX SURFACE AREAS AND VOLUMES 1

5. Sphere (i) Volume 4 r 3 3 (ii) Total surface area = 4 r2

6. Hemi-sphere (i) Volume 2 r 3 3 (ii) Curved surface area = 2r2 (iii) Total surface area = 3r2

ILLUSTRATIVE EXAMPLES
Example 1. A cubical box has each edge 10 cm and another cuboidal box is 12.5 cm long, 10 cm wide and 8 cm high. (i) Which box has the greater lateral surface area and by how much? (ii) Which box has the smaller surface area and by how much? NCERT Solution. (i) Lateral surface area (L1) of cubical box = 4 (edge)2 = 4 (10)2 cm2 = 400 cm2 and, lateral surface area (L2) of cuboidal box = 2 (length + breadth) height = 2 (12.5 + 10) 8 cm2 = 360 cm2 Clearly, L1 > L2. Now, L1 L2 = 400 cm2 360 cm2 = 40 cm2 The cubical box has larger lateral surface area and is greater by 40 cm2. (ii) Total surface area of the cubical box (S1) = 6 (edge)2 = 6 (10)2 cm2 = 600 cm2 Total surface area of the cuboidal box (S2) = 2 (lb + lh + bh) = 2 (12.5 10 + 10 8 + 8 12.5) cm2 = 2 (125 + 80 + 100) cm2 = 610 cm2 Clearly, S2 > S1. S2 S1 = 610 cm2 600 cm2 = 10 cm2 Thus, the cuboidal box has greater surface area and is greater by 10 cm2. Example 2. Two cubes each of 15 cm edge are joined end to end. Find the surface area of the resulting cuboid. Solution. here, l = length of resulting cuboid = 15 cm + 15 cm = 30 cm b = breadth of resulting cuboid = 15 cm h = height of resulting cuboid = 15 cm 2 SURFACE AREAS AND VOLUMES MATHEMATICSIX

Surface area of resulting cuboid = 2 (lb + bh + lh) = 2 (30 15 + 15 15 + 30 15) cm2 = 2 (450 + 225 + 450) cm2 = 2 (1125) cm2 = 2250 cm2 Ans. Example 3. A plastic box 1.5 m long, 1.25 m wide and 65 cm deep is to be made. It is to be open at the top. Ignoring the thickness of the plastic sheet, determine: (i) The area of the sheet required for making the box. (ii) The cost of sheet for it, if a sheet measuring 1 m2 costs Rs. 20. NCERT Solution. We have, Length; l = 1.5 m, Breadth, b = 1.25 m and Depth = Height, h = 0.65 m (i) Since the plastic box is open at the top, Plastic sheet required for making such box = 2 (l + b) h + lb = 2 (1.5 + 1.25) 0.65 m2 + 1.5 1.25 m2 = 2 2.75 0.65 m2 + 1.875 m2 = 3.575 m2 + 1.875 m2 = 5.45 m2 (ii) Cost of 1 m2 of sheet = Rs. 20 Total cost of 5.45 m2 of sheet = Rs. 5.45 20 = Rs. 109 Ans. Example 4. Parveen wanted to make a temporary shelter for her car, by making a box-like structure with tarpaulin that covers all the four sides and the top of the car (with the front face as a flap which can be rolled up). Assuming that the stitching margins are very small, and therefore negligible, how much tarpaulin would be required to make the shelter of height 2.5 m, with base dimensions 4 m 3 m? NCERT Solution. here, l = 4m, b = 3m, h = 2.5 m. Since there is no tarpaulin for the floor. Tarpaulin required = [2 (l + b) h + lb] = [2 (4 + 3) 2.5 + 4 3] m2 = (2 7 2.5 + 12) m2 = (35 + 12) m2 = 47 m2 Ans. Example 5. The sum of length, breadth and height of a cuboid is 21 cm and the length of its diagonal is 12 cm. Find the surface area of the cuboid. Solution. Let the length, breadth and height of the cuboid be l cm, b cm and h cm respectively. Then, l + b + h = 21 ...(1) Now, diagonal = 12 cm
l 2 b 2 h 2 12

l2 + b2 + h2 = 144 ...(2) Now, l + b + h = 21 Squaring both sides, we get (l + b + h)2 = (21)2 l2 + b2 + h2 + 2lb + 2bh + 2lh = 441 144 + 2 (lb + bh + lh) = 441 2 (lb + bh + lh) = 441 144 = 297 Surface area of the cuboid is 297 cm2 Ans. MATHEMATICSIX

( using (2))

SURFACE AREAS AND VOLUMES

Example 6. Aggarwal sweets stall was placing an order for making card board boxes for packing their sweets. Two size of boxes were required. The bigger of dimensions 25 cm 20 cm 5 cm and the smaller of dimensions 15 cm 12 cm 5 cm. 5% of the total surface area is required extra, for all the overlaps. If the cost of cardboard is Rs. 5 for 1000 cm2, find the cost of cardboard required for supplying 300 boxes of each kind. Solution. Surface area of Ist box = 2 (25 20 + 20 5 + 25 5) cm2 = 2 (500 + 100 + 125) cm2 = 1450 cm2 Surface area of IInd box = 2 (15 12 + 12 5 + 15 5) cm2 = 2 (180 + 60 + 75) cm2 = 630 cm2 Total combined surface area = 1450 cm2 + 630 cm2 = 2080 cm2 Area of overlaps = 5% of 2080 cm2 5 2080 cm 2 104 cm 2 100 Total surface area of 2 boxes (one of each kind) = (2080 + 104) cm2 = 2184 cm2 Surface area of 300 boxes of each kind = 300 2184 cm2 = 655200 cm2 Now, cost of cardboard for 1000 cm2 = Rs. 5 Cost of cardboard for 1 cm 3 Rs.
5 1000
5 655200 1000

Cost of cardboard for 655200 cm2 Rs.

= Rs. 3276 Ans. Example 7. A cylindrical pillar is 50 cm in diameter and 3.5 m in height. Find the cost of painting the curved surface of the pillar at the rate of Rs. 12.50 per m2. NCERT Solution. Diameter of cylindrical pillar = 50 cm radius (r)
50 cm 25 cm 0.25 m 2 also, height (h) = 3.5 m

Now, curved surface = 2rh 2 22 0.25 3.5 m 2 = 5.5 m2 7 Cost of painting 1 m2 = Rs. 12.50 Cost of painting 5.5 m2 = Rs. 12.50 5.5 = Rs. 68.75 Ans. Example 8. In the given figure, you see the frame of a lampshade. It is to be covered with a decorative cloth. The frame has a base diameter of 20 cm and height of 30 cm. A margin of 2.5 cm is to be given for folding it over the top and bottom of the frame. Find how much cloth is required for covering the lampshade. NCERT

SURFACE AREAS AND VOLUMES

MATHEMATICSIX

Solution.

Here, radius (r )

20 cm 10 cm 2 and, height (h) = 30 cm + 2 2.5 cm = 35 cm (for margin) Cloth required for covering the lampshade = Its curved surface area = 2 rh
2 22 10 35 cm 2 = 2200 cm2 Ans. 7

Example 9. It is required to make a closed cylindrical tank of height 120 cm and base diameter 140 cm from a metal sheet. How many square metres of the sheet is required for the same? Solution. We have, diameter of base = 140 cm.
140 cm 70 cm 2 and, height of cylinder = 120 cm. Total surface area of required tank = 2 r (r + h)

radius of base

22 70 (70 120) cm 2 7

83600 2 m 8.36 m 2 Ans. 10000 Example 10. The students of a Vidyalaya were asked to participate in a competition for making and decorating penholders in the shape of a cylinder with a base, using cardboard. Each penholder was to be of radius 3 cm and height 10.5 cm. The Vidyalaya was to supply the competitors with cardboard. If there were 35 competitors, how much cardboard was required to be bought for the competition? NCERT Solution. Cardboard required by each competitor = curved surface area of one penholder + base area = 2rh + r2, where r = 3 cm, h = 10.5 cm 440 190 cm 2 83600 cm 2

22 22 2 3 10.5 (3) 2 cm 2 7 7

= (198 + 28.28) cm2 = 226.28 cm2 (approx) Cardboard required for 35 competitors = 35 226.28 cm2 = 7920 cm2 (approx) Ans. Example 11. The diameter of a roller is 84 cm and its length is 100 cm. It takes 300 complete revolution to move once over to level a playground. Find the area of the playground in m2. Solution. Radius of roller (r )
84 cm 42 cm 2

Length of the cylindrical roller (h) = 100 cm. Area moved by the roller in one revolution
2 rh 2 22 42 100 cm 2 7

MATHEMATICSIX

SURFACE AREAS AND VOLUMES

Area moved in 300 revolutions


2

22 42 100 300 cm 2 7 = 44 180000 cm2 = 44 18 m2 = 792 m2 Ans. Example 12. Find (i) the curved surface area of a cylindrical petrol storage tank that is 4.2 m in diameter and

4.5 m high. (ii) how much steel was actually used if making the closed tank? Solution.
4.2 m 2.1 m , h 4.5 m 2 (i) Curved surface area = 2 r h

1 of the steel actually used was wasted in 12 NCERT

Here, r

22 2.1 4.5 m 2 7 = 59.4 m2 (ii) Total surface area of closed tank 2

2 r h 2 r 2 2r (r h)
22 2.1 ( 2.1 4.5) m 2 7 = 87.12 m2 2

Let the total sheet used for making the cylindrical tank be x m2. Given, wastage according to given question, x
x 87.12 12

x 2 m . 12

87.12 12 11 x 87.12 x 95.04 m 2 11 12 Steel used for making closed tank including wastage = 95.04 m2 Ans. Example 13. Curved surface area of a cone is 308 cm2 and its slant height is 14 cm. Find (i) radius of the base and (ii) total surface area of the cone. NCERT Solution. (i) Here, r l = 308 cm2, l = 14 cm.

22 308 14 r 308 r 7 cm. 7 22 2 and, (ii) Total surface area = r (r + l)

22 7 (7 14) cm 2 7 = 22 21 cm2 = 462 cm2 Example 14. How many meters of cloth, 5 m wide, will be required to make a conical tent, the radius of whose base is 7 m and height is 24 m? Solution. Radius of the tent, r = 7 m height of the tent, h = 24 m

slant height, l r 2 h 2 7 2 24 2 m 625 m 25 m SURFACE AREAS AND VOLUMES MATHEMATICSIX

curved surface area = r l

22 7 25 m 2 550 m 2 7 i.e., area of the cloth = 550 m2

Now, length of cloth required area 550 m 110 m width 5 length of cloth required = 110 m. Example 15. A conical tent is 10 m high and the radius of its base is 24 m. Find : (i) slant height of the tent. (ii) cost of the canvas required to make the tent, if the cost of 1 m2 canvas is Rs. 70. Solution. (i) here, r = 24 m, h = 10 m Let l be the slant height of the cone. then, l2 = h2 + r2 l h 2 r 2 24 2 10 2
576 100 676 26 m.

NCERT

(ii) Canvas required to make the conical tent = curved surface of the cone
rl 22 24 26 cm 2 7

Now, Rate of canvas for 1 m2 = Rs. 70 Total cost of canvas Rs. 22 24 26 70 = Rs. 137280 Ans. 7 Example 16. A jokers cap is in the form of a right circular cone of base radius 7 cm and height 24 cm. Find the area of the sheet required to make 10 such caps. NCERT Solution. here, radius of cap (r) = 7 cm height of cap (h) = 24 cm Let l be the slant height. then, l h 2 r 2 24 2 7 2 576 49 625 25 cm Sheet required for one cap = curved surface of the cone = rl
22 7 25 cm 2 550 cm 2 7 Sheet required for 10 such caps = 10 550 cm2 = 5500 cm2. Ans. Example 17. A bus stop in barricaded from the remaining part of the road, by using 50 hollow cones made of recycled cardboard. Each cone has a base diameter of 40 cm and height 1 m. If the outer side of each of the cones is to be painted and the cost of painting is Rs. 12 per m2, what will be the cost

of painting all these cones? (Use = 3.14 and 1.04 1.02 ). Solution. here, radius (r )
40 cm 20 cm 0.2 m and height (h) = 1 m 2

NCERT

slant height (l ) r 2 h 2 0.04 1 1.04 1.02 m MATHEMATICSIX SURFACE AREAS AND VOLUMES 7

Now, curved surface of 1 cone = rl

2 0.2 1.02 m 2 7

Curved surface of 50 such cones 50 22 0.2 1.02 m 2 7 Now, cost of painting 1 m2 = Rs. 12 Total cost of painting Rs. 12 50
22 0.2 1.02 = Rs. 384.68 (approx) Ans. 7

Example 18. A corn cob (see figure), shaped somewhat like a cone, has the radius of its broadest end as 2.1 cm and length as 20 cm. If each 1 cm2 of the surface of the cob carries an average of four grains, find how many grains you would find on entire cob? (NCERT)

Solution.

We have, r = 2.1 cm, h = 20 cm let, slant height be l cm. then, l r 2 h 2 (2.1) 2 (20) 2 cm 4.41 400 cm
404.41 cm 20.11 cm curved surface area of corn cob = r l

22 2.1 20.11 cm 2 7 = 132.726 cm2 Now, number of grains on 1 cm2 = 4 number of grains on 132.726 cm2 = 4 132.726 = 530.904 531 Hence, total number of grains on the corn cob = 531 Ans. Example 19. The surface area of a sphere is 154 cm2. Find its radius. Solution. Let the radius of the sphere be r cm. then, 4 r2 = 154 (given)

NCERT

r2

154 154 7 49 4 4 22 4

r
8

49 7 cm cm 4 2
7 cm. 2

radius of the sphere

SURFACE AREAS AND VOLUMES

MATHEMATICSIX

Example 20. The radius of a spherical baloon increases from 7 cm to 14 cm as air is being pumped into it. Find the ratio of surface areas of the balloon in the two cases. NCERT Solution. Let S1 and S2 be the total surface area in two cases of r = 7 cm and R = 14 cm. S1 = 4 r2 = 4 (7)2 cm2 and S2 = 4 R2 = 4 (14)2 cm2 Required ratio
S1 4 7 7 1 i.e. 1 : 4 Ans. S 2 4 14 14 4

Example 21. The diameter of the moon is approximately one fourth of the diameter of the earth. Find the ratio of their surface areas. NCERT Solution. Let the diameter of earth be R and that of the moon will be The radii of moon and earth are
R R and respectively. . 8 2
2

R 4

R 1 4 8 Ratio of their surface area 64 1 4 1 i .e. 1 : 16 Ans. 2 1 64 1 16 R 4 4 2 Example 22. A hemispherical bowl is made of steel, 0.25 cm thick. The inner radius of the bowl is 5 cm. Find the outer curved surface area of bowl. NCERT Solution. Inner radius, r = 5 cm Thickness of Steel = 0.25 cm Outer radius, R = (r + 0.25) cm = (5 + 0.25) cm = 5.25 cm
22 Outer curved surface 2 r 2 2 5.25 5.25 cm 2 7

= 173.25 cm2 Example 23. The internal and external diameters of a hollow hemispherical vessel are 20 cm and 28 cm respectively. Find the cost of painting the vessel all over at 15 paisa per cm2. Solution. Outer radius of vessel, R = 14 cm Inner radius of vessel, r = 10 cm Area of the outer surface = 2 R2 = 2 (14)2 cm2 = 392 cm2 Area of the inner surface = 2 r2 = 2 (10)2 cm2 = 200 cm2 Area of the ring at the top = (R2 r2) = (142 102) cm2 = (14 10) (14 + 10) cm2 = 96 cm2 Total area to be painted =3 92 cm2 + 200 cm2 + 96 cm2 = 688 cm2 Now, cost of painting 1 cm2 = 15 paisa 15 Rs. 100 MATHEMATICSIX SURFACE AREAS AND VOLUMES 9

15 688 Rs 100 15 22 688 Rs 100 7 = 324.34 Rs. Ans. Example 24. A cuboidal water tank is 6 m long, 5 m wide and 4.5 m deep. How many litres of water can it hold? (1 m3 = 1000 l). NCERT Solution. Here, l = 6m, b = 5m and h = 4.5 m Volume of the tank = lbh = (6 5 4.5) m3 = 135 m3 The tank can hold = 135 1000 litres = 135000 litres of water. ( 1 m3 = 1000 litres) Example 25. The capacity of a cuboidal tank is 50000 litres of water. Find the breadth of the tank, if its length and depth are respectively 2.5 m and 10 m. NCERT Solution. Given capacity of a cuboidal tank 50000 3 50000 l m 50 m 3 1000 Let the breadth of cuboidal tank be b m. according to given question, we have 2.5 b 10 = 50 50 25 b 50 b b2 25 breadth of the tank is 2m. Ans. Example 26. A river 3 m deep and 40 m wide is flowing at the rate of 2 km per hour. How much water will fall into the sea in a minute? NCERT Solution. Volume of water that flows in 1 hour (60 minutes) = volume of water of a cuboid whose dimensions are 3 m, 40 m and 2000 m. ( 2 km = 2000 m) = 3 40 2000 m3 Volume of water that flows in 1 minute

Cost of painting 688 cm 2

3 40 2000 3 m 4000 m 3 Ans. 60 Example 27. Three cubes whose edges are 3 cm, 4 cm and 5 cm respectively are melted and recast into a single cube. find the surface area of the new cube. Solution. Let x cm be the edge of new cube. Then, volume of the new cube = sum of the volumes of three cubes. x3 = 33 + 43 + 53 = 27 + 64 + 125 = 216 = (6)3 x = 6 cm. Edge of the new cube is 6 cm. and, surface area of the new cube = 6 (6)2 cm2 = 216 cm2 Ans. Example 28. A village, having a population of 4000, requires 150 litres of water per head per day. It has a tank measuring 20 m 15 m 6 m. For how many days will the water of this tank last? NCERT Solution. Here l = 20 m, b = 15 m, and h = 6 m Capacity of the tank = lbh = (20 15 6) m3 = 1800 m3 Water requirement per person per day = 150 litres

10

SURFACE AREAS AND VOLUMES

MATHEMATICSIX

Water required for 4000 person per day = (4000 150) l


4000 150 3 3 m 600 m 1000

Number of days the water will last


Capacity of tank Total water required per day
1800 30 600

Thus, the water will last for 30 days Ans. Example 29. A godown measures 60 m 25 m 10 m. Find the maximum number of wooden crates each measuring 1.5 m 1.25 m 0.5 m that can be stored in the godown. NCERT 3 3 Solution. Volume of the godown = (60 25 10) m = 15000 m Volume of 1 crate = (1.5 1.25 0.5) m3 = 0.9375 m3 Number of crate that can be stored in the godown
Volume of the godown Volume of 1 crate
15000 16000 Ans. 0.9375

Example 30. If the lateral surface of cylinder is 94.2 cm2 and its height is 5 cm, then find (i) radius of its base (ii) its volume (use = 3.14) . NCERT Solution. (i) Let r be the radius of the base and h be the height of the cylinder. Then, Lateral surface = 94.2 cm2 2rh = 94.2 2 3.14 r 5 = 94.2
r 94.2 3 2 3.14 5

Thus, the radius of its base = 3 cm. (ii) Volume of the cylinder = r2h = (3.14 32 5) cm3 = 141.3 cm3 Ans. Example 31. It costs Rs. 2200 to paint the inner curved surface of a cylindrical vessel 10 m deep. If the cost of painting is at the rate of Rs. 20 per m2, Find : (i) inner curved surface area of the vessel (ii) radius of the base (iii) capacity of the vessel NCERT Solution. (i) inner curved surface area of the vessel
2200 2 2 m 110 m 20

Total cost of painting Rate of painting

MATHEMATICSIX

SURFACE AREAS AND VOLUMES

11

(ii) Let r be the radius of the base and h be the height of the cylindrical vessel. 2rh = 110
2 22 r 10 110 7 110 7 7 1.75 2 22 10 4

Thus, the radius of the base = 1.75 m


22 7 7 (iii) Capacity of the vessel = r2h 10 m 3 7 4 4

= 96.25 m3 Example 32. The capacity of a closed cylindrical vessel of height 1 m is 15.4 litres. How many square metres of metal sheet would be needed to make it? NCERT Solution. Capacity of a closed cylindrical vessel = 15.4 litres
1 3 3 15.4 m 0.0154 m 1000

Let r be the radius of the base and h be the height of the vessel. Then, Volume = r2h = r2 1 = r2 ( h = 1m) r2 = 0.0154
22 2 r 0.0154 7

r2

0.0154 7 0.0049 22

r 0.0049 0.07

Thus, the radius of the base of vessel = 0.07 m. Metal sheet needed to make the vessel = Total surface area of the vessel = 2rh + 2r2 = 2r (h + r)
2 22 0.07 (1 0.07) m 2 7

= 44 0.01 1.07 m2 = 0.4708 m2 Example 33. A patient in a hospital in given soup daily in a cylindrical bowl of diameter 7 cm. If the bowl is filled with soup to a height of 4 cm, how much soup the hospital has to prepare daily to serve 250 patients? NCERT Solution. Diameter of the cylindrical bowl = 7 cm
7 cm 2 Height of serving bowl = 4 cm.

Radius

12

SURFACE AREAS AND VOLUMES

MATHEMATICSIX

Soup served to 1 patient = Volume of the bowl = r2h


22 7 7 4 cm 3 = 154 cm3 7 2 2 Soup served to 250 patients = (250 154) cm3 = 38500 cm3 = 38.5 l. Ans.

Example 34. The inner diameter of a cylindrical wooden pipe is 24 cm and its outer diameter is 28 cm. The length of the pipe is 35 cm. Find the mass of the pipe, if 1 cm3 of wood has a mass of 0.6 gm. NCERT Solution. here, inner radius (r ) and outer radius (R) =
24 cm 12 cm 2

28 cm 14 cm 2 h = length of the pipe = 35 cm. volume of wood used in making the pipe

R 2h r 2h h (R 2 r 2 )

22 35 [14 2 12 2 ] cm 3 7
22 35 (14 12) (14 12) cm 3 7

22 35 2 26 cm 3 5720 cm 3 7 Now, 1 cm3 of wood = 0.6 g 5720 cm3 of wood = 0.6 5720 g = 3432.0 g = 3.432 kg. Ans. Example 35. A lead pencil consists of a cylinder of wood with a solid cylinder of graphite filled in the interior. The diameter of the pencil is 7 mm and the diameter of the graphite is 1 mm. If the length of the pencil is 10 cm, find the weight of the whole pencil if the specific gravity of the wood is 0.7 gm/cm3 and that of the graphite is 2.1 gm/cm3. NCERT Solution. For graphite cylindrical rod :

radius (r) of graphite cylinder 1 1 cm 1 cm 2 10 20 and, length of graphite rod (h) = 10 cm. volume of graphite cylindrical rod = r2 h
22 1 10 cm 3 7 20
2

Weight of graphite used for pencil = volume specific gravity


22 1 1 10 2.1 gm 7 20 20

( 1 cm3 2.1 gm)

= 0.165 gm. MATHEMATICSIX SURFACE AREAS AND VOLUMES 13

Again, for pencil including graphite rod, we have, radius of pencil (R)
7 7 mm cm 2 20 and, length of pencil (h) = 10 cm

volume of pencil = R 2 h
22 7 10 cm 3 7 20
2

volume of wood used for pencil

R2 h r 2 h
22 7 7 22 1 1 1 10 cm 3 cm 3 7 20 20 7 20 20 10 11 1 48 cm 3 7 20

11 1 48 0.7 gm ( 1 cm3 0.7 gm) 7 20 = 2.64 gm. Total weight = 2.64 gm + 0.165 gm = 2.805 gm Ans. Example 36. A well of diameter 3m is dug 14 m deep. The earth taken out of it has been spread evenly all around it to a width of 4 m to form an embankment. Find the height of the embankment.

weight of wood

Solution.

3 m , height (h) = 14 m 2 Volume of the earth taken out of the well

Radius of the well (r )

22 3 3 14 m 3 99 m 3 7 2 2 Outer radius of the embankment r2 h

11 3 m4 m m 2 2 Area of embankment = outer area inner area R

R 2 r 2

2 2 22 11 3 22 11 3 11 3 7 2 2 7 2 2 2 2

22 7 4 88 m 2 7 Height of the embankment


Volume 99 9 m m 1.125 m Ans. Area 88 8

14

SURFACE AREAS AND VOLUMES

MATHEMATICSIX

Example 37. The height of a cone is 15 cm. If its volume is 1570 cm3. Find the radius of the base (Use = 3.14). NCERT Solution. Here h = 15 cm and volume = 1570 cm3 Let the radius of the base of the cone be r cm. Now, Volume = 1570 cm3
1 2 r h 1570 3 1 3.14 r 2 15 1570 3 1570 r2 100 3.14 5

r 100 10 Thus, the radius of the base of cone is 10 cm Ans. Example 38. The radius and height of a right circular cone are in the ratio of 5 : 12. If its volume is 2512 cm3, find the slant height and radius of the base of the cone. (use = 3.14) Solution. Let the radius of cone be 5x and height be 12 x. Volume of the cone 1 r 2 h 3 according to question, we get
1 3.14 (5 x ) 2 (12 x) 2512 3

1 314 25 x 2 12 x 2512 3 100

2512 8 x 2 314 radius of base = 5x = 5 2 cm = 10 cm and, height = 12 x = 12 2 cm = 24 cm x3

slant height r 2 h 2 10 2 24 2 100 576 676 26 radius of cone = 10 cm and slant height = 26 cm Ans. Example 39. The volume of a right circular cone is 9856 cm3. If the diameter of the base is 28 cm, find (i) height of the cone (ii) slant height of the cone (iii) curved surface area of the cone NCERT Solution. Given, volume of cone = 9856 cm3 and, radius of base (r)
28 cm 14 cm 2

(i) we know, volume of cone 1 r 2 h 3 1 22 14 14 h 9856 3 7 MATHEMATICSIX SURFACE AREAS AND VOLUMES 15

3 9856 cm 48 cm. 22 2 14
h2 r 2

(ii) Slant height of a cone (l)

(48) 2 (14) 2 cm

2304 196 cm 2500 cm 50 cm (iii) Curved surface area of a cone =rl


22 14 50 cm 2 2200 cm 2 . 7 Example 40. A heap of wheat is in the form of a cone whose diameter is 10.5 m and height is 3 m. Find its volume. The heap is to be covered by canvas to protect it from rain. Find the area of the canvas required. NCERT

Solution.

Diameter of the base of the cone = 10.5 m


10.5 radius r m 5.25 m 2 Height of the cone = 3m

Volume of the cone To find the slant height l

1 22 3 1 2 r h 5.25 5.25 3 m = 86.625 m3 3 7 3

We have, l 2 h 2 r 2 32 (5.25)2 = 9 + 27.5625 = 36.5625


l 36.5625 6.0467 m (approx.)

Canvas required to protect wheat from rain = Curved surface area


22 rl 5.25 6.0467 m 2 = 99.77 m2 (approx) 7

Example 41. A right triangle ABC with sides 5 cm, 12 cm and 13 cm is revolved about the side 12 cm. Find the volume of the solid so obtained? Solution. The solid obtained is a cone with r = 5 cm and h = 12 cm. Volume 1 r 2 h 3
1 3.14 5 5 12 cm 3 3

100 3.14 cm3


= 314 cm3 Ans. 16 SURFACE AREAS AND VOLUMES MATHEMATICSIX

Example 42. If the surface area of a sphere is 616 cm2, find its volume. Solution. Let r be the radius of sphere. then, 4 r2 = 616
r2 616 616 7 49 4 4 22

r 49 cm 7 cm

volume of sphere 4 r 3 4 22 7 7 7 cm 3 3 3 7 = 1437.3 cm3 Ans. Example 43. The diameter of the moon is approximately one-fourth the diameter of the earth. What fraction of the volume of the earth is the volume of the moon? NCERT Solution. Let the radius of earth be R. then, radius of moon volume of earth
R . 4
4 R3 3

and, volume of moon

4 R . 3 4

4 R3 volume of earth 64 3 volume of moon 4 R 3 1 3 4

i.e., volume of earth is 64 times the volume of the moon.


1 times that of earth. 64 Example 44. The diameter of a metallic ball is 4.2 cm. What is the mass of the ball, if the density of the metal is 8.9 g per cm3? NCERT Solution. Diameter of the ball = 4.2 cm

i.e., volume of moon is

4.2 Radius cm 2.1 cm 2

Volume of the ball 4 r 3 3


4 22 2.1 2.1 2.1 cm 3 38.808 cm 3 3 7

Now, Density of metal = 8.9 gm per cm3 Mass of the ball = 38.808 8.9 g = 345.3912 g = 345.4 g (approx) Ans. MATHEMATICSIX SURFACE AREAS AND VOLUMES 17

Example 45. A hemispherical tank is made up of an iron sheet 1 cm thick. If the inner radius is 1m, then find the volume of the iron used to make the tank. NCERT Solution. Let R cm and r cm be respectively the external and internal radii of the hemispherical vessel, then R = 1.01 m and r = 1 m ( as thickness = 1cm = 0.01 m) Volume of iron used = External volume Internal volume
2 3 2 3 2 R r ( R 3 r 3 ) 3 3 3

2 22 [(1.01)3 (1)3 ] m 3 3 7

44 44 (1.030301 1) m 3 0.030301 m 3 21 21 = 0.06348 m3 (approx) Example 46. A dome of a building is in the form of hemi-sphere. From inside, it was white-washed at the cost of Rs. 498.96. If the cost of white-washing is Rs. 2 per square meter, find: (i) the inside surface area of the dome (ii) the volume of the air inside the dome. NCERT Solution. Let r be the inner radius of the hemispherical dome. Then, inside surface area of the hemisphere = 2 r2. Since, at the rate of Rs. 2 per square metre, the total cost of white-wash is Rs. 498.96,

surface area of hemisphere

498.96 2 m 249.48 m 2 2

according to question, 2r 2 249.48


r2 249.48 7 39.69 2 22

r 39.69 m 6.3 m (i) Inside surface are of the dome = 2 r2 = 249.48 m2 (ii) Inside volume of the dome

(calculated above)

2 3 2 22 r (6.3) 3 m 3 3 3 7 = 523.908 m3 Ans. Example 47. Twenty seven solid iron spheres, each of radius r and surface area S are melted to form a sphere with surface are S. Find the: (i) radius r of the new sphere. (ii) ratio of S and S. NCERT

Solution.

18

4 (i) Volume of 27 solid spheres of radius r 27 r 3 3 4 3 Volume of the new sphere of radius r r 3 SURFACE AREAS AND VOLUMES

...(1) ...(2)

MATHEMATICSIX

According to the problem, we have,


4 3 4 r 27 r 3 3 3

r3 27r 3 (3r )3 r 3r
S 4r 2 r2 r2 1 2 1: 9 2 2 S 4r 9 (3r ) 9r

(ii) Required ratio

Example 48. A wooden bookshelf has external dimensions as follows : Height = 110 cm, Depth = 25 cm, Breadth = 85 cm (see figure). The thickness of the plank is 5 cm everywhere. The external faces are to be polished and the inner faces are to be painted. If the rate of polishing is 20 paisa per cm2 and the rate of painting is 10 paisa per cm2, find the total expenses required for polishing and painting the surface of the bookshelf. NCERT

Solution.

Area to be polished = (110 85 + 2 85 25 + 2 25 110 + 4 75 5 + 2 110 5) cm2 = (9350 + 4250 + 5500 + 1500 + 1100) cm2 = 21700 cm2 cost of polishing @ 20 paisa per cm2 = 21700
20 Rs. 4340 100

Also, Area to be painted = (6 75 20 + 2 90 20 + 75 90) cm2 = (9000 + 3600 + 6750) cm2 = 19350 cm2 Cost of painting @ 10 paisa per cm2
19350 10 Rs. Rs. 1935 100

Total expense = Rs. 4340 + Rs. 1935 = Rs. 6275 Ans. Example 49. The front compound wall of a house is decorated by wooden spheres of diameter 21 cm, placed on small supports as shown in figure. Eight such spheres are used for this purpose, and are to be painted silver. Each support is a cylinder of radius 1.5 cm and height 7 cm and is to be painted black. Find the cost of paint required if silver paint costs 25 paisa per cm2 and black paint costs 5 paisa per cm2. NCERT MATHEMATICSIX SURFACE AREAS AND VOLUMES 19

Solution.

Clearly, we have to subtract the area of the circle on which sphere is resting while calculating the cost of silver paint. Surface area to be painted silver = 8 (curved surface area of the sphere area of circle on which sphere is resting) = 8 (4 R2 r2) where R
21 cm , r 1.5 cm 2

441 8 4 2.25 cm 2 8 (441 2.25) cm 2 8 (438.75) cm 2 4

Cost of silver paint @ 25 paisa per cm2


25 22 Rs. 8 438.75 Rs. 2757.86 (approx) 100 7

Again, surface area to be painted black = 8 curved surface area of cylinder = 8 2rh = 8 2
22 1.5 7 cm 2 528 cm 2 7

Cost of black paint @ 5 paisa per cm2


Rs. 528 5 Rs. 26.40 100

Total cost of painting = Rs. 2757.86 + Rs. 26.40 = Rs. 2784.26 (approx) Ans. Example 50. The diameter of a sphere is decreased by 25%. By what percent does its curved surface decrease? NCERT Solution.
d Let d be the diameter of the sphere. Then, its surface area 4 d 2 2 On decreasing its diameter by 25%,
2

New diameter d1 d 25% of d d 25 d 75 d 3 d 100 100 4 20 SURFACE AREAS AND VOLUMES MATHEMATICSIX

d 1 3d New surface area 4 1 4 2 4 2

4.

9d 2 9 . d 2 64 16

Decrease in surface area


d 2 9 9 7 d 2 1 d 2 d 2 16 16 16

Percentage decrease in surface area


decrease in surface area 100% original surface area

7 d 2 7 16 2 100% 100% = 43.75% Ans. 16 d

PRACTICE EXERCISE
Questions based on Surface area of cuboid and cube 1. Find the surface area of a cuboid 16 m long, 14 m broad and 7 m high. 2. Find the length of the longest pole that can be placed in a room 12 m long, 8 m broad and 9 m high. 3. The length, breadth and height of a room are 5 m, 4 m and 3 m respectively. Find the cost of whitewashing the walls of the room and the ceiling at the rate of Rs. 8.50 per m2. 4. Find the percentage increase in the surface area of a cube when each side is doubled. 5. The paint in a certain container is sufficient to paint an area equal to 9.375 m 2. How many bricks of dimensions 22.5 cm 10 cm 7.5 cm can be painted out of this container? NCERT 6. A small indoor green house (herbarium) is made entirely of glass panes (including base) held together with tape. It is 30 cm long, 25 cm wide and 25 cm high. (i) What is the area of the glass? (ii) How much of tape is needed for all the 12 edges? NCERT 7. Three cubes each of side 6 cm are joined end to end. Find the surface of the resulting cuboid. 8. A plastic box 1.5 m long, 1.25 m wide and 65 cm deep is to be made. It is to be open at top. Ignoring the thickness of the plastic sheet, find: (i) area of the sheet required to make the box. (ii) the cost of the sheet for it, if a sheet measuring 1 m2 cost Rs. 22. 9. If the surface area of the cube is 96 cm2, find its edge and length of its diagonal. 10. The dimensions of a rectangular box are in the ratio of 2 : 3 : 4 and the difference between the cost of covering it with sheet of paper at the rate of Rs. 4 and Rs. 4.50 per m2 is Rs. 650. Find the dimensions of the box. 11. Mary wants to decorate her christmas tree. She wants to place the tree on a wooden box covered with coloured paper with picture of Santaclaus on it (see figure). She must know the exact quantity of paper to buy for this purpose. If the box has length, breadth and height as 80 cm, 40 cm and 20 cm respectively, how many square sheets of paper of side 40 cm would she require? NCERT MATHEMATICSIX SURFACE AREAS AND VOLUMES 21

12. The length and breadth of a hall are in the ratio 4 : 3 and its height is 5.5 meters. The cost of decorating its walls (including doors and windows) at Rs. 6.60 per m2 is Rs. 5082. Find the length and breadth of the room. Questions based on Surface area of a cylinder 13. The curved surface area of a right circular cylinder of height 14 cm is 176 cm2. Find the diameter of the base of the cylinder. 14. A metal pipe is 77 cm long. The inner diameter of a cross-section is 4 cm, the outer diameter being 4.4 cm. (see figure). Find its : NCERT

15.

16. 17. 18. 19.

(i) inner curved surface area (ii) outer curved surface area (iii) Total surface area The inner diameter of a circular well is 3.5 m. It is 10 m deep. Find: (i) its inner curved surface area, (ii) the cost of plastering this curved surface at the rate of Rs. 40 per m2. An iron pipe 20 cm long has exterior diameter 25 cm. If the thickness of the pipe is 1 cm, find the total surface area of the pipe. A rectangular sheet of paper 88 cm 50 cm is rolled along its length and a cylinder is formed. Find curved surface area of the cylinder formed. A solid cylinder has total surface area of 462 cm2. Its curved surface area is one-third of its total surface area. Find the radius and height of the cylinder. The total surface area of a hollow metal cylinder, open at both the ends of external radius 8 cm and height 10 cm is 338 cm2. Find thickness of the metal in the cylinder. SURFACE AREAS AND VOLUMES MATHEMATICSIX

22

20. In the given figure, you see the frame of a lamp shade. It is to be covered with a decorative cloth. The frame has a base diameter of 20 cm and height of 30 cm. A margin of 2.5 cm is to be given for folding it over the top and bottom of the frame. Find how much cloth is required for covering the lampshade. NCERT

Question based on surface area of a Cone 21. Find the curved surface area of a cone, if its slant height is 50 cm and the diameter of its base is 28 cm. 22. Find the total surface area of a cone, if its slant height is 21 cm and diameter of its base is 24 cm. NCERT
22 23. The curved surface area of a cone is 4070 cm2 and its radius is 35 cm. What is its slant height? use 7

24. The radius and slant height of a cone are in the ratio 4 : 7. If its curved surface area is 792 cm2, find its
22 radius. use 7

25. The circumference of the base of a 10 m high conical tent is 44 m. Calculate the length of canvas used in making the tent if width of canvas is 2 m. use 22 7 26. What length of tarpaulin 3 m wide will be required to make conical tent of height 8 m and base radius 6 m? Assume that the extra length of material that will be required for stitching margins and wastage in cutting is approximately 20 cm. (use = 3.14) NCERT 27. The slant height and base diameter of a conical tomb are 25 m and 14 m respectively. Find the cost of white-washing its curved surface at the rate of Rs. 210 per 100 m2? NCERT 2 28. The curved surface area of a cone of radius 6 cm is 188.4 cm . Find its height. Questions based on surface area of sphere and hemi-sphere 29. The surface area of a sphere is 5544 cm2. Find its radius. 30. A hemi-spherical bowl made of brass has inner diameter 10.5 cm. Find the cost of tin-plating it on the inside at the rate of Rs. 16 per 100 cm2. NCERT MATHEMATICSIX SURFACE AREAS AND VOLUMES

23

31. The diameter of the moon is approximately one fourth of the diameter of the earth. Find the ratio of their surface areas. NCERT 32. In the given figure, a right cylinder just encloses a sphere of radius r. Find

(i) Surface area of the sphere (ii) curved surface area of the cylinder (iii) ratio of the areas obtained in (i) and (ii). 33. The internal and external diameters of a hollow hemi-spherical vessel are 20 cm and 28 cm respectively. Find the cost of painting the vessel all over at 15 paisa per cm2. 34. A toy is in the form of a cone mounted on a hemi-sphere. The diameter of the base and the height of the cone are 6 cm and 4 cm respectively. Find the surface area of the toy. (use = 3.14) Question based on Volume of Cuboid and Cube 35. The total surface area of a cube is 1350 cm2. Find its volume. 36. 500 persons took dip in a rectangular tank which is 80 m long and 50 m broad. What is the rise in the level of water in the tank, if the average displacement of water by a person is 4 m3? 37. Three cubes of a metal with edges 6 cm, 8 cm and 10 cm respectively are melted and formed into a single cube. Find the edge of the new cube formed. Also, find its volume. 38. The volume of a cuboid is 1536 m3. Its length is 16 m, and its breadth and height are in the ratio 3 : 2. Find surface area of the cuboid. 39. A field is 70 m long and 40 m broad. In one corner of the field, a pit which is 10 m long, 8 m broad and 5 m deep, has been dug out. The earth taken out of it is evenly spread over the remaining part of the field. Find the rise in the level of the field. 40. The areas of three adjacent faces of a cuboid are 15 cm2, 40 cm2 and 24 cm2. Find the volume of the cuboid. 41. How many bricks, each measuring 25 cm 15 cm 8 cm will be required to build a wall 10 m 4 dm 5 m when one-tenth of its volume is occupied by mortar? 42. A rectangular reservoir is 120 m long and 75 m wide. At what speed per hour must water flow into it through a square pipe of 20 cm wide so that the water rises by 2.4 m in 18 hours. Question based on Volume of a Cylinder 43. The radius of a cylinder is 14 cm and its height is 40 cm. Find (i) curved surface area (ii) the total surface area (iii) volume of the cylinder. 44. The total surface area of a cylinder is 462 cm2. Its curved surface is one-third of its total surface area. Find the volume of the cylinder. 45. The curved surface area and the volume of a pillar are 264 m2 and 396 m3 respectively. Find the diameter and the height of the pillar. 24 SURFACE AREAS AND VOLUMES MATHEMATICSIX

46. The sum of the height and radius of the base of a solid cylinder is 37 m. If the total surface area of the cylinder is 1628 m2, find its volume. 47. A cylindrical tube, open at both ends, is made up of metal. The internal radius of the tube is 5.2 cm and its length is 25 cm. The thickness of the metal is 8 mm. Calculate the volume of the metal. 48. A soft drink is available in two packs : (i) a tin can with a rectangular base of length 5 cm and width 4 cm, having a height of 15 cm and (ii) a plastic cylinder with circular base of diameter 7 cm and height 10 cm. Which container has greater capacity and by how much? NCERT 49. If the diameter of the cross-section of a wire is decreased by 5%, how much percent will the length be increased so that the volume remains the same? NCERT 50. Water flows out through a circular pipe, whose internal diameter is 2 cm, at the rate of 70 cm per second into a cylindrical tank, the radius of whose base is 40 cm. By how much time will the level of water rise in half an hour? 51. A rectangular piece of paper is 22 cm long and 12 cm wide. A cylinder is formed by rolling the paper along its length. Find the volume of the cylinder. 52. A well, with inner radius 4m, is dug 14 m deep. The earth taken out of it has been spread evenly all round
22 it to a width of 3m to form an embankement. Find the height of this embankement. use 7

Question based on Volume of a Cone 53. The base radii of two cones of the same height are in the ratio 3 : 4. Find the ratio of their volumes.
22 54. A cone of height 24 cm has curved surface area 550 cm2. Find its volume. use 7

55. The radius and height of a right circular cone are in the ratio of 5 : 12. If its volume is 314 cm3, find the slant height and radius of the base of the cone. (use = 31.4) 56. Find the slant height and curved surface area of a cone whose volume is 12935 cm3 and the radius of the base is 21 cm. 57. A semi-circular thin sheet of metal of diameter 28 cm is bent and an open conical cup is made. Find the capacity of the cup. 58. Monica has a piece of canvas whose area is 551 m2. She uses it to have a conical tent made with a base radius of 7 m. Assuming that all the stitching margins and the wastage incurred while cutting, amounts to approximately 1 m2, find the volume of the tent that can be made with it. NCERT 59. Find the volume of the largest right circular cone that can be cut out of a cube whose edge is 9 cm. 60. A right angled triangle of which the sides containing the right angle are 6.3 cm and 10 cm in length, is made to turn round on the longer side. Find the volume of the solid, thus generated. Also, find its curved surface area. Question based on Volume of a Sphere and Hemi-Sphere 61. Find the surface area of a sphere whose volume is 606.375 m3. 62. A solid sphere of radius 3 cm is melted and then cast into smaller spherical balls, each of diameter 0.6 cm. Find the number of small balls thus obtained. 63. The surface areas of two spheres are in the ratio 1 : 4. Find the ratio of their volumes. 64. The diameter of a metallic sphere is 6 cm. It is melted and drawn into a wire having diameter of the crosssection as 2 mm. Find the length of the wire. MATHEMATICSIX SURFACE AREAS AND VOLUMES 25

65. The radii of the internal and external surfaces of a metallic spherical shell are 3 cm and 5 cm respectively. It is melted and recast into a solid cylinder of height 10
2 cm. Find the diameter of the base of the 3

cylinder. 66. The largest sphere is carved out of a cube of side 7 cm. Find the volume of the sphere. (use = 3.14) 67. Determine the ratio of the volume of a cube to that of a sphere which will exactly fit inside the cube. 68. A hemi-spherical bowl is made of steel 0.5 cm thick. The inside radius of the bowl is 4 cm. Find the volume of the steel used in making it. 69. A hemi-spherical bowl of internal radius 9 cm contains a liquid. This liquid is to be filled into cylindrical shaped small bottles of diameter 3 cm and height 4 cm. How many bottles are required to empty the bowl? 70. A hemi-sphere of lead of radius 8 cm is cast into a right circular cone of base radius 6 cm. Find the height of the cone. Miscellaneous Questions 71. The height of a cone is 30 cm. A small cone is cut off at the top by a plane parallel to its base. If its volume be
1 of the volume of the given cone, at what height above the base is the section made? 27

72. A circus tent consists of a cylindrical base surmounted by a conical roof. The radius of the cylinder is 20 m. The height of the tent is 63 m and that of the cylindrical base is 42 m. Find the volume of air contained in the tent and the area of canvas used for making it. 73. How many litres of water flows out of pipe having an area of cross-section of 5 cm2 in one minute, if the speed of water in the pipe is 30 cm/sec ? 74. A sphere of diameter 6 cm is dropped into a cylindrical vessel partly filled with water. The radius of the vessel is 6 cm. If the sphere is completely submerged in water, find by how much will the surface level of water be raised. 75. An ice-cream cone has a hemispherical top. If the height of the conical portion is 9 cm and base radius
22 2.5 cm, find the volume of ice-cream in the ice-cream cone. use 7 76. In the given figure, a solid is made of a cylinder with hemispherical ends. If the entire length of the solid is 108 cm and the diameter of the hemispherical ends is 36 cm, find the cost of polishing the surface of the solid at the rate of 7 paisa per cm2.

77. A spherical copper ball of diameter 9 cm is melted and drawn into a wire, the diameter of whose thickness is 2 mm. Find the length of the wire in meters. 78. The difference between the inside and outside surfaces of a cylindrical water pipe 14 m long is 88 m2. If the volume of pipe be 176 m3. Find the inner and outer radii of the water pipe. 26 SURFACE AREAS AND VOLUMES MATHEMATICSIX

79. Water is flowing at the rate of 2.5 km/hr through a circular pipe 20 cm internal diameter, into a circular cistern of diameter 20 m and depth 2.5 m. In how much time will the cistern be filled? 80. A conical vessel of radius 6 cm and height 8 cm is filled with water. A sphere is lowered into the water (see figure), and its size is such that when it touches the sides of the conical vessel, it is just immersed. How much water will remain in the cone after the overflow?

PRACTICE TEST
M.M. : 30 General Instructions : Q. 1-4 carry 2 marks, Q. 5-8 carry 3 marks and Q. 9-10 carry 5 marks each. 1. The floor of a rectangular hall has a perimeter 250 m. If the cost of painting the four walls at the rate of Rs. 10 per m2 is Rs. 15000, find the height of the hall. 2. A cylindrical pillar is 50 cm in diameter and 3.5 m in height. Find the cost of painting the curved surface of the pillar at the rate of Rs. 1250 per m2. 3. A right triangle PQR with sides 5 cm, 12 cm and 13 cm is revolved about the side 12 cm. Find the volume of the solid so obtained. 4. How many litres of milk can a hemispherical bowl of diameter 10.5 cm hold? 5. A jokers cap is in the form of a right circular cone of base 7 cm and height 24 cm. Find the area of the sheet required to make 10 such caps. 6. A hemispherical bowl is made of steel, 0.25 cm thick. The inner radius of the bowl is 5 cm. Find the outer curved surface area of the bowl. 7. A village, having a population of 4000, requires 150 l of water per head per day. It has a tank measuring 20 m 15 m 6 m. For how many days will the water of this tank last ? 8. A patient in a hospital is given soup daily in a cylindrical bowl of diameter 7 cm. If the bowl is filled with soup to a height of 4 cm, how much soup the hospital has to prepare daily to serve 250 patients? 9. The radius and height of a cone are in the ratio 4 : 3. The area of the base is 154 cm2. Find the area of the curved surface. 10. The diameter of a sphere is decreased by 25%. By what percent its curved surface area decrease? Time : 1 hour

ANSWERS OF PRACTICE EXERCISE


1. 868 cm2 5. 100 9. 4 cm, 4 3 cm 13. 4 cm MATHEMATICSIX 2. 17 m 3. Rs. 629 4. 300% 8. (i) 5.45 m2 (ii) 119.90 Rs. 12. 40 m, 30 m 15. (i) 110 m2 (ii) Rs. 4400 27 6. (i) 4250 cm2 (ii) 320 cm 7. 360 cm2 10. 10 m, 15 m, 20 m 11. 7

14. (i) 968 cm2 (ii) 1064.80 cm2 (iii) 2038.08 cm2 SURFACE AREAS AND VOLUMES

16. 3168 cm2 20. 2200 cm2 24. 12 cm 28. 8 cm

17. 440 cm2 21. 2200 cm2 25. 134.2 m 29. 21 cm

18. r = 7 cm, h = 14 cm 22. 1244.57 cm2 26. 63 m 30. Rs. 27.72 33. 324.34 Rs 37. 12 cm, 1728 cm3 41. 6000 44. 539 cm3

19. 3 cm 23. 37 cm 27. Rs. 1155 31. 1 : 16 34. 103.62 cm2 38. 832 cm2 42. 30 km/hr 45. 6 m, 14 m

32. (i) 4 r2 (ii) 4r2 (iii) 1 : 1 35. 3375 cm3 39. 14.7 cm 36. 50 cm 40. 120 cm3

43. (i) 3520 cm2 (ii) 4752 cm2 (iii) 24640 cm3 46. 4620 m3 50. 78.75 cm 54. 1232 cm3 58. 1232 m3 62. 1000 66. 179.5 cm3 70. 28.44 cm 74. 1 cm 78. 1.5 m, 2.5 m 47. 704 cm3 51. 462 m3 55. 13 cm, 5 cm 59. 190.93 cm3 63. 1 : 8 67. 6 : 71. 20 cm 75. 91.66 cm3 79. 10 hours

48. cylindrical tin, 85 cm2 49. 10.8% 52. 6.8 m (approx) 56. 35 cm, 2310 cm2 60. 415.8 cm3, 233.9 cm2 64. 36 m 68. 56.83 cm3 72. 61600 m3, 7102.85 m2 76. Rs. 855.36 80. 188.57 cm3 53. 9 : 16 57. 622.3 cm2 61. 346.5 m2 65. 7 cm 69. 54 73. 9 l 77. 121.5 m

ANSWERS OF PRACTICE TEST


1. 6 m 5. 5500 cm2 9. 192.5 cm2 2. Rs. 68.75 6. 173.25 cm2 10. 43.75% 3. 100 cm3 7. 3 days 4. 0.303 l (approx.) 8. 38.5 l

28

SURFACE AREAS AND VOLUMES

MATHEMATICSIX

CHAPTER

14
STATISTICS

Points to Remember : 1. Facts or figures, collected with a definite pupose, are called Data. 2. Statistics is the area of study dealing with the collection, presentantion, analysis and interpretation of data. 3. The data collected by the investigator himself with a definite objective in mind are known as Primary data. 4. The data collected by somone else, other than the investigator, are known as Secondary data. 5. Any character which is capable of taking reversal different values is called a variable. 6. Each group into which the raw data are condensed is known as class-interval. Each class is bounded by two figures known as its limits. The figure on the left is lower limit and figure on the right is upper limit. 7. The difference between true upper limit and true lower limit of a class is known as its class-size. 8. Mid-value of a class (or class mark) = 9. 10. 11. 12. 13.
upper limit lower limit 2

Class size is the difference between any two successive class marks (mid-values). The difference between the maximum value and the minimum value of the variable is known as Range. The count of number of observations in a particular class is known as its Frequency. The data can be presented graphically in the form of bar graphs, histograms and frequency polygons. The three measures of central tendency for an ungrouped data are : (i) Mean : It is found by adding all the values of the observations and dividing it by the total number of observations. It is denoted by x .
n

So,

x1 x2 ........ xn i 1 . n n For an ungrouped frequency distribution, x


n

xi

f1 x1 f 2 x2 ...... f n x n f1 f 2 ........ f n

f i xi
i 1 n

fi
i 1

(ii) Median : It is the value of the middle-most observation(s).


n 1 If n is an odd number, then median = value of the 2
th

observation.
th th

n n and, if n is an even number, then median = mean of the values of and 1 2 2 (iii) Mode : The mode is the most frequently occurring observation. Empirical formula for calculating mode is given by, Mode = 3 (Median) 2 (Mean)

observations.

MATHEMATICSIX

STATISTICS

173

ILLUSTRATIVE EXAMPLES
Example 1. The relative humidity (in %) of a certain city for a month of 30 days was as follows : 98.1 98.6 99.2 90.3 86.5 95.3 92.9 96.3 94.2 95.1 89.2 92.3 97.1 93.5 82.7 95.1 97.2 93.3 95.2 97.3 96.2 92.1 84.9 90.2 95.7 98.3 97.3 96.1 92.1 89.0 (i) Construct a grouped frequency distribution table with classes 84-86, 86-88 etc. (ii) Which month or season do you think this data is about ? (iii) What is the range of this data? NCERT Solution. (i) Frequency distribution table
Relative humidity (in %) Tally Marks Frequency | 84 86 1 86 88 88 90 90 92 92 94 94 96 96 98 | || || |||| || |||| | |||| || 1 2 2 7 6 7

|||| 98 100 4 Total 30 (ii) Month-June or season-Monsoon (iii) Range = Maximum observation minimum observation = 99.2 84.9 = 14.3 Example 2. The value of upto 50 decimal places is given below : 3.14159265358979323846264338327950288419716939937510 (i) List the digits from 0 to 9 and make a frequency distribution of the digits after the decimal point. (ii) What are the most and the least frequency occurring digits? NCERT Solution. (i) Frequency distribution table
Digit Tally Marks Frequency 0 || 2 1 2 3 4 5 6 7 8 9 Total |||| |||| |||| ||| |||| |||| |||| |||| |||| |||| ||| 5 5 8 4 5 4 4 5 8 50

(ii) Most frequency occuring digits are 3 and 9, and least occurring digit is 0. 174 STATISTICS MATHEMATICSIX

Example 3. The following table gives the life times of 400 neon lamps:
Life time (in hours) 300 - 400 400 - 500 500 - 600 600 - 700 700 - 800 800 - 900 900 - 1000 No. of lamps 14 56 60 86 74 62 48

(i) Represent the given information with the help of a histogram. (ii) How many lamps have a life time of more than 700 hours? NCERT Solution. (i)

(ii) No. of lamps having life time more than 700 hours = 74 + 62 + 48 = 184 Example 4. The following two tables give the distribution of students of two sections according to the marks obtained by them : Section A Marks 0-10 10-20 20-30 30-40 40-50 Frequency 3 9 17 12 9 Marks 0-10 10-20 20-30 30-40 40-50 Section B Frequency 5 19 15 10 1

Represent the marks of the students of both the sections on the same graph by frequency polygon. MATHEMATICSIX STATISTICS 175

Solution.

Required frquency polygon is as follows :

Example 5. Draw histogram of the weekly pocket expenses of 125 students of a school given below :
Weekly pocket expenses (in Rs.) 10 - 20 20 - 30 30 - 50 50 - 60 60 - 90 90 - 100 No. of students 10 15 40 25 30 05

Solution.

Here, we observe that class intervals are unequal, so we will first adjust the frequencies of each class interval. Here, the minimum class size is 10. We know, Adjusted frequency of a class interval Minimum class size frequency of the class class size The adjusted frequency of each class interval is given below :
Weekly pocket expenses Frequency Adjusted frequency 10 10 10 10 - 20 10 10 10 15 15 20 - 30 15 10 10 40 20 30 - 50 40 20 10 25 25 50 - 60 25 10 10 30 10 60 - 90 30 30 10 5 05 90 - 100 05 10

176

STATISTICS

MATHEMATICSIX

So, required histogram is given below.

Example 6. In a mathematics test given to 15 students, the following marks (out of 100) are recorded : 41, 48, 39, 46, 52, 54, 62, 40, 96, 52, 98, 40, 42, 52, 60. Find the mean, median and mode of the above marks.
15

xi
Solution. (i) Mean ( x )
i 1

15

41 48 39 46 52 54 56 62 40 96 52 98 40 42 52 60 15

822 54.8 15 (ii) Arranging the data in the ascending order :

39, 40, 40, 41, 42, 46, 48, 52, 52, 52, 54, 60, 62, 96, 98 here, n = 15, which is odd.
n 1 median = value of 2
th

observation

15 1 2

th

observation = 8th observation = 52

(iii) Since, 52 occurs most frequently i.e. 3 times, so mode is 52. MATHEMATICSIX STATISTICS 177

Example 7. Find the mean salary of 60 workers of a factory from the following table:
Salary (in Rs) 3000 4000 5000 6000 7000 8000 9000 10000 No. of workers 16 12 10 8 6 4 3 1

NCERT

Solution.
Salary (in Rs.) xi 3000 4000 5000 6000 7000 8000 9000 10000 Total No. of workers f i 16 12 10 8 6 4 3 1 f i 60 f i xi 48000 48000 50000 48000 42000 32000 27000 10000 f i xi 305000

Mean ( x )

f i xi 305000 Rs. 5083.33 Ans. fi 60

Example 8. The mean of 5 numbers is 18. If one number is excluded, their mean is 16. Find the excluded number. Solution. Here, n = 5, x 18. Now, x
xi xi 5 18 90 n

So, total of 5 numbers is 90. Let the excluded number be a. Then, total of 4 numbers is 90 a. Mean of 4 numbers
90 a 4

90 a 16 ( Given, new mean = 16) 4 90 a = 64 a = 26 the excluded number is 26. Ans.

178

STATISTICS

MATHEMATICSIX

Example 9. The median of the observations 11, 12, 14, 18, x + 2, x + 4, 30, 32, 35, 41 arranged in ascending order is 24. Find x. Solution. Here, n = 10. Since n is even : th th n n observation 1 observation 2 Median 2 2 5th observation 6th observation 24 2 ( x 2) ( x 4) 2x 6 24 24 24 x 3 2 2 x = 21 Ans. Example 10. Find the mode for the following data : 14, 25, 28, 14, 18, 17, 18, 14, 23, 22, 14, 18. Solution. Arranging the given data in ascending order: 14, 14, 14, 14, 17, 18, 18, 18, 22, 23, 25, 28 Since, 14 occurs maximum number of times (4 times), 14 is the required mode.

PRACTICE EXERCISE
1. Construct a frequency table for the following ages (in years) of 30 students using equal class intervals, one of them being 9-12, where 12 is not included. 18, 12, 7, 6, 11, 15, 21, 9, 13, 8, 15, 17, 19, 22, 14, 21, 8, 23, 12, 17, 6, 18, 15, 23, 16, 22, 9, 21, 16, 11 2. The electricity bills (in Rs.) of 40 houses in a locality are given below : 116, 127, 100, 107, 80, 82, 65, 91, 101, 95, 87, 105, 81, 129, 92, 75, 78, 89, 61, 121, 128, 63, 76, 84, 62, 98, 65, 95, 108, 115, 101, 65, 52, 59, 81, 87, 130, 118, 108, 116 Construct a grouped frequency table. 3. For the following data of weekly wages (in Rs.) received by 30 workers in a factory, construct a grouped frequency distribution table. 258, 215, 320, 300, 290, 311, 242, 272, 268, 210, 242, 258, 268, 220, 210, 240, 280, 316, 306, 215, 236, 319, 304, 278, 254, 292, 306, 332, 318, 300 4. Construct a frequency table, with equal class-intervals from the following data on the weekly wages (in Rs.) of 25 labourers working in a factory, taking one of the class-intervals as 460-500 (500 not included). 580, 625, 485, 537, 540, 425, 637, 605, 607, 430, 611, 632, 600, 640, 638, 612, 584, 440, 536, 515, 449, 480, 556, 561, 508 5. Given below are two cumulative frequency distribution tables. Form a frequency distribution table for each of these. (i) (ii)
Ages ( in years ) No . of persons Below 10 15 Below 20 28 Below 30 Below 40 Below 50 Below 60 39 60 73 80 Marks obtained More than 10 More than 20 More than 30 More than 40 More than 50 More than 60 No. of Students 0 17 27 39 52 60

MATHEMATICSIX

STATISTICS

179

6. On a certain day, the temperature in a city was recorded as under : Time 5 a.m. 8 a.m. 11a.m. 3 p.m. 6 p.m. Temperature (in C ) 20 24 26 22 18 Draw a bar graph to represent the above data. 7. Read the bar graph given below and answer the questions given below : (i) What information is given by the bar graph? (ii) In which year was the production maximum? (iii) After which year was there a sudden fall in the production? (iv) Find the ratio between the maximum and minimum production during the given period.

8. The table given below shows the number of blinds in a village :


Age group 0 20 20 40 40 60 60 80 80 100 No . of blinds 5 9 10 4 2

Total 30 Draw a histogram to represent the above data. 9. The following table shows the average daily earnings of 40 general stores in a market, during a certain week. Daily earning (in Rs.) No. of stores

600 - 650 650 - 700 700 - 750 750 - 800 800 - 850 850 - 900 Total

5 10 2 7 12 4 40

180

STATISTICS

MATHEMATICSIX

Draw a histogram to represent the above data. 10. The following table gives the heights of 50 students of a class. Draw a frequency polygon to represent this.
Height (in cm) No . of Students 120 -135 1 135 - 150 18 150 -165 165 - 180 180 -195 Total 23 7 1 50

11. In a study of diabetic patients in a village, the following observations were noted. Represent the given data by a frequency polygon.
Age (in Years) No. of Patients 10 - 20 20 - 30 30 - 40 40 - 50 50 - 60 60 - 70 Total 2 6 12 20 9 4 53

12. Draw a histogram and a frequency polygon on the same graph to represent the following data :
Weight (in cm) No . of Persons 40 - 50 30 50 - 60 25 60 - 70 70 - 80 80 - 90 Total 40 30 10 135

13. Draw a histogram to represent the following frequency distribution.


Class Interval Frequency 10 - 15 6 15 - 20 20 - 30 30 - 50 50 - 80 9 10 8 18

MATHEMATICSIX

STATISTICS

181

14. Draw a histogram for the marks of students given below :


Marks No . of Students 0 - 10 8 10 - 30 32 30 - 45 45 - 50 50 - 60 Total 18 10 6 74

15. The runs scored by two teams A and B on the first 120 balls in a cricket match are given below :
Number of balls Team A Team B 0 - 12 10 4 12 - 24 12 2 24 - 36 36 - 48 48 - 60 60 - 72 72 - 84 84 - 96 96 - 108 108 - 120 4 20 10 12 6 8 16 20 16 18 8 10 12 20 12 4

Represent the data of both the teams on the same graph with the help of frequency polygons. 16. Find the mean for each of the following sets of numbers : (i) 25, 12, 37, 19, 43, 40, 11 (ii) 6.2, 4.9, 7.1, 2.9, 5.7, 8.3 2 2 2 2 2 2 (iii) 1 , 2 , 3 , 4 , 5 , 6 (iv) 13, 23, 33, 43, 53 17. Calculate the mean ( x ) for each of the following distribution : (i) x f 2 4 6 8 10 1 3 5 2 6 (ii) x f 5 10 15 20 25 30 6 3 2 5 1 3

18. The following table shows the number of accidents met by 120 workers in a factory during a month : No. of accidents 0 1 2 3 4 5 No. of workers 36 34 21 25 3 1 Find the average number of accidents per workers. 19. The marks obtained out of 50 by 80 students in a test are given below : Marks 15 20 22 24 25 30 33 38 35 No. of Students 5 8 7 16 12 18 7 3 4 Calculate the average marks. 20. If the mean of the following data is 18.75, find the value of p. xi 10 15 p 25 30 f i 5 10 7 8 2 182 STATISTICS MATHEMATICSIX

21. The average of height of 30 boys out of a class of 50, is 160 cm. If the average height of the remaining boys is 165 cm, find the average height of the whole class. 22. The average of six numbers is 30. If the average of first four is 25 and that of last three is 35, find the fourth number. 23. The mean of 100 observations was calculated as 40. It was found later on that one of the observations was misread as 83 instead of 53. Find the corrected mean. 24. The mean of 10 numbers is 18. If 3 is subtracted from every number, what will be the new mean?
n

25. If x is the mean of n observations x1, x2, ...., xn, then prove that

( xi x ) 0 i.e. the algebraic sum of


i 1

deviations from mean is zero. 26. Find the median of following data : (i) 14, 6, 18, 9, 23, 22, 10, 19, 24 (ii) 17, 13, 28, 19, 23, 22, 12, 32 27. The numbers 5, 7, 10, 12, 2x 8, 2x + 10, 35, 41, 42, 50 are arranged in ascending order. If their median is 25, find the value of x. 28. Find the median of the following observations : 46, 64, 58, 87, 41, 77, 35, 55, 90, 92, 33. If 92 is replaced by 99 and 41 by 43 in the above data, find the new median. 29. Find out the mode of the following data : (i) 14, 28, 19, 25, 14, 31, 17, 14, 12, 27 (ii) 8.3, 8.9, 8.1, 8.7, 8.9, 7.9, 8.7, 8.9, 8.1 30. Given below is the number of pairs of shoes of different sizes sold in a day by the owner of the shop. Size of shoe 1 2 3 4 5 6 7 8 9 No. of pairs sold 2 2 3 4 5 5 6 9 1 What is the modal shoe size?

PRACTICE TEST
MM : 15 General Instructions : Each question carry 3 marks. 1. Three coins were tossed 30 times simultaneously. Each time the number of heads occuring was noted down as follows: 0 1 2 2 1 2 3 1 3 0 1 3 1 1 2 2 0 1 2 1 3 0 0 1 1 2 3 2 2 0 Prepare a frequency distribution table for this data. 2. A random survey of the number of children of various age groups playing in a park was found as follows: Age ( in years ) 1 - 2 2 - 3 3 - 5 5 - 7 7 - 10 10 - 15 15 -17 No. of children 5 3 6 12 9 10 4 Draw a histogram to represent the data above. MATHEMATICSIX STATISTICS 183 Time :
1 2

hour

3. Find mean ( x ) for the following distribution: x 10 15 20 25 30 35 f 3 2 4 7 3 1 4. The following observations have been arranged in ascending order. If the median of the data is 63, find the value of x. 29, 32, 48, 50, x, x + 2, 72, 78, 84, 95 5. The marks obtained by 80 students in a test are given. Find the modal marks. Marks 4 12 20 28 36 44 No. of students 8 10 15 24 15 8

ANSWERS OF PRACTICE EXERCISE


1. Class 6 - 9 9 -12 12 - 15 15 -18 18 - 21 21 - 24 Frequency 5 4 4 7 3 7

2.

Class 50 - 60 60 - 70 70 - 80 80 - 90 90 - 100 100 - 110 110 - 120 120 - 130 130 -140 Frequency 2 6 3 8 5 7 4 4 1

3.

Weekly Wages 200 - 220 220 - 240 240 - 260 260 - 280 280 - 300 300 - 320 320 - 340 ( in Rs.) No. of 4 2 6 4 3 9 2 workers

4.

Weekly Wages 420 - 460 460 - 500 500 - 540 540 - 580 580 - 620 620 - 660 ( in Rs.) No. of 4 2 4 3 7 5 workers

5. (i)

Age (in year ) 0 - 10 10 - 20 20 - 30 30 - 40 40 - 50 50 - 60 No. of persons 15 13 11 21 13 7

(ii)

Marks obtained 0 - 20 20 - 40 40 - 60 60 - 80 80 - 100 No. of Students 8 13 12 10 17

184

STATISTICS

MATHEMATICSIX

6.

7. (i) The given bar graph shows the production (in million tonnes) of food grains during the period from 2000 to 2004. (ii) 2002 (iii) 2000 (iv) 5 : 2

8.

9.

10.

11.

MATHEMATICSIX

STATISTICS

185

6 5

12.

13.
3 2

14.

15.

186

STATISTICS

MATHEMATICSIX

16. (i) 26.71 17. (i) 7.05 21. 162 cm 26. (i) 18 29. (i) 14

(ii) 5.85 (ii) 15.25 22. 25 (ii) 20.5 (ii) 8.9

(iii) 15.16 18. 1.4 23. 39.7 27. 12 30. 8

(iv) 45 19. 26.7 (approx) 24. 15 28. 58, 58 20. 20

ANSWERS OF PRACTICE TEST


3. 22 4. 62 5. 28

MATHEMATICSIX

STATISTICS

187

CHAPTER

15
PROBABILITY

Points to Remember : 1. An activity which gives a result is called an experiment. 2. An experiment which can be repeated a number of times under the same set of conditions, and the outcomes are not predictable is called a Random Experiment. 3. Performing an experiment is called a trial. 4. Any outcome of an experiment is known as an event. 5. In n trials of a random experiments, if an event E happens m times, then the probability of happening of E is given by,
P(E ) Number of outcomes favour to E m Total number of possible outcomes n

6. For any event E, which is associated to an experiment, we have 0 P ( E ) 1. 7. If E1, E2, E3, ....., En are n elemantary events associated to a random experiment, then P(E1) + P(E2) + P(E3) + ........... + P(En) = 1

ILLUSTRATIVE EXAMPLES
Example 1. In a cricket match, a batswoman hits a boundary 6 times out of 30 balls she plays. Find the probability that she did not hit the boundary. NCERT Solution. Since, she hits a boundary 6 times, it means she has missed 30 6 = 24 times. Required probability = P (she did not hit the boundary) Number of time she did not hit the boundary Total number of balls she plays
24 4 0.8 Ans. 30 5 Example 2. Three coins are tossed simultaneously 200 times with the following frequencies of different outcomes:

Outcome Frequency

3 heads 2 heads 1 heads no heads 23 72 77 28

Solution.

If the three coins are simultaneously tossed again, compute the probability of 2 heads coming up. NCERT Out of total of 200 outcomes, 2 heads come for 72 times. P (2 heads coming up) 72 9 Ans. 200 25

188

PROBABILITY

MATHEMATICSIX

Example 3. An organisation selected 2400 families at random and surveyed them to determine a relationship between income level and the number of vehicles in a family. The information gathered is listed in the table below: Monthly income (in Rs.) Less than 7000 7000-10000 10000-13000 13000-16000 16000 or more Vehicles per family 1 2 160 305 535 469 579 25 27 29 59 82

0 10 0 1 2 1

Above 2 0 2 1 25 88

Solution.

Suppose a family is chosen. Find the probability that the family chosen is : (i) earning Rs. 10000-13000 per month and owning exactly 2 vehicles. (ii) earning Rs. 16000 or more per month and owning exactly 1 vehicle. (iii) earning less than Rs. 7000 per month and does not own any vehicle. (iv) earning Rs. 13000-16000 per month and owning more than 2 vehicles. (v) owning not more than 1 vehicle. NCERT The total number of families = 2400 (i) Number of families earning Rs. 10000 13000 per month and owning exactly 2 vehicles = 29 Required probability 29 2400 (ii) Number of families earning Rs. 16000 or more per month and owning exactly 1 vehicle = 579 Required probability 579 193 2400 800 (iii) Number of families earning less than Rs. 7000 per month and does not own any vehicle = 10 Required probability 10 1 2400 240 (iv) Number of families earning Rs. 13000 16000 per month are owning more than 2 vehicles = 25.

Required probability 25 1 2400 96 (v) Number of families owning not more than 1 vehicle = families having no vehicle + families having 1 vehicle = (10 + 0 + 1 + 2 + 1) + (160 + 305 + 535 + 469 + 579) = 2162 Required probability 2162 1031 2400 1200 Example 4. A die is throw 300 times and the outcomes are noted in as given below : Outcome 1 2 3 4 5 6

Frequency 70 45 47 30 35 48 If a die is thrown at random, find the probability of getting. (i) 1 (ii) 2 (iii) 3 (iv) 4 MATHEMATICSIX PROBABILITY (v) 5 (vi) 6 189

Solution.

Total number of trials = 300 (i) P (getting 1) 70 7 0.23 300 30 (ii) P (getting 2) 45 0.15 300 (iii) P (getting 3) 47 0.156 0.16 300 (iv) P (getting 4) 30 0.1 300 (v) P (getting 5) 35 0.116 300

48 0.16 300 Example 5. The table given below shows the marks obtained by 90 students of a class in a test with maximum marks 100.

(iv) P (getting 6)

Marks 0 15 15 30 30 45 45 60 60 75 Above 75 No. of students 7 14 18 25 17 9 A student of the class is selected at random. Find the probability that student gets : (i) less than 30% marks (ii) 60 or more marks (iii) marks between 45 and 75. (iv) distinction Total number of students = 90 (i) P (getting less than 30% marks) 7 14 21 0.233 90 90 (ii) P (getting 60 or more marks) 17 9 26 0.288 90 90 (iii) P (getting marks between 45 and 75) 25 17 42 0.466 90 90 (iv) P (getting distinction) 9 0.1 90 Example 6. A tyre manufacturing company kept a record of the distance covered before a type to be replaced. Following table shows the result of 1000 cases. Distance (in km) less than 400 400 900 900 1400 more than 1400 No. of tyres 200 335 375 90 If you buy a tyre of this company, what is the probability that : (i) it will need to be replaced before it has covered 400 km. (ii) it will last more than 900 km? 190 PROBABILITY MATHEMATICSIX

Solution.

Solution.

(iii) it will need to be replaced? (iv) it will not need to be replaced at all? (v) it will need to be replaced after it has covered somewhere between 400 km and 1400 km? We have, total number of trials = 1000 (i) The number of tyres that needs to be replaced before it has covered 400 km = 200. Required probability 200 0.2 1000 (ii) The number of tyres that last more than 900 km = 375 + 90 = 465 Required probability 465 0.465 1000 (iii) Since, all the tyres we have considered to be replaced. Required probability 1000 1 1000 (iv) The number of tyres that do not need to be replaced at all = 0
0 0 1000 (v) The number of tyres which require replacement after covering somewhere between 400 km and 1400 km is 335 + 375 = 710.

Required probability

Required probability 710 0.71 1000 Example 7. A bag contains 3 red balls, 5 black balls and 4 white balls. A ball is drawn at random from the bag. What is the probability that the ball drawn is: (i) White (ii) not black (iii) Red or white Solution. Total number of balls = 3 + 5 + 4 = 12. (i) P (white ball) 4 1 12 3 (ii) P (not black) = 1 P (black) 1 (iii) P (red or white)
5 12 5 7 12 12 12

3 4 7 . 12 12 Example 8. Cards marked with the numbers 2, 3,...., 101 are placed in a box and mixed thoroughly. One card is drawn from this box. Find the probability that the number on the card is: (i) an even number (ii) a number less than 14 (iii) a perfect square number (iv) a prime number less than 20. Solution. Total numbers = 100 (i) Total number of even numbers = 50

P (an even number) 50 1 100 2 (ii) P (a number less than 14)


P ( 2 , 3,....,13) 12 3 100 25

MATHEMATICSIX

PROBABILITY

191

(iii) P (a perfect square number) = P (4, 9, 16, ....., 100) 9 100 (iv) P (a prime number less than 20) = P (2, 3, 5, 7, 11, 13, 17, 19)
8 2 100 25 Example 9. Find the probability that a leap year, selected at random will have 53 sundays. Solution. A leap year has 366 days. 52 weeks = 52 7 = 364 days Days left = 366 364 = 2 52 weeks means there will be 52 sundays. From remaining 2 days, the possibilities are as follows : (Sunday, Monday), (Monday, Tuesday), (Tuesday, Wednesday), (Wednesday, Thursday), (Thursday, Friday), (Friday, Saturday) and (Saturday, Sunday). So, from 7 possibilities, we have 2 cases in which we have 53rd sunday.

2 Ans. 7 Example 10. A bag contains 10 white balls and x black balls. If the probability of drawing a black ball is double that of a white ball, find x. Solution. Total number of balls = 10 + x.

Required probability

P (black ball)

x 10 x

10 10 x according to given question, P (black ball) = 2 P (white ball)

and, P (white ball)

x 2 10 10 x 10 x x = 20 Ans. ( 10 + x 0)

PRACTICE EXERCISE
1. A number is chosen from 1 to 20. Find the probability that the number chosen is : (i) a prime number (ii) a composite number (iii) a square number (iv) an odd number (v) an even number (vi) number between 7 and 14 2. A bag contains 9 red and 6 blue balls. Find the probability that a ball drawn from a bag at random is (i) Red ball (ii) blue ball 3. In a sample of 500 items, 120 are found to be defective. Find the probability that the item selected at random is (i) defective (ii) non-defective 4. In a school of 1800 students, there are 875 girls. Find the probability that a student chosen at random is (i) a boy (ii) a girl 5. In a cricket match, a batsman hit a boundary 12 times out 45 balls he plays. Find the probability that he did not hit a boundary. 192 PROBABILITY MATHEMATICSIX

6. A coin is tossed 700 times and we get head : 385 times; tail : 315 times. When a coin is tossed at random, what is the probability of getting : (i) a head? (ii) a tail? 7. Two coins are tossed 600 times and we get two heads : 138 times, one head : 192 times ; no head : 270 times. When two coins are tossed at random, what is the probability of getting : (i) 2 heads? (ii) 1 head? (iii) no head? 8. Three coins are tossed 250 times and we get: 3 heads : 46 times; 2 heads : 56 times; 1 head : 70 times; 0 head : 78 times. When three coins are tossed at random, what is the probability of getting : (i) 3 heads ? (ii) 2 heads? (iii) atleast 2 heads? (iv) atmost 2 heads? 9. A die is thrown 300 times and the outcomes are noted as given below : Outcomes 1 2 3 4 5 6 Frequencies 58 75 52 39 42 34 When a die is thrown at random, what is the probability of getting a: (i) 4 (ii) 6 (iii) number less than 3 (iv) number which is prime 10. In a survey of 350 ladies, it was found that 235 like coffee, while rest of them dislike it. Find the probability that a lady chosen at random: (i) likes coffee (ii) dislikes coffee. 11. On one page of a telephone directory, there are 200 phone numbers. The frequency distribution of their units digit is given below : Units digit 0 1 2 3 4 5 6 7 8 9 Frequency 21 20 21 21 22 25 22 17 15 16 One of the numbers is chosen at random from the page. What is the probability that the units digit of the chosen number is : (i) 5 (ii) 8 (iii) an even number (iv) an odd number 12. The blood groups of 30 students of class IX are recorded as follows : A, B, O, AB, O, A, O, O, B, A, O, A, B, O, O, A, O, B, A, B, O, A, B, AB, O, O, A, A, O, AB A student is selected at random from the class for blood donation. Find the probability that the blood group of the student chosen is: (i) A (ii) B (iii) AB (iv) O 13. Following are the ages (in years) of 350 patients, getting medical treatment in a hospital. Age (in years) 10 20 20 30 30 40 40 50 50 60 60 70 No. of Patients 85 50 55 75 55 30 One of the patients is selected at random. Find the probability that his age is : (i) 40 years or more but less than 50 years. (ii) 30 years or less than it. (iii) less than 10 years. (iv) 50 years or more. MATHEMATICSIX PROBABILITY 193

14. Given below is the frequency distribution of wages (in Rs.) of 40 workers in a certain factory: Wages (in Rs) 100 120 120 140 140 160 160 180 180 200 200 220 220 240 No. of workers 4 3 6 5 9 8 5 A workers is selected at random. Find the probability that his wages are: (i) less than Rs. 160. (ii) atleast Rs. 180. (iii) more than or equal to Rs. 140 but less than Rs. 200. (iv) more than Rs. 200. 15. The following table gives the life time of 500 neon lamps:
Life time 300 400 400 500 500 600 600 700 700 800 800 900 900 1000 (in hours) No. of 24 66 75 96 89 72 78 Lamps

A lamp is selected at random. Find the probability that the life time of the selected lamp is: (i) less than 400 hours (ii) atleast 800 times (iii) atmost 600 hours (iv) between 500 hours to 900 hours.

PRACTICE TEST
MM : 15 General Instructions : Each Questions carry 3 marks. 1. Two coins are tossed simultaneously 350 times with the following frequencies of different outcomes: Two heads : 105 times; One head : 125 times; no head : 120 times. Find the probability of getting : (i) 2 heads (ii) atleast one head 2. Following table shows the marks scored by 80 students in a mathematics test of 100 marks. Marks 0 20 20 30 30 40 40 50 50 60 60 70 70 100 No. of students 5 8 10 18 15 18 6 Find the probability that a student obtained: (i) less than 40% marks. (ii) 60 or more marks. 3. 1200 families with 2 children were selected at random and the following data were recorded; No. of girls in a family 0 1 2 No. of families 255 515 430 194 PROBABILITY MATHEMATICSIX Time :
1 2

hour

If a family is chosen at random, compute the probability that it has : (i) no girl (ii) 2 girls (iii) at most one girl (iv) at least one girl 4. The table given below shows the ages of 90 teachers in a school. Age (in years) 18 29 30 39 40 49 50 59 No. of teachers 8 32 40 10 A teacher from this school is chosen at random. What is the probability that the selected teacher is : (i) 40 or more than 40 years old ? (ii) age less than 40 years ? 5. An insurance company selected 2000 drivers at random, in a particular city to find out a relationship between age and accidents. The data obtained are given in the following table : Age of drivers (in years) 18-29 30-50 above 50 0 440 505 360 Accidents in one year 1 2 160 125 45 110 60 35 3 61 22 15 over 3 35 18 9

Find the probabilities of the following events for a driven chosen at random from the city: (i) being 18-29 years of age and exactly 2 accidents in one year. (ii) being 30-50 years of age and having one or more accidents in a year. (iii) having no accidents in one year.

ANSWERS OF PRACTICE EXERCISE


1. (i) 2. (i) 4. (i) 6. (i) 8. (i) 9. (i) 10. (i)
2 5 3 5 37 72 11 20 23 125
13 100

(ii) (ii) (ii) (ii) (ii) (ii) (ii)

11 20 2 5 35 72 9 20 28 125
17 150

(iii)

1 5 6 25

(iv) (ii)

3 10 19 25

3. (i) 5.
11 15

7. (i) (iii) (iii)

23 100

(ii) (iv) (iv)

32 100 102 125


169 300

(iii)

9 20

51 250
133 300

47 70

23 70

MATHEMATICSIX

PROBABILITY

195

11. (i) 12. (i) 13. (i) 14. (i) 15. (i)

1 8 3 10

(ii) (ii) (ii) (ii) (ii)

3 40 1 5

(iii) (iii)

101 200 1 10

(iv) (iv) (iv) (iv) (iv)

99 200 2 5

3 14
13 40 6 125

27 70 11 20 3 10

(iii) 0 (iii) (iii)


1 2
33 100

17 70 13 40 83 125

ANSWERS OF PRACTICE TEST


1. (i) 2. (i) 3. (i) 4. (i) 5. (i)
3 10
33 80 17 80

(ii) (ii) (ii) (ii) (ii)

23 35
3 10 43 120

(iii)

77 120

(iv)

63 80

5 9 11 200

4 9 9 80

(iii)

261 400

196

PROBABILITY

MATHEMATICSIX

You might also like